Jos maapalloa tarkkaillaan?

Karvanoppa1

Eli katsellaan vaikka 10000 valovuoden päästä kaukoputkella niin mitä aikaa ne näkevät meidän ajanlaskumme mukaan?

184

1133

    Vastaukset

    Anonyymi (Kirjaudu / Rekisteröidy)
    5000
    • Näkevät Maapallon kuin se oli 10 000 vuotta sitten, luonnollisesti.

      • nualesammakkoa

        Kunnioitukseni hra Kollimaattoria kohtaan kasvoi jälleen huimasti tämän loistavan vastauksen luettuani.


      • bv35345
        nualesammakkoa kirjoitti:

        Kunnioitukseni hra Kollimaattoria kohtaan kasvoi jälleen huimasti tämän loistavan vastauksen luettuani.

        Vastaus oli lyhyt, ytimekäs ja oikea. Oma kommenttisi tosin tuoksahtaa jostain syystä sarkasmilta.


      • Fotoninnussia

        Hmmm
        Vähän vaille?
        Tos 10 000 vuodessa avaruus on laajentunu jonku verran, ni pikkasen jää vajaaks?


      • Ohman
        Fotoninnussia kirjoitti:

        Hmmm
        Vähän vaille?
        Tos 10 000 vuodessa avaruus on laajentunu jonku verran, ni pikkasen jää vajaaks?

        10 000 valovuoden päässä ollaan vielä meidän galaksissamme Linnunradassa. Linnunrata ei laajene, sen gravitaatio pitää sen koossa vsaikka avaruus laajenee.

        Ohman


      • Gravitaationnussia
        Ohman kirjoitti:

        10 000 valovuoden päässä ollaan vielä meidän galaksissamme Linnunradassa. Linnunrata ei laajene, sen gravitaatio pitää sen koossa vsaikka avaruus laajenee.

        Ohman

        Riippuu mihin suuntaan lähtee?
        Jos lähtee "ylös" tai "alaspäin" (linnunradan kiekon tasoon nähden) niin ollaan jo paljon ennen tota linnunradan ulkopuolella. (Mut toki gravitaatio vaikuttaa varmaan vielä sielläkin, joten: Ehkei sittenkään. Myönnän.)


      • Ohman
        Gravitaationnussia kirjoitti:

        Riippuu mihin suuntaan lähtee?
        Jos lähtee "ylös" tai "alaspäin" (linnunradan kiekon tasoon nähden) niin ollaan jo paljon ennen tota linnunradan ulkopuolella. (Mut toki gravitaatio vaikuttaa varmaan vielä sielläkin, joten: Ehkei sittenkään. Myönnän.)

        Ei olla ulkopuolella. Nykykäsityksen mukaan koko galaksimme muodostuu suuresta "halosta", jonka pieni näkyvä osa on tuo taivaalla loistava valovuo jota Linnunradaksi kutsuttiin.
        Tässä halossa on "stellar halo", joka on litistynyt sferoidi ja sen säde on > 20 kpc (kiloparsekkia).
        Tämä s-halo sisältyy vielä suurempaan pimeän aineen haloon joka havaitaan vain gravitaationsa avulla.

        Mutta itse tuosta laajenemisesta näytämme olevan samaa mieltä.

        Ohman


      • Halonnussia
        Ohman kirjoitti:

        Ei olla ulkopuolella. Nykykäsityksen mukaan koko galaksimme muodostuu suuresta "halosta", jonka pieni näkyvä osa on tuo taivaalla loistava valovuo jota Linnunradaksi kutsuttiin.
        Tässä halossa on "stellar halo", joka on litistynyt sferoidi ja sen säde on > 20 kpc (kiloparsekkia).
        Tämä s-halo sisältyy vielä suurempaan pimeän aineen haloon joka havaitaan vain gravitaationsa avulla.

        Mutta itse tuosta laajenemisesta näytämme olevan samaa mieltä.

        Ohman

        No ainaki wiki antais linnunradan paksuudeks aurinkokunnan kohdalla 3 000 -10 000 valovuotta, joten kyl mun mielestä sillon melkolailla ollaan? Ja muistaakseni linnunrata sijatsi vaan pikkase keskilinjan yläpuolella?
        Ja eikös se halo ollu suunnilleen vaan ton keskipullistuman kohdalla? Ja me ei kyllä olla lähelläkään sitä, vaan täällä "lonkeroitten" keskivaiheilla. Ja mun tietääkseni noilla tiedemiehilläki vähän huonot havainnot noista linnunradan ulko-osista, ku niitäkino kohtalaisen vaikee tutkiakin, ku tää perkeleen hassakka on tässä itse näkömaskina ni sinne ei tahdonoikeen nähdä. Mut jotain uusia, pienehköjä "lonkeroitahan" ne sielt tais ainakin löytää. Pimeeseen aineeseen mä en o pruukannu ottaa mitää kantaa, enneku se on ihan todistettu asia ja sitä kyetään jollain havaitsemaankin. Ja toi halokin on sit kuitenki niin paljon harvempaa ainetta, et ei se kyl vedä mitenkä painovoimallaan vertoja koko tän hyrrän massalle, et ... tuskinpa sillä juurikaann vaikutusta on? Ja eipä täst pitkä matka (itseasiassa ollaan kiinni) noihin pariin snadimpaan satelliitti-galaksiin, joten pelkästään niitten vetovpima vaikuttaa enemmän ku toi halo, jos sattuu väärään suuntaan lähtemään. Eikä noi Magellanitkaan keuheen kaukana ole, joten niittenki vetovoima alkaa vaikuttaa suht äkkiä (jos 10 000 valovuoden matkaa nyt voi kutsua "äkilliseksi" :P :D )

        Ja juu ollaan. Ihan liian "lyhyt" matkahan toi siihen vielä olikin.


      • älytön_nopeus

        Miten niin? Eikö aika pysähdykään valon nopeudessa?


      • HassuValo

        Valon nopeus avaruuden tilassa on 300 000 kilometriä sekunnissa. Valovuosi on matka, jonka valo kulkee yhdessä vuodessa. Jos siis lähdemme raketilla valonnopeudella ja matkataan 10 000 valovuoden päähän, näemme maapallon suurinpiirtein sellaisena kun se oli meidän sieltä lähtiessämme. Kun palaamme sitten valonnopeudella takaisin maapallolle, on lähdöstä kulunut jo tuplasti sama aika. Tuskin he tunnistavat planeetta enää samaksi, saati löydä siltä ainuttakaan ihmistä. Jäljellä on pelkkä saastunut planeetta ja hiljalleen katovat sivilisaation rauniot. Eli jos siis lähetämme ulkoavaruuteen ihmisiä, meidän on ratkaistava se miten heidät saa takaisin ajassa taaksepäin tai he joutuvat kauas tulevaisuuteen. Sama juttu on auringossa, eli jos katsomme aurinko taivaalla, näemme sen sellaisena kuin se oli vielä kahdeksan minuuttia sitten. Maapallo on Auringosta keskimäärin noin 150 miljoonan kilometrin etäisyydellä.


      • 20000vuotta.pielessä

        "..Näkevät Maapallon kuin se oli 10 000 vuotta sitten, luonnollisesti..."

        Väärin meni . He näkevät maapallon 10 000 vuotta vanhempana, eli vuonna 12017.
        Jos he näkisivät maapallon, kuin se oli 10 000 vuotta sitten, heidän olisi pitäny kiikaroida maapalloa 20 000 vuotta sitten.


      • 20000vuotta.pielessä kirjoitti:

        "..Näkevät Maapallon kuin se oli 10 000 vuotta sitten, luonnollisesti..."

        Väärin meni . He näkevät maapallon 10 000 vuotta vanhempana, eli vuonna 12017.
        Jos he näkisivät maapallon, kuin se oli 10 000 vuotta sitten, heidän olisi pitäny kiikaroida maapalloa 20 000 vuotta sitten.

        Miten toi sun logiikkas tuossa kulkee? Mä en saanut kiinni sun ajatuksesta.


      • 20000vuotta.pielessä
        planeetta kirjoitti:

        Miten toi sun logiikkas tuossa kulkee? Mä en saanut kiinni sun ajatuksesta.

        Mietippä sitä. !

        No väännetään sitten rautalangasta.. Jos 10 000 valovuoden päässä joku meidät näkee tässä nykyajasa eli vuonna 2017, on hänen täytynyt zoomailla tänne 10 000 vuotta sitten. Ja jos hän näkee maapallon 10 000 vuotta sitten niin, 10 000 10 000 = 20 000v.

        ps. Ajan käsite avaruudessa näyttää olevan joillekkin vaikeaa..


      • AnnaMunKaikkiKestää
        20000vuotta.pielessä kirjoitti:

        Mietippä sitä. !

        No väännetään sitten rautalangasta.. Jos 10 000 valovuoden päässä joku meidät näkee tässä nykyajasa eli vuonna 2017, on hänen täytynyt zoomailla tänne 10 000 vuotta sitten. Ja jos hän näkee maapallon 10 000 vuotta sitten niin, 10 000 10 000 = 20 000v.

        ps. Ajan käsite avaruudessa näyttää olevan joillekkin vaikeaa..

        Ai se meneekin NOIN!
        Ensin vähennetään ajasta toi matka, ja sit vähennetään se uudestaan kun vastaanotetaan se valo!

        Juu juu! No NYT ymmärsin! :D


      • planeetta_ei.kirj
        AnnaMunKaikkiKestää kirjoitti:

        Ai se meneekin NOIN!
        Ensin vähennetään ajasta toi matka, ja sit vähennetään se uudestaan kun vastaanotetaan se valo!

        Juu juu! No NYT ymmärsin! :D

        Mulla on vielä prosessointiongelmaa, mut enköhän mä saa sen vietyä loppuun tässä illan aikana :-/
        Hei kiitos selityksestä! Luulen, että ymmärsin aloituksen eritavalla, enkä saa käännettyä aivoa uuteen asentoon. Aivojumi.


      • Kapteenihaddock
        planeetta_ei.kirj kirjoitti:

        Mulla on vielä prosessointiongelmaa, mut enköhän mä saa sen vietyä loppuun tässä illan aikana :-/
        Hei kiitos selityksestä! Luulen, että ymmärsin aloituksen eritavalla, enkä saa käännettyä aivoa uuteen asentoon. Aivojumi.

        Eihän tuota kuulukaan ymmärtää, koska selitys on väärin.
        Jos tällä hetkellä joku meitä katselee tuolta 10000 valovuoden päässä, niin he näkevät valon joka on täältä heijastunut 10000 vuotta sitten. Eli näkevät maapallon niin kuin se oli 10000 vuotta sitten. Jos me odotamme 10000 vuotta ja kysymme täysin saman kysymyksen sillä hetkellä, niin tuo tarkkailija näkisi maan niin kuin se on nyt meillä täällä vuonna 2017.


      • älytön_nopeus
        HassuValo kirjoitti:

        Valon nopeus avaruuden tilassa on 300 000 kilometriä sekunnissa. Valovuosi on matka, jonka valo kulkee yhdessä vuodessa. Jos siis lähdemme raketilla valonnopeudella ja matkataan 10 000 valovuoden päähän, näemme maapallon suurinpiirtein sellaisena kun se oli meidän sieltä lähtiessämme. Kun palaamme sitten valonnopeudella takaisin maapallolle, on lähdöstä kulunut jo tuplasti sama aika. Tuskin he tunnistavat planeetta enää samaksi, saati löydä siltä ainuttakaan ihmistä. Jäljellä on pelkkä saastunut planeetta ja hiljalleen katovat sivilisaation rauniot. Eli jos siis lähetämme ulkoavaruuteen ihmisiä, meidän on ratkaistava se miten heidät saa takaisin ajassa taaksepäin tai he joutuvat kauas tulevaisuuteen. Sama juttu on auringossa, eli jos katsomme aurinko taivaalla, näemme sen sellaisena kuin se oli vielä kahdeksan minuuttia sitten. Maapallo on Auringosta keskimäärin noin 150 miljoonan kilometrin etäisyydellä.

        Mutta jos aika pysähtyy valon nopeudessa niin tällöinhän me näemme millainen aurinko on nyt eikä sellaisena kuin se oli kahdeksan minuuttia sitten? Eli pysähtyykö aika valon nopeudessa vai ei? Myönnettäköön että käsite 'aika' on aika kimuranttinen.


      • planeetta_ei.kirj
        älytön_nopeus kirjoitti:

        Mutta jos aika pysähtyy valon nopeudessa niin tällöinhän me näemme millainen aurinko on nyt eikä sellaisena kuin se oli kahdeksan minuuttia sitten? Eli pysähtyykö aika valon nopeudessa vai ei? Myönnettäköön että käsite 'aika' on aika kimuranttinen.

        Virallinen selitys alla :-)

        Kyllä se aika pysähtyy tai pysähtyi, tavallaan, mutta vaan sille fotonille.

        Fotoni on massaton energiakvantti, joka kantaa mukanaan pienen sähkömagneettisen varauksen, jonka suuruus määräytyy taajuuden mukaan.

        Sen tehtävä on toimia välittäjänä ja välitysnopeus on c (se noin 300000km/s).

        Mikään ei liiku sitä c:tä nopeammin eikä mikään millä on massaa voi liikkua nopeudella c vaan aina hitaammin (koska mistään ei löydy niin paljoa energiaa, että massa saataisiin kiihtymään nopeuteen c). Valonnopeus c on siis absoluuttinen raja, jonka yli ei voi mennä.

        Näin sanoo teoria. Tai teoria ennustaa, että on olemassa raja ja rajaksi on havaittu olevan c.

        Suhteellisuusteorian mukaan aika on suhteellinen "havaitsijalle". Jokaisella "havaitsijalla" on oma koordinaatistonsa (sanotaan inertiaalikoordinaatistoksi). On olemassa tapa laskea miten nämä koordinaatistot pelaavat toistensa suhteen: Lorentz- muunnokset. Suuren nopeuden aikaan aiheuttama efekti on nimeltään aikadilaatio, ja senkin voi laskea.

        (Linkki: https://fi.wikipedia.org/wiki/Lorentz-muunnos )
        (Linkki: https://fi.wikipedia.org/wiki/Erityinen_suhteellisuusteoria#Aikadilataatio )

        Anyway, meidän kellolla mitattuna fotoni tulee Auringosta 8 minuutissa ja toimittaa meille oman energiavarauksensa. Vaikka aika olikin fotonille stopissa ja aikadilaatiot päällä, kvantti tuli perille, eikä muuttanut lopputulosta. Se sähkömagneettinen varaus on se joka merkkasi, ei se miten se koki ajan kuluneen, koska ajan kuluminen ei merkitse siinä mitään. Me tulkitaan sitä kvanttia silmän kautta aivossa. Me ajatellaan, että asia tapahtui nyt, mutta tiedämme, että se tapahtui itseasiassa 8 min sitten, koska se fotoni syntyi 8 min sitten ja sillä otti sen 8 min aikaa tuoda se juttu meille.

        Mutta jos onkin kaksi havaitsijaa, joista toinen liikkuu ja toinen on paikallaan, ja molemmat mittaavat omassa koordinaatistossaan omalla kellollaan ajankulkua, melko "hitaissakin" nopeuksissa alkaa eroja syntymään "äkkiä". Tällöin toisen kellon mukaan aika kuluu nopeammin kuin toisen: se joka liikkuu suhteessa nopeammin, hänelle aika kuluu hitaammin suhteessa hitaamminliikkuvaan, kello jätättää. Ja hitaammin liikkuvan näkökulmasta (jos toinen on vaikka kiihtyvässä liikeessä) kun vauhti kasvaa, sen toisen liikkeet myös alkavat näyttää hitaammilta ja hitaammilta.

        Esim. GPS paikantaminen, ellei GPS satelliittien osalta oltaisi tehty mm. suuren nopeuden (maan suhteen) aiheuttaman aikadilaation korjausta (erityinen tai suppea suhteellisuusteoria) tai painovoiman vaikutuksen korjausta (kiertoradalla on pienempi painovoiman vaikutus aikaan, kuin maanpinnalla, yleinen suhtellisuusteoria), ne olisi aivan käyttökelvottomia aparaatteja. Virheen määrä olisi jopa 10km pieleen per vuorokausi vaikka virhettä syntyy miljoonasosa sekunteja.

        (Linkki: https://fi.wikipedia.org/wiki/Yleinen_suhteellisuusteoria )

        Toinen esim. Lorentz muunnoksien ehkä "tunnetuin" käyttötapausesimerkki on sellainen, että kaveri seisoo oikeassa kädessä lamppu, joka osoittaa oikealle, ja vasemmassa toinen lamppu, joka osoittaa vasemmalle. Lamput laitetaan päälle, molemmista lampuista lähtevät fotonit liikkuu valonnopeudella c kaverista poispäin. Äkkiäajateltuna ne fotonit etenee toisistaan poispäin 2c nopeudella, mutta näin ei ole. Se mitä se on, saadaan muunnosten kautta ja on c.

        Jep. Aika on kimuranttinen. Elämäkin joskus.


      • EttäSellaiKiihytyskisa
        planeetta_ei.kirj kirjoitti:

        Virallinen selitys alla :-)

        Kyllä se aika pysähtyy tai pysähtyi, tavallaan, mutta vaan sille fotonille.

        Fotoni on massaton energiakvantti, joka kantaa mukanaan pienen sähkömagneettisen varauksen, jonka suuruus määräytyy taajuuden mukaan.

        Sen tehtävä on toimia välittäjänä ja välitysnopeus on c (se noin 300000km/s).

        Mikään ei liiku sitä c:tä nopeammin eikä mikään millä on massaa voi liikkua nopeudella c vaan aina hitaammin (koska mistään ei löydy niin paljoa energiaa, että massa saataisiin kiihtymään nopeuteen c). Valonnopeus c on siis absoluuttinen raja, jonka yli ei voi mennä.

        Näin sanoo teoria. Tai teoria ennustaa, että on olemassa raja ja rajaksi on havaittu olevan c.

        Suhteellisuusteorian mukaan aika on suhteellinen "havaitsijalle". Jokaisella "havaitsijalla" on oma koordinaatistonsa (sanotaan inertiaalikoordinaatistoksi). On olemassa tapa laskea miten nämä koordinaatistot pelaavat toistensa suhteen: Lorentz- muunnokset. Suuren nopeuden aikaan aiheuttama efekti on nimeltään aikadilaatio, ja senkin voi laskea.

        (Linkki: https://fi.wikipedia.org/wiki/Lorentz-muunnos )
        (Linkki: https://fi.wikipedia.org/wiki/Erityinen_suhteellisuusteoria#Aikadilataatio )

        Anyway, meidän kellolla mitattuna fotoni tulee Auringosta 8 minuutissa ja toimittaa meille oman energiavarauksensa. Vaikka aika olikin fotonille stopissa ja aikadilaatiot päällä, kvantti tuli perille, eikä muuttanut lopputulosta. Se sähkömagneettinen varaus on se joka merkkasi, ei se miten se koki ajan kuluneen, koska ajan kuluminen ei merkitse siinä mitään. Me tulkitaan sitä kvanttia silmän kautta aivossa. Me ajatellaan, että asia tapahtui nyt, mutta tiedämme, että se tapahtui itseasiassa 8 min sitten, koska se fotoni syntyi 8 min sitten ja sillä otti sen 8 min aikaa tuoda se juttu meille.

        Mutta jos onkin kaksi havaitsijaa, joista toinen liikkuu ja toinen on paikallaan, ja molemmat mittaavat omassa koordinaatistossaan omalla kellollaan ajankulkua, melko "hitaissakin" nopeuksissa alkaa eroja syntymään "äkkiä". Tällöin toisen kellon mukaan aika kuluu nopeammin kuin toisen: se joka liikkuu suhteessa nopeammin, hänelle aika kuluu hitaammin suhteessa hitaamminliikkuvaan, kello jätättää. Ja hitaammin liikkuvan näkökulmasta (jos toinen on vaikka kiihtyvässä liikeessä) kun vauhti kasvaa, sen toisen liikkeet myös alkavat näyttää hitaammilta ja hitaammilta.

        Esim. GPS paikantaminen, ellei GPS satelliittien osalta oltaisi tehty mm. suuren nopeuden (maan suhteen) aiheuttaman aikadilaation korjausta (erityinen tai suppea suhteellisuusteoria) tai painovoiman vaikutuksen korjausta (kiertoradalla on pienempi painovoiman vaikutus aikaan, kuin maanpinnalla, yleinen suhtellisuusteoria), ne olisi aivan käyttökelvottomia aparaatteja. Virheen määrä olisi jopa 10km pieleen per vuorokausi vaikka virhettä syntyy miljoonasosa sekunteja.

        (Linkki: https://fi.wikipedia.org/wiki/Yleinen_suhteellisuusteoria )

        Toinen esim. Lorentz muunnoksien ehkä "tunnetuin" käyttötapausesimerkki on sellainen, että kaveri seisoo oikeassa kädessä lamppu, joka osoittaa oikealle, ja vasemmassa toinen lamppu, joka osoittaa vasemmalle. Lamput laitetaan päälle, molemmista lampuista lähtevät fotonit liikkuu valonnopeudella c kaverista poispäin. Äkkiäajateltuna ne fotonit etenee toisistaan poispäin 2c nopeudella, mutta näin ei ole. Se mitä se on, saadaan muunnosten kautta ja on c.

        Jep. Aika on kimuranttinen. Elämäkin joskus.

        Toisaalta: Toi sun "hitaasti" liikkuvakin kohde kyl suhahtaa siitä sun paikallaan olevasta havaitsijasta ohi semmosta haipakkaa, et ei siinä jää paljon aikaa kuulumisia, saati sit edes kellonaikoja, vaihtaa? :P :D
        (Vaikka tokikin suhteellisuusteorian mukaan sille suhahtajalle tosiaankin jää ENEMMÄN aikaa vaihtaa niitä kuulumisia ;) )

        Ja tossakin sä hieman kämmäsit, et ei sen paikallaanolijan kello siinä paikallaan ollessa kyllä kulu yhtään sen hitaammin tai nopemmin kuin ennenkään!? Eli: Mä muuttasin sun tän lauseen: "Tällöin toisen kellon mukaan aika kuluu nopeammin kuin toisen" muotoon: "Tällöin toisen kellon mukaan aika kuluu nopeammin". Koska sen toisen kellon aika ei tosiaankaan muutu mihinkään ainakaan sille paikalleenolijalle. Toisaalta sen suhahtajan näkökulmasta taas sen oma kello rullaa ihan samanlailla kun ennenkin, mut sen paikallaanolijan (joka on nyt sit jo useemman miljoonan km päässä? :P Et saa olla melko hyvät kiikarit jos meinaa sen nähdä?) kulkee kaikenaikaa hitaammin, mitä enemmän vauhti kiihtyy. Tos oot toki oikeessa, et sen paikallaanolijan mielestä sen suhahtajan aika hidastuu, mut siinä vaiheessa se on suhahtanu jo niin kauas, et sillä ei sille o enää juurikaan merkitystä.

        Toisaalt jos ois joku keino kiihdyttää massallinen kappale ihan järjettömän nopeesti tosi lähelle valonnopeutta ihannhetkessä, ni se vois olla vinkee tapaus ku se suhahtaja jämähtäis ku seinään siinä sen silmien edessä, vaikka sen itsensä mielestä se olis menevinään kaameeta haipakkaa? :D
        Valon nopeus kiihdytyskisat avaruudessa ei olis ainakaan mikään yleisölaji, koska yksikään kiihdytysvekotin ei pääsis koskaan maaliin saakka katsojien mielestä! :D


      • Ohman
        planeetta_ei.kirj kirjoitti:

        Virallinen selitys alla :-)

        Kyllä se aika pysähtyy tai pysähtyi, tavallaan, mutta vaan sille fotonille.

        Fotoni on massaton energiakvantti, joka kantaa mukanaan pienen sähkömagneettisen varauksen, jonka suuruus määräytyy taajuuden mukaan.

        Sen tehtävä on toimia välittäjänä ja välitysnopeus on c (se noin 300000km/s).

        Mikään ei liiku sitä c:tä nopeammin eikä mikään millä on massaa voi liikkua nopeudella c vaan aina hitaammin (koska mistään ei löydy niin paljoa energiaa, että massa saataisiin kiihtymään nopeuteen c). Valonnopeus c on siis absoluuttinen raja, jonka yli ei voi mennä.

        Näin sanoo teoria. Tai teoria ennustaa, että on olemassa raja ja rajaksi on havaittu olevan c.

        Suhteellisuusteorian mukaan aika on suhteellinen "havaitsijalle". Jokaisella "havaitsijalla" on oma koordinaatistonsa (sanotaan inertiaalikoordinaatistoksi). On olemassa tapa laskea miten nämä koordinaatistot pelaavat toistensa suhteen: Lorentz- muunnokset. Suuren nopeuden aikaan aiheuttama efekti on nimeltään aikadilaatio, ja senkin voi laskea.

        (Linkki: https://fi.wikipedia.org/wiki/Lorentz-muunnos )
        (Linkki: https://fi.wikipedia.org/wiki/Erityinen_suhteellisuusteoria#Aikadilataatio )

        Anyway, meidän kellolla mitattuna fotoni tulee Auringosta 8 minuutissa ja toimittaa meille oman energiavarauksensa. Vaikka aika olikin fotonille stopissa ja aikadilaatiot päällä, kvantti tuli perille, eikä muuttanut lopputulosta. Se sähkömagneettinen varaus on se joka merkkasi, ei se miten se koki ajan kuluneen, koska ajan kuluminen ei merkitse siinä mitään. Me tulkitaan sitä kvanttia silmän kautta aivossa. Me ajatellaan, että asia tapahtui nyt, mutta tiedämme, että se tapahtui itseasiassa 8 min sitten, koska se fotoni syntyi 8 min sitten ja sillä otti sen 8 min aikaa tuoda se juttu meille.

        Mutta jos onkin kaksi havaitsijaa, joista toinen liikkuu ja toinen on paikallaan, ja molemmat mittaavat omassa koordinaatistossaan omalla kellollaan ajankulkua, melko "hitaissakin" nopeuksissa alkaa eroja syntymään "äkkiä". Tällöin toisen kellon mukaan aika kuluu nopeammin kuin toisen: se joka liikkuu suhteessa nopeammin, hänelle aika kuluu hitaammin suhteessa hitaamminliikkuvaan, kello jätättää. Ja hitaammin liikkuvan näkökulmasta (jos toinen on vaikka kiihtyvässä liikeessä) kun vauhti kasvaa, sen toisen liikkeet myös alkavat näyttää hitaammilta ja hitaammilta.

        Esim. GPS paikantaminen, ellei GPS satelliittien osalta oltaisi tehty mm. suuren nopeuden (maan suhteen) aiheuttaman aikadilaation korjausta (erityinen tai suppea suhteellisuusteoria) tai painovoiman vaikutuksen korjausta (kiertoradalla on pienempi painovoiman vaikutus aikaan, kuin maanpinnalla, yleinen suhtellisuusteoria), ne olisi aivan käyttökelvottomia aparaatteja. Virheen määrä olisi jopa 10km pieleen per vuorokausi vaikka virhettä syntyy miljoonasosa sekunteja.

        (Linkki: https://fi.wikipedia.org/wiki/Yleinen_suhteellisuusteoria )

        Toinen esim. Lorentz muunnoksien ehkä "tunnetuin" käyttötapausesimerkki on sellainen, että kaveri seisoo oikeassa kädessä lamppu, joka osoittaa oikealle, ja vasemmassa toinen lamppu, joka osoittaa vasemmalle. Lamput laitetaan päälle, molemmista lampuista lähtevät fotonit liikkuu valonnopeudella c kaverista poispäin. Äkkiäajateltuna ne fotonit etenee toisistaan poispäin 2c nopeudella, mutta näin ei ole. Se mitä se on, saadaan muunnosten kautta ja on c.

        Jep. Aika on kimuranttinen. Elämäkin joskus.

        Siinä koordinaatistossa, missä se lamppuja pitelelevä on paikallaan, fotonit kyllä etenevät toisistaan nopeudella 2 c.
        Tässä ei tapahdu mitään koordinaatistojen välistä, jolloin tuo ES:n nopeuksien yhteenlaskukaava tulee kysymykseen, vaan on vain yksi koordinaatisto.Tässä koordinaatistossa mikään ei liiku nopeudella joka olisi suurempi kuin valonnopeus.Vain fotonien välimatka kasvaa nopeudella 2 c.
        Kun meillä on koordinaatistossa A koordinaatisto B ja joka liikkuu A:n suhteen nopeudella u ja tässä B:ssä jokin kappale liikkuu nopeudella v niin A-koordinaatistossa tuo kappale liikkuu ES:n yhteenlaskukaavan mukaisesti nopeudella (u v) / (1 (uv)/c^2). Tästä nähdään, että vaikka olisi u=v=c niin tuo nopeus on c. Mutta tuo lamppuesimerkki ei siis ole tällainen tapaus vaan ne fotonit liikkuvat yhdessä ja samassa koordinaatistossa.
        Ohman


      • Kiitokset korjauksista koskien epätarkkuuksia ja asiavirheitä.


    • Ohosano

      On siis turha vilkutella öisellä tähtitaivalla heille.

    • planeetta_ei.kirj

      Asiasta hieman toiseen.

      Lueskelin tuossa HD10180 tähdestä. Sijainti noin 40pc päässä.

      Suurinpiirtein samaa sarjaa, kuin Aurinko, koolleen, koostumukselleen ja kirkkaudelleen. 7-9 planeetan systeemi. Tähti ei näy paljaalla silmällä Maasta katsottuna, mutta pienellä kaukoputkella kyllä. Planeettoja ei olla suoraan havaittu (visuaalisesti), vaikka osa on suuria, eikä ne mene täältä katsottuna tähden yli. Olemassaoloa, ratoja ja kokoja on arvoitu mm. spektrograafeilla tehdyistä mittauksista (Keppler, Harps, LaSilla). Havainto planeetoista siis liittyy tähden "huojuntaan".

      (Lisää linkki: https://en.wikipedia.org/wiki/HD_10180 )

      Ehkä nämä meidän vermeet ovat alkeellisia, mutta 3000pc päässä olevien kohteiden havainnointi on vieläkin hankalampaa. Siltä etäisyydeltä varmat havainnot planeetoista alkavat olla enempivähempi vain tähden editse meneviä tähden ylityksiä. Toistaiseksi kaukaisimmat ihmisten havaitsemat tähteä kiertävät planeetat ovat noin 8500pc etäisyydellä.

      (Lisää linkki: https://fi.wikipedia.org/wiki/Luettelo_ennätyksellisistä_eksoplaneetoista )

      Joten, jos joku siltä 3000pc etäisyydeltä tiirailee, se havaitsee meidät todennäköisesi aina kun menemme Auringon editse, eli meidän termein "kerran vuodessa". Eli havainnoitsija on suurinpiirtein samassa tasossa, kuin on Maan kiertorata Auringon ympäri.

      Se mikä tekee asian vähän mielenkiintoiseksi on se, että kun on arvailtu todennäköistä(?) etäisyyttä toiseen sivilisaatioon, yksi arvaus on, että se olisi jossain 3000pc etäisyydellä.

      (Lisää linkki: https://www.ursa.fi/yhd/komeetta/esitelma/hackma.htm )

      Joten, jos olisi yhtään vainoharhaa, keskittyisin tuolla etäisyydellä plusmiinus ja maan kiertoradan tasolla plusmiinus muutama aste olevien tähtien (joilla on planeettpja) kyyläämiseen :-)

      • NytEiYmmärtää

        Nojoo aika lailla samankokoi, lämpöi ja kirkkauksinenki toi HD_10180 ku toi omaki pallukkamme, mut: Toi ei nyt iha menny jakeluu et miks siel on toi gravitaatio 4,39 cgs (vaik sen massaki ihan hikisesti aurinkoo isompi) ku se taas auringolla näyttäs olevan 27,5 cgs? Vai enks mä nyt vaa jotenki ymmärrä tota yksikköä? Mut jos noi yksiköt on suoraa verrannollisia, ni samallai pallukka, ja massaki suht sama, mut toisessa 5 kertaa kovempi painovoima ei kyl ihan me yks-yhtee mun fysiikan oppieni kans? :/


      • planeeta_ei.kirj
        NytEiYmmärtää kirjoitti:

        Nojoo aika lailla samankokoi, lämpöi ja kirkkauksinenki toi HD_10180 ku toi omaki pallukkamme, mut: Toi ei nyt iha menny jakeluu et miks siel on toi gravitaatio 4,39 cgs (vaik sen massaki ihan hikisesti aurinkoo isompi) ku se taas auringolla näyttäs olevan 27,5 cgs? Vai enks mä nyt vaa jotenki ymmärrä tota yksikköä? Mut jos noi yksiköt on suoraa verrannollisia, ni samallai pallukka, ja massaki suht sama, mut toisessa 5 kertaa kovempi painovoima ei kyl ihan me yks-yhtee mun fysiikan oppieni kans? :/

        Se on mittayksikkökikkailua. Normaalisti SI järjestelmän mukaan käytetään mittayksiköissä metriä, kilogrammaa, sekuntia jne. Tuossa cgs:ssä on käytetty senttiä, grammaa ja sekuntia, siitä tuo lyhenne (centimeter, gram, second). Lisäksi tässä yhteydessä on arvo väännetty log-10 logaritmiseksi, siksi siellä on se maininta (g log) mukana.

        Kyllä siitä putoamiskiihtyvyyteen pääsee peruuttamalla takaisinpäin, mutta vähän joutuu laskemaan.

        Ja kun noita cgs (log g) mittayksikössä olevia miettii, niin ensin pitää ymmärtää mitä tarkoittaa 10-kantainen logaritmi.

        1=1, 10=2, 1000 = 3, 10000=4 jne.

        Esim. Maan cgs (log g) arvo on melko lähellä 3.

        (Koska cgs:inä g on 100 kertaa putoamiskiihtyvyys 9,81 normaalimmassa yksikössä m/s², eli siitä tulee ensin 9,81m/s² -> 981cm/s² ja kun tuo on "melkein" 1000, 10-kantaisena logaritmina se on "melkein" 3, jotain 2,9 jne).

        Se miksi tuollaista käytetään johtuu varmaan siitä, että logaritmin takia todella suuret luvut ovat ehkä kivemman näköisiä. Esim. tähtitieteen suuruusluokan luku 1,0 * 10²¹ on 10-kanta logaritmina 21. Mutta ei toisiaankaan siinä tapauksessa tarkoita, että se luku on 21 kertaa suurempi, kuin 1 :-)


      • NoNytAukes
        planeeta_ei.kirj kirjoitti:

        Se on mittayksikkökikkailua. Normaalisti SI järjestelmän mukaan käytetään mittayksiköissä metriä, kilogrammaa, sekuntia jne. Tuossa cgs:ssä on käytetty senttiä, grammaa ja sekuntia, siitä tuo lyhenne (centimeter, gram, second). Lisäksi tässä yhteydessä on arvo väännetty log-10 logaritmiseksi, siksi siellä on se maininta (g log) mukana.

        Kyllä siitä putoamiskiihtyvyyteen pääsee peruuttamalla takaisinpäin, mutta vähän joutuu laskemaan.

        Ja kun noita cgs (log g) mittayksikössä olevia miettii, niin ensin pitää ymmärtää mitä tarkoittaa 10-kantainen logaritmi.

        1=1, 10=2, 1000 = 3, 10000=4 jne.

        Esim. Maan cgs (log g) arvo on melko lähellä 3.

        (Koska cgs:inä g on 100 kertaa putoamiskiihtyvyys 9,81 normaalimmassa yksikössä m/s², eli siitä tulee ensin 9,81m/s² -> 981cm/s² ja kun tuo on "melkein" 1000, 10-kantaisena logaritmina se on "melkein" 3, jotain 2,9 jne).

        Se miksi tuollaista käytetään johtuu varmaan siitä, että logaritmin takia todella suuret luvut ovat ehkä kivemman näköisiä. Esim. tähtitieteen suuruusluokan luku 1,0 * 10²¹ on 10-kanta logaritmina 21. Mutta ei toisiaankaan siinä tapauksessa tarkoita, että se luku on 21 kertaa suurempi, kuin 1 :-)

        No täähän selvens, ja selitti, ton mysteerin vallan mainiosti!

        Kiitti vaan vaivannäöstäsi.


      • AukeeVieläkin
        planeeta_ei.kirj kirjoitti:

        Se on mittayksikkökikkailua. Normaalisti SI järjestelmän mukaan käytetään mittayksiköissä metriä, kilogrammaa, sekuntia jne. Tuossa cgs:ssä on käytetty senttiä, grammaa ja sekuntia, siitä tuo lyhenne (centimeter, gram, second). Lisäksi tässä yhteydessä on arvo väännetty log-10 logaritmiseksi, siksi siellä on se maininta (g log) mukana.

        Kyllä siitä putoamiskiihtyvyyteen pääsee peruuttamalla takaisinpäin, mutta vähän joutuu laskemaan.

        Ja kun noita cgs (log g) mittayksikössä olevia miettii, niin ensin pitää ymmärtää mitä tarkoittaa 10-kantainen logaritmi.

        1=1, 10=2, 1000 = 3, 10000=4 jne.

        Esim. Maan cgs (log g) arvo on melko lähellä 3.

        (Koska cgs:inä g on 100 kertaa putoamiskiihtyvyys 9,81 normaalimmassa yksikössä m/s², eli siitä tulee ensin 9,81m/s² -> 981cm/s² ja kun tuo on "melkein" 1000, 10-kantaisena logaritmina se on "melkein" 3, jotain 2,9 jne).

        Se miksi tuollaista käytetään johtuu varmaan siitä, että logaritmin takia todella suuret luvut ovat ehkä kivemman näköisiä. Esim. tähtitieteen suuruusluokan luku 1,0 * 10²¹ on 10-kanta logaritmina 21. Mutta ei toisiaankaan siinä tapauksessa tarkoita, että se luku on 21 kertaa suurempi, kuin 1 :-)

        Ja senverran vielä, et varmaan tohon logaritmin käyttöön on päädytty, koska tuol taivaalla mollottaa kohtalaisen paljon isompia pallukoita ku toi omamme. Ja nyt tota samaa yksikköä voi käyttää vallan mainiosti myös kokonaisten galaksien tapauksessakin?

        Mut ei o ite tätä ennen tullu koskaan kiinnitettyä oikeen mitään huomiota noihin gravitaatioihin, ni toi oli mulle suht uus yksikkö kokonaan!? :P


      • AukeeVieläkin kirjoitti:

        Ja senverran vielä, et varmaan tohon logaritmin käyttöön on päädytty, koska tuol taivaalla mollottaa kohtalaisen paljon isompia pallukoita ku toi omamme. Ja nyt tota samaa yksikköä voi käyttää vallan mainiosti myös kokonaisten galaksien tapauksessakin?

        Mut ei o ite tätä ennen tullu koskaan kiinnitettyä oikeen mitään huomiota noihin gravitaatioihin, ni toi oli mulle suht uus yksikkö kokonaan!? :P

        Näinpä. Kylläkai galakseillekin voi jotain sentyyppistä miettiä. Mut joo muuta yksinkertaista selitystä logaritmin käytölle en ole vielä keksinyt, tosin se johtuu varmaan siitä, että en minäkään tuon asian kanssa päivittäin ole tekemisissä.

        Jos jaksaisi miksi- kysymystä googlata, niin ehkä jotain muunlaista enemmän käytännöllistä(?) selitystä saattaisi tuolle mittayksikölle löytyäkin.


      • EiKaiNäitäKaikkiaTarvi
        planeetta kirjoitti:

        Näinpä. Kylläkai galakseillekin voi jotain sentyyppistä miettiä. Mut joo muuta yksinkertaista selitystä logaritmin käytölle en ole vielä keksinyt, tosin se johtuu varmaan siitä, että en minäkään tuon asian kanssa päivittäin ole tekemisissä.

        Jos jaksaisi miksi- kysymystä googlata, niin ehkä jotain muunlaista enemmän käytännöllistä(?) selitystä saattaisi tuolle mittayksikölle löytyäkin.

        Nii no en tiedä sit et onko noitten tiedemiesten suosimien yksiköiden liiallisesta tuntemuksesta ihan tälläselle harrastelijalle edes sanottavampaa hyötyä? :/ Mä en o oikee ikinä bonjannu edes sitä et mitä järkeä ton parsekin käytössäkään on? :D


      • nytlähdenetelään
        NytEiYmmärtää kirjoitti:

        Nojoo aika lailla samankokoi, lämpöi ja kirkkauksinenki toi HD_10180 ku toi omaki pallukkamme, mut: Toi ei nyt iha menny jakeluu et miks siel on toi gravitaatio 4,39 cgs (vaik sen massaki ihan hikisesti aurinkoo isompi) ku se taas auringolla näyttäs olevan 27,5 cgs? Vai enks mä nyt vaa jotenki ymmärrä tota yksikköä? Mut jos noi yksiköt on suoraa verrannollisia, ni samallai pallukka, ja massaki suht sama, mut toisessa 5 kertaa kovempi painovoima ei kyl ihan me yks-yhtee mun fysiikan oppieni kans? :/

        Miksi täällä jotkut ns. älyköksi itseään kuvittelevat, luulevat tietävänsä jotain ajasta, avaruudesta, ikuisuudesta, olemattomuudesta jollain matemaattisilla kaavoilla ( ja toiset hörhöt, varmaan suurin osa maailman väestöstä, lukevat omia eri raamattujaan uskoen niihin).
        Miksei voisi tunnustaa,
        että on liian paljon asioita , mitä ei voi ihmisen järjellä koskaan selittää, ymmärtää?

        Yritetään keksiä uusia merkityksiä yksinkertaisille asioille, joka kaikki hengissä säilyneet eläinlajit ovat tienneet jo kauan, kauan sitten .


      • MinenLäheMihinkää
        nytlähdenetelään kirjoitti:

        Miksi täällä jotkut ns. älyköksi itseään kuvittelevat, luulevat tietävänsä jotain ajasta, avaruudesta, ikuisuudesta, olemattomuudesta jollain matemaattisilla kaavoilla ( ja toiset hörhöt, varmaan suurin osa maailman väestöstä, lukevat omia eri raamattujaan uskoen niihin).
        Miksei voisi tunnustaa,
        että on liian paljon asioita , mitä ei voi ihmisen järjellä koskaan selittää, ymmärtää?

        Yritetään keksiä uusia merkityksiä yksinkertaisille asioille, joka kaikki hengissä säilyneet eläinlajit ovat tienneet jo kauan, kauan sitten .

        En tiiä? Kannattais varmaan kysyy joltain älyköltä tota? Koska mä oon kaukana sellasesta! :D Ja matikasta varsinka en juuri mitää :/

        Ja ei nää ny mitää mystisiä juttuja o joista täällä jutellaan, vaan kyl nää on ihan ja juurikin ihmisten selvittämiä asioita. Ei tääl kuitenka niin viisasta porukkaa (varmaan?) ole joka pystys ihan uusiakaan tieteellisiä teorioita tosta vaan kikkailemaan keskusteltavaks.

        Ja miksikö, kysyt? No muo nyt sattuu ainaki avaruus ja fysiikka jostain kumman syystä kiinnostamaan, enkä o ees paljoo pohtinu et misko? :/
        Ehkä se on se, et noissa on niin paljon vielä tyntemattomia asioita et on ihan mielenkiintosta seurata et mitähän ne seuraavaks keksii? :P Varsinki ku tää maapallo alkaa olee suht tarkkaan koluttu, et tuski täält iha hevillä mitää uusiakaan mysteereitä ilmenee? Ja tokihan se helpompaa olis, ku kattelis vaa töllöttimestä Idolseja ja Emmerdalia, mutta kun mä mieluummin pitäsin hoksottimeni edes jonkulaisessa liikkeessä, ku turruttaisin tän lopunki vähäsen järkeni töllöttämällä liikkuvia kuvia. Joitten ihmissuhde-hömppä ei jaksa kovinka kauaa kiinostaa. Varsinki ku noitten juonenkäänteet summuut sävelet on tullu jo nii monia satoja kertoja ennenki nähtyy, sillon ku jakso viel ees jonku verran tollasiakin seurata :/

        Ja mä kyl veikkaan, et fysiikasta ja kosmologiasta edes jotain tietäviä eläinlajeja on maaimalla just tismalleen yks kappale!? :D Siis ei todellakaan kaikki!


    • Naftalefa

      Riippuu siitä millaiset vehkeet maata mahdollisesti tiirailevilla on. Kuitenkin näkisivät aika lailla erilaisen maan mitä nyt on.

    • hähähähähähähähäääää

      Hah hah, kai olette netistä katsoneet miltä tähdet oikeasti näyttävät..., nehän ovat vain sähköisiä palleroita.

      Niille jotka eivät ole niitä nähneet silleen oikeesti niin tässä ne ovat, olkaa hyvä:
      https://www.youtube.com/watch?v=hdNFo5eWf9g

      Hauskaa tässä on se että tämän voi jokainen itse varmentaa vaikka Nikon P900:lla hintaan 569 €.

      • Itseasiassa, yllättävän hieno suoritus! Ja viihdyttävä myös :-)
        "Hieman" kieli poskessa tehty, ainakin toivon niin.


      • Kapteenihaddock

        Tästä paskasta meillä on kiittäminen nikonia kun teki tuollaisen suhthalvan kameran missä on suuri optine suurennos tosi pienellä kennolla ja objektiivilla. Sitä ei koskaa tarkoitettu avaruuden kuvaamiseen. Jos nämä youtubesankarit olisivat edes joskus katsoneet tähtiä vaikka kunnollisen 300 millisen peilikaukoputken läpi, niin näitä kikkeliskokkelissähkötähtivideoita ei edes olisi.


      • Kapteenihaddock

        Ja paljon halvemmalla saa kunnollisen kaukoputken millä voi omin silmin katsoa taivasta.


      • Kapteenihaddock

        Luulen silti et sä et ole ikinä edes itse katsonut vastaavan kaukoputken läpi. Perustat mielipiteesi vain tuntemattomien ihmisten youtubevideoihin? Onko näin? Luulen että on näin. Suosittelen suuresti hankkimaan tähtien katseluun tarkoitetun kaukoputken. Eivät ole hirvittävän kalliita. Ne voi myös avata ja havaita että siellä ei ole mitään nasan projektoreita tuottamassa valheellisia kuvia. Ne ovat vain linssejä ja peilejä.
        Luulenpa silti että oot täällä vain hieman trollaamassa, mutta ei se mitään. Kaikkien pitäisi joskus katsoa avaruutta kunnollisella putkella.


      • PerinHuvittavaJoo

        Lähinnä toi must osotti ainoastaan sen, et kui surkeita noi nykyset digipokkarit on kuvaamaa heikosti valastuja kohteita? :/

        Ja toi lähinnä herätti hilpeyttä kutsua Betelgeusea "supermassiivisen kaukana" olevaks, ku sehän on täs ihan meidän "hoodeilla" ja muistaakseni kaikista lähin punanen jättiläinen!? :D

        Ja tohon en ees alennu vastaamaan et osottaako noi tähdet meitä kohti! :/


    • hähähähähäänKaveri

      Noh, pojat. Laittakaapa sitten tulemaan kuvaa sillä ihanalla 300 millisellä peilikaukoputken läpi otettuna. Ei pitäisi olla vaikeaa.
      Odottelemisiin.

    • tähdetTähdetkuinkaKaipaa

      Tuo edellä linkitetty otos oli otettu Nikon COOLPIX P610:llä.
      Ovatko filminpätkät vääristyneitä vai väärennettyjä?
      Ilmakehän vaikutuksen voinemme jättää pois koska korvaavaksi katselun lähteeksi suositellaan vaatimatonta kaukoputkea.
      En tiedä nyt mihin uskoa.

      • starmann

        Kun kerran katsot laadukkaalla kaukoputkella tähtitaivasta niin tiedät mihin uskoa.

        Suosittelen käyntiä lähimmällä ursan tähtitornilla jonakin selkeänä yönä kun tornilla on yleisönäytös.


      • kiljaus

        Voiko noihin kaukoputkiin liittää kameran niin näkisimme sitten miksi ne näyttävät erilaisilta kuin suoraan huippukameroilla kuvattaessa?


      • kiljaus kirjoitti:

        Voiko noihin kaukoputkiin liittää kameran niin näkisimme sitten miksi ne näyttävät erilaisilta kuin suoraan huippukameroilla kuvattaessa?

        Voi liittää, mutta tähtivalokuvaus edellyttää kuitenkin tähtivalokuvauksen alkeiden omaksumista ja soveltamista.

        Joutuu siis opettelemaan.

        Tähtivalokuvauksen alkeet opittuaan luultavasti ja toivottavasti ymmärtää, että hyvälläkin digikameralla tähtiä videokuvatessaan isolla suurennoksella tulee kuvanneeksi lähinnä kameran kuvakennon kohinaa, enkooderin häiriöitä ja interferenssiä.


      • kiljaus kirjoitti:

        Voiko noihin kaukoputkiin liittää kameran niin näkisimme sitten miksi ne näyttävät erilaisilta kuin suoraan huippukameroilla kuvattaessa?

        Käy lähimmässä kirjastossa ja ota nippu Tähdet ja Avaruus-lehtiä. Niitä selaamalla löydät harrastelijoiden ottamia tähtikuvia sekä kaukoputkella että ilman.


      • TähtiTiirailemaanLomps

        Juu ja täst löyty ainaki yks saitti mis on kivasti infoa jolla pääsee hyvin alkuun jos haluu alkaa kilpailemaan Hubblen kanssa cooleimmista avaruuskuvista:

        astrobackyard.(kopsaa)com/astrophotography-equipment/


      • planeetta_ei.kirj
        TähtiTiirailemaanLomps kirjoitti:

        Juu ja täst löyty ainaki yks saitti mis on kivasti infoa jolla pääsee hyvin alkuun jos haluu alkaa kilpailemaan Hubblen kanssa cooleimmista avaruuskuvista:

        astrobackyard.(kopsaa)com/astrophotography-equipment/

        Suomalaisia harrastajia on myös ja heistä ainakin osa käyttää Taivaanvahti- palvelua. Kuvia löytyy laidasta laitaan haloista, sateenkaarista ja kuukuvista kauempanakin oleviin kohteisiin, niissä yleensä on käytetty jotain vähän järeämpää kalustoa, vaikka kotikonsteinkin saa, jos jaksaa investoida ja käyttää aikaa vaikkapa sitten kymmenien kuvien pinojen vääntämiseen.

        (linkki palveluun: http://www.taivaanvahti.fi/ )


    • näkevät_jääkauden

      Näkevät jääkauden loppua maapallolla. Suomen pinta-alasta näkyisi vain puolet maata. 10000v sitten napapiirin pohjoispuolella oli pelkkää jäätä.

    • kiljaus
      • Ursan taivaanvahti-palvelusta löydät runsaasti suomalaisten harrastajien napsimia kuvia tähdistä. Eivät ne ole mitään maalauksia.

        https://urly.fi/PJB


    • Ajatusleikkiä

      On kyllä mielenkiintoinen ajatus. Jos pääsisin riittävänj kauaksi maapallolta silmänräpäyksessä voisin sieltä käsin tarkkaila vaikka oman syntymäni jos on riittävän tehokas kaukoputki ja koko elämääni.

    • Aloittaja esittää hyvän kysymyksen mutta siihen on hankala vastata. Aloituksen kysymys kun on suhteellisuusteoria huomioon ottaen huonosti asetettu eli siihen ei ole yksikäsitteistä oikeaa vastausta. Samanaikaisuus on suurista etäisyyksistä puhuttaessa ongelmallista.

      Syynä ongelmaan on se, että ajankohtaa "nyt" ei ole määritelty yksikäsitteisesti muualla kuin havaitsijan omassa sijainnissa. Kahdessa eri pisteessä tapahtuvat asiat eivät ole kaikkien havainnoitsijoiden mielestä samanaikaisia jos pisteiden välillä on yhtään etäisyyttä. Mitä suurempi etäisyysero sitä suurempi on mahdollinen vaihtelu ajan määrittämisessä, vaikka ei edes liikuttaisi relativistisilla nopeuksilla.

      Ongelman aiheuttaa jälkimmäinen termi v*x/c² ajan Lorenzin muunnoksessa

      t'=(t-v*x/c²)/sqrt(1-v²/c²),

      kun liike ja etäisyys on x-akselin suuntaista. Havainnoitsijan liiketila siis vaikuttaa aikakoordinaattiin t' sitä enemmän, mitä suurempi on havaitsijan ja havaittavan välinen etäisyys x ja mitä suurempi on havaitsijan ja havaittavan välinen suhteellinen nopeus v. Vaikka etäisyys x pysyisi samana niin vaihtelemalla nopeutta v saadaan erilaisia arvoja ajalle t'.

      Ilmeisesti kysyjä tarkoitti tilannetta, jossa 10000 valovuoden päässä olevan tarkkailijan "nyt" eli määritellään näin: Kello on kaukaiselle tarkkailijalle "nyt" kun meidän sijainnistamme 10000 vuotta sitten lähtenyt valo saavuttaisi heidän sijaintinsa. Lisäksi ilmeisesti oletetaan, että havaitsijan ja Aurinkokunnan nopeusero v on koko ajan ollut nolla.

      Tuolla määritelmällä sikäläinen tarkkailija tänne päin katsoessaan näkisi mitä Aurinkokunnassa tapahtui 10000 vuotta sitten, jos havaintoväline olisi kulmaresoluutioltaan riittävä erottamaan planeetat Auringon säteilystä ja matkan varrella oleva pöly ja kaasu ei saisi sirotettua Auringosta tulevaa valoa esteeksi havaintojen tekemiselle.

      Lorentzin muunnoksella on helppo katsoa mitä tapahtuu, jos tuon kaukaisen havaitsijan nopeus meihin nähden onkin koko ajan ollut esimerkiksi v=30km/s eli valon nopeuden kymmenestuhannesosa (0.0001c). Siitä tulee vuoden verran siirtymää aikakoordinaattiin ja siirtymän suunta riippuu nopeuden suunnasta.

      • TekniikkaPettää

        Ja tollasta teleskooppiahan ei ainaka meidän tekniikalla o pystytty vielä rakentamaanka, jolla sais kuvia otettua exoplaneetoista! Eks ne o yhen ainoon suttusen tuheron jostain saanu otettua, josta ei sit tiiä sanoo et onks se kärpäsen jätös vai planeetta? :D Mut ehkä niillä uffomiehillä "siellä jossain" on paremmat linssit?

        PS: Ja ihan hyvä huomio kyl toikin, mut: Ehkä ton /- 1 vuoden toleranssin kans pystyy elämään tolla 10 000 vuoden aikajanalla? :P


    • hähähähähhähääääää

      Tuossa kehotettiin katsomaan Ursan taivaanvahti- kuvia. Meninhän minä niitä toki niitä katsomaan. Kategoriota siellä ovat:
      - syvä avaruus,
      - aurinkokunta,
      - pimennys,
      - tulipallo,
      - revontulet,
      - harvinaiset pilvityypit,
      - myrskyilmiö,
      - haloilmiö,
      - muu ilmiö.
      Zoomattuja kuviä tähdistä ei ole, puhumattakaan zoomattua videokuvaa. Miksi ei?

      Laitan tähän siis vielä kerran nuo oikeat kuvat tähdistä:
      https://www.youtube.com/watch?v=hdNFo5eWf9g

      • sadddsaaaa

        Jos mielestäsi tähdet ovat tuollaisia niin sulla varmaan on oma teoria missä maapallo on?


      • KunnonKuvia

        No joks nää kelpais?

        https://s14-eu5.ixquick.com/cgi-bin/serveimage?url=https://img.purch.com/w/660/aHR0cDovL3d3dy5zcGFjZS5jb20vaW1hZ2VzL2kvMDAwLzA0MS83MjAvb3JpZ2luYWwvTk9BTy1QbGVhaWRlcy1zZXZlbi1zaXN0ZXJzLW5yYW8uanBn&sp=c0a2d075547933608e1cb1fc6e011590

        Ja otappas ny sit huomioon se, et se sun "oikea" markan tähtikaukoputkikameras näyttäis juurikin noi tossakin kuvassa näkyvät tähdet sun himoomina lautasen muotosina, kun sen erottelutarkkuus on niin erinomanen et se ei älyä et missä kohtaa sen tähden hohka alkaa ja mihin se loppuu, vaan näyttää koko paskan lähinnä jonain "nappina" taivaalla. Ja sellaisiahan tähdet toki ihan oikeasti on? :)


      • Villlllle

        "Zoomattuja kuviä tähdistä ei ole, puhumattakaan zoomattua videokuvaa. Miksi ei?"

        Koska millä vain olemassa olevalla kaukoputkella katsottuna tähdet ovat aina pistemäisiä kohteita. Ne sijaitsevat liian kaukana näkyäkseen minään muuna.


      • Villlllle kirjoitti:

        "Zoomattuja kuviä tähdistä ei ole, puhumattakaan zoomattua videokuvaa. Miksi ei?"

        Koska millä vain olemassa olevalla kaukoputkella katsottuna tähdet ovat aina pistemäisiä kohteita. Ne sijaitsevat liian kaukana näkyäkseen minään muuna.

        Lähin tähti on Aurinko. Siitä on ihan hyvää settiä olemassa, pilkkuja ja purkauksia jne. Muista tähdistä ei sellaisia kuvia vielä ole.

        Suurista tähdistä oli tässä ihan vastikään yhtä kuvattu ilmeisesti niin hyvin, kuin nykytekniikka antaa myöten. Tähti on Betelgeuse. Tämän kesän uutisia.

        (Linkki: https://astronomynow.com/2017/06/30/see-the-sharpest-ever-view-of-giant-betelgeuse/ )


      • KieliposkessaTehty
        planeetta kirjoitti:

        Lähin tähti on Aurinko. Siitä on ihan hyvää settiä olemassa, pilkkuja ja purkauksia jne. Muista tähdistä ei sellaisia kuvia vielä ole.

        Suurista tähdistä oli tässä ihan vastikään yhtä kuvattu ilmeisesti niin hyvin, kuin nykytekniikka antaa myöten. Tähti on Betelgeuse. Tämän kesän uutisia.

        (Linkki: https://astronomynow.com/2017/06/30/see-the-sharpest-ever-view-of-giant-betelgeuse/ )

        Etkö tiennyt? Aurinko ei ole tähti vaan litteää Maata kiertävä itsestään hohtava planeetta kuten kuu. Tähdet ovat pieniä "sähkö kipinöitä" vain muutaman valovuoden päässä :b


      • KieliposkessaTehty kirjoitti:

        Etkö tiennyt? Aurinko ei ole tähti vaan litteää Maata kiertävä itsestään hohtava planeetta kuten kuu. Tähdet ovat pieniä "sähkö kipinöitä" vain muutaman valovuoden päässä :b

        Siis tosi vaikea keksiä tohon lisättävää, silleen jotenkin tyhjentävä selitys. Pääsi yllättämään jotenkin takavasemmalta, ihan hiljaiseksi vetää. Kertakaikkiaan. Joka päivä sitä oppii jotain uutta.


      • KehitysHuononee
        planeetta kirjoitti:

        Siis tosi vaikea keksiä tohon lisättävää, silleen jotenkin tyhjentävä selitys. Pääsi yllättämään jotenkin takavasemmalta, ihan hiljaiseksi vetää. Kertakaikkiaan. Joka päivä sitä oppii jotain uutta.

        Ja mulla taas tuli mieleen et:
        "Eikös niissä muropakkauksissa enää nykyään ole niitä leluja joilla vois viihdyttää itseään"?


      • jos.ruokit.trollia.niin

        " Zoomattuja kuviä tähdistä ei ole, puhumattakaan zoomattua videokuvaa. Miksi ei?

        Laitan tähän siis vielä kerran nuo oikeat kuvat tähdistä:
        https://www.youtube.com/watch?v=hdNFo5eWf9g "

        Samainen tyyppi jatkaa valokuvausryhmässä.

        https://keskustelu.suomi24.fi/t/15029479/nikon-p610-ja-tahdet-

        Nyt kun sille on selitetty nämä asiat niin tähtitiederyhmässä kuin valokuvausryhmässäkin niin tiedätte aivan varmasti, että jatkossa jankutus asiasta on hänen osaltaan tarkoituksellista trollausta. Jos siis ette sitä tähän mennessä ole trollaukseksi ymmärtäneet.


    • Löytyy "zoomattu" kuva tähdestä. Interferometrian avulla onnistuu kulmahalkaisijaltaan kaikkein suurimpien tähtien tarkastelu muutenkin kuin pistemäisenä kohteena. Se edellyttää todella suurta ja toisaalta kokoonsa nähden lähellä olevaa tähteä.

      Tässä löytyy kuva Antareksesta. Tähti näkyy Suomessa toukokuun alussa matalalla etelätaivaalla Skorpionin tähdistössä. Se on noin 550 valovuoden päässä sijaitseva punainen superjättiläinen, jonka halkaisija on melkein 900 - kertainen Aurinkoon verrattuna.

      https://www.tiede.fi/artikkeli/uutiset/vain-aurinkoa-kuvattu-tarkemmin-kuin-antaresta

      Antares ei ole ainoa tähti josta on optisen interferometrian avulla saatu tarkempaa kuvaa. Kun interferometrian tekniikka kehittyy niin kuvien resoluutio paranee samassa suhteessa. Tässä siis yhdistetään satojen metrien päässä toisistaan olevien yksittäisten teleskooppien valoa valoaaltojen vaihe säilyttäen. Mitä kauempana toisistaan nuo teleskoopit ovat sitä parempi on laitteen resoluutio.

      Antares kooltaan vastaisi meitä lähimpänä olevan tähdistön (Alfa Kentauri) kohdalla olevaa halkaisijaltaan Aurinkoon nähden noin 7 kertaista tähteä. Valitettavasti Alfa Centauri A on vain Auringon kokoluokkaa oleva tähti, joten siitä ei vielä saada tarkempaa kuvaa aikaan. Alfa Centaurin tähtien halkaisijat on kyllä saatu mitattua interferometrisesti jo vuonna 2003, joten kovin kaukana ei olla tarkemmista kuvista.

      • Toi on mun mielestä parempi kuva kuin se mitä tonne ylemmäs linkkasin Betelgeusesta. Ja hieman uudempaakin kuvamateriaalia. Tämä on varmaan yksi alueista, jossa on kovaa kisaa käynnissä siitä kuka tai ketkä saa tarkempaa kuvaa aikaiseksi.


      • hummaHirnuu

        Betelgause onkin hieno kuvauskohde. Siitä on oikein videokuvaakin olemassa. Näitä tähtihörhömme välttävät esittämästä viimeiseen asti.
        Tässä sitä on kuvattu ajassa 5.12:
        https://www.youtube.com/watch?v=hdNFo5eWf9g


      • Kiitos kun jaksat.


      • KojahdaJo
        hummaHirnuu kirjoitti:

        Betelgause onkin hieno kuvauskohde. Siitä on oikein videokuvaakin olemassa. Näitä tähtihörhömme välttävät esittämästä viimeiseen asti.
        Tässä sitä on kuvattu ajassa 5.12:
        https://www.youtube.com/watch?v=hdNFo5eWf9g

        Ja Betelgeuse on muutenki antosa kohde kuikuilla ihan öögalla, koska se on varsin helppo löytää, ja siin olis hyvät saumat nähdä kohtalaisen cool supernova jos se sattuis vaikka omana elinaikanaan possahtamaan. Mikään mahottomuushan se ei ainakaan ole. Sen verran toi siihen suuntaan jo oireilee.


    • Aihepilalla

      Miten nämä liittyvät aiheeseen?

      • AikaFaktaa

        "...Miten nämä liittyvät aiheeseen?..."
        Tietyt nimimerkit alkaa yleensä kirjoittelemaan jostain ihan muusta kuin aloituksen aiheesta. Olevinaan viisasta kun puoli sivua tekstiä ja videoita kopioituna muista linkeistä.

        Jos oletetaan, että joku tätä maapalloa tiirailee juuri tällä hetkellä 10 000 valovuoden päässä, menee näkemiseen aikaan se sama 10 000 valovuotta, joten maapallo on jo 10 000 vuotta vanhempi, eli maapallolla eletään kristillisen ajanlaskun mukaan vuotta 12017.

        Mutta, jos joku näkee kaukoputkella meidät juuri tässä nykyajassa 10 000valovuoden päässä, on hänen täytynyt kiikaroida tänne 10 000 vuotta sitten.


    • DEFDEWEW

      No voi helvetti. Meinaatko että katsominen kulkee valonnopeutta? Miten näkemiseen menee aikaa? Jos katson maapallolta 1000 valovuoden päässä olevaan kohteeseen näen sen ihan heti.

      • AikaFaktaa

        No joo,, toki näet heti 1000 valovuoden päähän, mutta näkymä jonka näet on 1000 vuotta vanhaa, eli planeeltalta kulkeva "valonäkymä" on kulkenut sinun silmääsi 1000 vuotta.


      • info.liikkuu.valon.nopeu

        Informaatio etenee aina enintään valon nopeudella. Katsominen on informaation vastaanottamista, ei sen lähettämistä. Informaatio katsottavasta kohteesta voi tulla luoksesi korkeintaan valon nopeudella. Lähettäjä ei tiedä sitä että informaatio saapui perille ellet lähetä siitä hänelle tietoa takaisinpäin.

        Katseella itsellään ei ole nopeutta. Siinähän vain käännetään vastaanottoantenni eli silmät suuntaan josta halutaan sähkömagneettista säteilyä vastaanottaa. Tieto katseen suunnasta välittyy esimerkiksi katsojan kasvoista heijastuvan valon avulla eli sekin tieto etenee enintään valon nopeudella.

        Mikään ei estä muuttamasta katselusuuntaansa sadan valovuoden päässä olevasta tähdestä tuhannen valovuoden päässä olevaan tähteen vaikkapa puolessa sekunnissa. Minkään informaation nopeus ei kuitenkaan ole tuolloin 900 valovuotta puolessa sekunnissa. Sadan valovuoden päässä olevan tähden kohdalla tuo katseen kääntäminen huomataan teleskoopilla katsomalla aikaisintaan sadan vuoden kuluttua. Tuhannen valovuoden päässä olevan tähden kohdalla sama havainto tehdään aikaisintaan tuhannen vuoden kuluttua.


      • AikaFaktaa
        info.liikkuu.valon.nopeu kirjoitti:

        Informaatio etenee aina enintään valon nopeudella. Katsominen on informaation vastaanottamista, ei sen lähettämistä. Informaatio katsottavasta kohteesta voi tulla luoksesi korkeintaan valon nopeudella. Lähettäjä ei tiedä sitä että informaatio saapui perille ellet lähetä siitä hänelle tietoa takaisinpäin.

        Katseella itsellään ei ole nopeutta. Siinähän vain käännetään vastaanottoantenni eli silmät suuntaan josta halutaan sähkömagneettista säteilyä vastaanottaa. Tieto katseen suunnasta välittyy esimerkiksi katsojan kasvoista heijastuvan valon avulla eli sekin tieto etenee enintään valon nopeudella.

        Mikään ei estä muuttamasta katselusuuntaansa sadan valovuoden päässä olevasta tähdestä tuhannen valovuoden päässä olevaan tähteen vaikkapa puolessa sekunnissa. Minkään informaation nopeus ei kuitenkaan ole tuolloin 900 valovuotta puolessa sekunnissa. Sadan valovuoden päässä olevan tähden kohdalla tuo katseen kääntäminen huomataan teleskoopilla katsomalla aikaisintaan sadan vuoden kuluttua. Tuhannen valovuoden päässä olevan tähden kohdalla sama havainto tehdään aikaisintaan tuhannen vuoden kuluttua.

        Joo, tuo oma aikasempi teoria meni hieman syteen, ja saveen.:-(
        Eli tosiaan, näkö ei kulje valon nopeutta, ?? , vaan se kohteesta tuleva näkymä , jota katselemme. jos joku tiirailee meitä 10 tuhannen valovuoden päästä, ja näkee tämän nykyisen maan vuodella 2017, niin hänen täytyy katsoa meitä 10 000 vuoden päässä tulevaisuudesta, eikä siis menneisyydestä.

        ..ei pitäis pohtia liian vaikeita asioita enää yöllä;-) .


      • trololooo
        AikaFaktaa kirjoitti:

        No joo,, toki näet heti 1000 valovuoden päähän, mutta näkymä jonka näet on 1000 vuotta vanhaa, eli planeeltalta kulkeva "valonäkymä" on kulkenut sinun silmääsi 1000 vuotta.

        *psssst*
        Rakas pieni trollini!
        Tällä kertaa sinä unohdit nyt kertoa kahdella tuon tuhat valovuotta, kuten juuri tolla ylempänä väitit! :)
        Mut ehkä sä vielä opit?


      • AikaFaktaa
        trololooo kirjoitti:

        *psssst*
        Rakas pieni trollini!
        Tällä kertaa sinä unohdit nyt kertoa kahdella tuon tuhat valovuotta, kuten juuri tolla ylempänä väitit! :)
        Mut ehkä sä vielä opit?

        No oli miten oli, saimpahan ainaki keskustelun kääntymään oikeille raiteille. Yli puolet vastauksista käsittelee jotain ihan muuta aihetta kuin, mitä aloittaja on kysynyt ?

        Myönsin virheen... ja korjasin väärän teorian oikeaksi. (Olen näissä tähtiasioissa vielä "maallikko". )


      • TähtiUrvelo
        AikaFaktaa kirjoitti:

        No oli miten oli, saimpahan ainaki keskustelun kääntymään oikeille raiteille. Yli puolet vastauksista käsittelee jotain ihan muuta aihetta kuin, mitä aloittaja on kysynyt ?

        Myönsin virheen... ja korjasin väärän teorian oikeaksi. (Olen näissä tähtiasioissa vielä "maallikko". )

        ojoo, ehkä oli vähän turhan töykeä kommentti multakin. Kaikkihan me on joskus noviisejakin oltu, ja onhan nää asiat vähän vaikeita käsittää aluks (ja monasti myös entistä vaikeampia "lopuks") joten: Sori, ja hyviä jatkoja tähtitieteen parissa. Onhan nää kuitenkin ihan mielenkiintosia juttuja, ja sitä mieenkiintosemmiks menee mitä syvemmälle sukeltaa. Mut virheitähän toki tehdään me kaikki, mut niistähän se sit parhaiten oppii, kunhan tajuu n
        vaan ottaa niistä opiks. Ja johan mä itsekin ainakin kertaalleen kämmäsin tässäkin ketjussa, et: Oppimista tässä on vielä mullakin! :D :/


    • ami46

      Enkelit tarkkailee maapalloa. Lähempänä kuin uskoisittekaan. vuonna 2016 se planeetta X kävi maapallon läheisyydessä.24.6.2016 klo 5.00 näkyi enkeleitä . Heitä sanottiin ufoiksi. 7.10.2016 klo14.04. Todellinen tappajamysky satoja Haitilaisia kuoli. Nyt se planeetta loittonee jo maapallosta, näkyy vielä kaukoputkella

      • 64ima

        Höpöhöpö.Olet taas vetänyt jotain psyykelääkkeitä.


    • Ajatusleikki:
      Jos kaukoputki olisi pituudeltaan fyysisesti tuon valtavan matkan pituinen ja sitä työntäisi toisesta päästä, niin miten teorian mukaan fyysinen objekti liikkuisi? Jos se kaukoputki työntyisi samaan aikaan siellä toisessa päässä, niin silloin käden liike olisi valoa nopeampaa ja välitöntä. Jos taas informaatio saavuttaisi toisen pään yhtä myöhään kuin valolla kestää, niin mihin se eteenpäin työnnetty fyysinen objekti siitä lyhentyisi. Ellei ollut teleskooppi :)

      • Todellinen toiminta on teorian mukaista, eli kappaleessa mekaaninen pulssi liikkuu äänen nopeudella väliaineessa.

        Esimerkiksi alumiinitangossa 6420 m/s.


      • Kollimaattori kirjoitti:

        Todellinen toiminta on teorian mukaista, eli kappaleessa mekaaninen pulssi liikkuu äänen nopeudella väliaineessa.

        Esimerkiksi alumiinitangossa 6420 m/s.

        Muuttuuko alumiinitanko aaltomuotoiseksi ja takaisin, kuten kvantti-ilmiöissä?


      • Polariz- kirjoitti:

        Muuttuuko alumiinitanko aaltomuotoiseksi ja takaisin, kuten kvantti-ilmiöissä?

        Alumiinitangossa liikkuu tangon päähän lyötäessä mekaaninen sokkiaalto. Mitään kvantti-ilmiöitä ei tarvita ilmiön kuvaamiseen tai ymmärtämiseen.


      • PitkätPutket

        Vois kyllä jäädä ihan pelkällä kädellä työntelemättä tollai 10 000 valovuotta pitkä kaukoputki!? Saattaa olla kohtalaisen painava? Ja mä en nyt oikei ymmärtäny tota sun valon nopeudella kulkeva käsi juttuas!? Mitä sä oikein sillä tarkotit? Ja toisaalta: Kyllä se fotoni kulkee siellä kaukoputken sisälläkin ohan sitä samaa nopeutta kun sen ulkopuolellakin (eli vaikka täysin ilman koko kaukoputkea) joten jos sä kuvittelit tekeväs tolla pelillä jonku oikotien nähdä sillä samalla hetkellä kun kurkkaat siihen ultrapitkään kaukoputkees 10 000 valovuoden päähän siellä päässä sen saman hetken, niin: Juuei!

        Mut jos sä jotenki sit saisit sen liikkumaan, ni ihan samanlailla se liikkuu ees-taas kuten mikä tahansa paljon lyhempikin kappale. Mut jos esmes siellä 10 000 valovuoden päässä joku "koppari" saa siitä sun kaukoputkesta kuhmun otsaansa, kun sä sitä työntelet, niin se sen sut kiroova tekstiviesti kestää tulla perille sun puhelimeen vasta 10 000 vuoden päästä, vaikka se liikkuiskin valon nopeutta.


      • PitkätPutket kirjoitti:

        Vois kyllä jäädä ihan pelkällä kädellä työntelemättä tollai 10 000 valovuotta pitkä kaukoputki!? Saattaa olla kohtalaisen painava? Ja mä en nyt oikei ymmärtäny tota sun valon nopeudella kulkeva käsi juttuas!? Mitä sä oikein sillä tarkotit? Ja toisaalta: Kyllä se fotoni kulkee siellä kaukoputken sisälläkin ohan sitä samaa nopeutta kun sen ulkopuolellakin (eli vaikka täysin ilman koko kaukoputkea) joten jos sä kuvittelit tekeväs tolla pelillä jonku oikotien nähdä sillä samalla hetkellä kun kurkkaat siihen ultrapitkään kaukoputkees 10 000 valovuoden päähän siellä päässä sen saman hetken, niin: Juuei!

        Mut jos sä jotenki sit saisit sen liikkumaan, ni ihan samanlailla se liikkuu ees-taas kuten mikä tahansa paljon lyhempikin kappale. Mut jos esmes siellä 10 000 valovuoden päässä joku "koppari" saa siitä sun kaukoputkesta kuhmun otsaansa, kun sä sitä työntelet, niin se sen sut kiroova tekstiviesti kestää tulla perille sun puhelimeen vasta 10 000 vuoden päästä, vaikka se liikkuiskin valon nopeutta.

        Paradoksin pointti ei ole siinä kuinka vahva olen tai kuinka toteuttamiskelpoinen tuollainen olisi, vaan miten suhteellisuusteoria ennustaisi käyvän.


    • Meinaan että mihin se eteenpäin työnnetty alumiini katoaa sillä aikaa, jos ei toiseen päähän.

      • ei_mihinkään

        Ei kai se mihinkään katoa, vaan siirtyy eteenpäin, eli poispäin työntäjästä. esim. jos 1 valovuoden pituista alumiinitankoa työntää metrin, niin se siirtyy metrin poispäin työntäjästä.

        Työnnetty matka näkyy molemmissa päässä yhtäaikaa, olettaen tietysti että tämä alumiinitanko ei veny, eikä kutistu matkalla, vaan on täysin kiinteää, ja kovaa materiaalia.


      • Mekaniikassa joskus käytetty käsite "jäykkä kappale" eli täysin kokoonpuristumaton materiaali ei ole yhteensopiva suhteellisuusteorian kanssa. Sellaisessa kappaleessa pitäisi paineaallon edetä äärettömän suurella nopeudella, mikä ei ole mahdollista.

        "Meinaan että mihin se eteenpäin työnnetty alumiini katoaa sillä aikaa, jos ei toiseen päähän."

        Tanko puristuu pitkältä matkalta kasaan kun työnnät sitä. Alumiinin atomit painuvat hieman tiheämpään kuin mitä ne tasapainotilassa olisivat. Tangosta tulee hieman paksumpi kun se myötää sivusuunnassa.

        Pitkää metallikappaletta voi puristaa varsin paljon kasaan ilman että se menee rikki. Esimerkkinä ratakiskot ja niiden lämpölaajeneminen. Kisko voi lämpölaajenemisen takia pidentyä metrin verran muutaman kilometrin matkalla, mutta kiskon suuntainen puristus pitää sen pituuden vakiona. Miljoonan kilometrin matkalle saisi metrin verran pituuden muutosta mahtumaan ilman että sitä juurikaan huomaisi tankoa puristavana voimana.

        Jos työnnät liian suurella voimalla niin tankoon tulee pysyviä muodonmuutoksia eli siitä tulee pysyvästi paksumpi ja lyhyempi, siihen tulee halkeamia, se katkeaa tai se nurjahtaa.

        Tällä ei enää ole juurikaan tekemistä tähtitieteen kanssa.


      • ItTakes2toTanko
        ExB kirjoitti:

        Mekaniikassa joskus käytetty käsite "jäykkä kappale" eli täysin kokoonpuristumaton materiaali ei ole yhteensopiva suhteellisuusteorian kanssa. Sellaisessa kappaleessa pitäisi paineaallon edetä äärettömän suurella nopeudella, mikä ei ole mahdollista.

        "Meinaan että mihin se eteenpäin työnnetty alumiini katoaa sillä aikaa, jos ei toiseen päähän."

        Tanko puristuu pitkältä matkalta kasaan kun työnnät sitä. Alumiinin atomit painuvat hieman tiheämpään kuin mitä ne tasapainotilassa olisivat. Tangosta tulee hieman paksumpi kun se myötää sivusuunnassa.

        Pitkää metallikappaletta voi puristaa varsin paljon kasaan ilman että se menee rikki. Esimerkkinä ratakiskot ja niiden lämpölaajeneminen. Kisko voi lämpölaajenemisen takia pidentyä metrin verran muutaman kilometrin matkalla, mutta kiskon suuntainen puristus pitää sen pituuden vakiona. Miljoonan kilometrin matkalle saisi metrin verran pituuden muutosta mahtumaan ilman että sitä juurikaan huomaisi tankoa puristavana voimana.

        Jos työnnät liian suurella voimalla niin tankoon tulee pysyviä muodonmuutoksia eli siitä tulee pysyvästi paksumpi ja lyhyempi, siihen tulee halkeamia, se katkeaa tai se nurjahtaa.

        Tällä ei enää ole juurikaan tekemistä tähtitieteen kanssa.

        Ja voi vaatia hieman energiaakin ton pitusen tangon työntäminen?
        Tuli nyt huvitettua itteensä laskemalla moisen massa, ja näyttäs olevan:
        8x10^17 kg. (Jos en tehny paljoa pilkkuvirheitä) joten työn kaavastahan ton sit helposti sais tsekattua, et monta wattia. Ja jos nyt olettais et sen toinen pää olis ankkuroituna maahan, ni vois vaatii kohtalaisen hyvät perustuksetkin?
        Ja joku lujuuslaskennan hallitseva inssi varmaan seuraavaks laskee et toi tanko hajois tossa tapauksessa jo omaan painoonsa? Ellei sit kelluis kokonaa avaruudessa?


      • ItTakes2toTanko kirjoitti:

        Ja voi vaatia hieman energiaakin ton pitusen tangon työntäminen?
        Tuli nyt huvitettua itteensä laskemalla moisen massa, ja näyttäs olevan:
        8x10^17 kg. (Jos en tehny paljoa pilkkuvirheitä) joten työn kaavastahan ton sit helposti sais tsekattua, et monta wattia. Ja jos nyt olettais et sen toinen pää olis ankkuroituna maahan, ni vois vaatii kohtalaisen hyvät perustuksetkin?
        Ja joku lujuuslaskennan hallitseva inssi varmaan seuraavaks laskee et toi tanko hajois tossa tapauksessa jo omaan painoonsa? Ellei sit kelluis kokonaa avaruudessa?

        Joo, alumiinin käyttäytymisellä ei ole tekemistä tähtitieteen kanssa, mutta entäs tuo että suhteellisuusteoriaan ei sovi ilmeisesti edes teorian mukaan "jäykän kappaleen" liikkuminen sen jälkeen kun sen pituus ylittää tietyn rajan, ainakaan sinun mukaasi. Luulisi että toimiva teoria antaisi ennustettavan selitysmallin sille mitä aivan konkreettisen kappaleen liikkeelle tapahtuisi siinä... ilman että täytyy turvautua aineen lytistymiseen tai taipumiseen, koska paradoksista voi sulkea ne seikat pois lisäämällä kappaleen/ kehikon kestävyyttä niin paljon kun tarvitaan.


      • VaniTasVanitatumKaniPani
        Polariz- kirjoitti:

        Joo, alumiinin käyttäytymisellä ei ole tekemistä tähtitieteen kanssa, mutta entäs tuo että suhteellisuusteoriaan ei sovi ilmeisesti edes teorian mukaan "jäykän kappaleen" liikkuminen sen jälkeen kun sen pituus ylittää tietyn rajan, ainakaan sinun mukaasi. Luulisi että toimiva teoria antaisi ennustettavan selitysmallin sille mitä aivan konkreettisen kappaleen liikkeelle tapahtuisi siinä... ilman että täytyy turvautua aineen lytistymiseen tai taipumiseen, koska paradoksista voi sulkea ne seikat pois lisäämällä kappaleen/ kehikon kestävyyttä niin paljon kun tarvitaan.

        Hyvä pointti! Lättymaan kannalta lujuuslaskenta ja suhtis ovat kumpikin ikäviä juttuja ja ne pitääkin osoittaa paikkansapitämättömiksi mieluummin omilla pätevyysalueillaan, mutta mikä tahansa muukin alue käy. Epäilyksen varjo lankesi nyt vahvasti teorioiden niskaan. Hieno homma.


      • Realisti2

        Alumiinin käyttäytymisellä ei ehkä ole tekemistä tähtitieteen kanssa suoraan, mut välillisesti kyllä, kun sillä on merkitystä vaikkapa tähtitieteen havaintojen tekemisessä.
        Alumiina käytetään hienona pulverina ja perkloraattina (hapetin) rakettipolttoaineissa, kun se on niin riivatun reaktiivista ja maankuoren runsaimpana metallina keskimäärin melko halpaa hankkia. Puhdasta alumiinia taas ei kai juuri kukaan selväjärkinen kuvittele avaruuteenkaan tankona työntävänsä. Seoksiin alumiini hakeutuu kyllä hanakasti vapaissa määräsuhteissa. Avaruus- ja teollisuuslaadut eivät välttämättä poikkea toisistaan paljoa. Millä standardiprofiililla ollaan nyt liikkeellä ja montako triljoonaa tonnia laitetaan ensi alkuun? Avaruudessa alumiini ei eloksoidu tai välttämättä edes kylmähitsaudu, mutta pitääkö silti pinnoittaa ja millä aineella?

        Jos sitten ajateltaisiin avaruuteen törröttävän alumiiniputken liittämistä Maahan, niin kantsii varmaan ajatella uudestaan. Vapaasti "tähtienvälisessä avaruudessa" kelluessaankaan ei valovuosien pituinen alumiinitanko välttämättä pitkään pysyisi suorana. Jos joku tähti osuisi lähettyville niin putkikin taipuisi mm. säteilyn aiheuttamien lämpölaajenemisten ja -kutistumisten yms. seurauksena. Nurjahduksen kaveriksi ilmaantuisi lommahdus jos läpimittaa kasvattaisi lisälujuuden hankkimiseksi. (vrt. alumiinitölkit) Miten 10000 vv:n törö edes kuvitteellisesti liitettäisiin Maahan niin, että se mukautuisi Maan liikkeisiin? Tyssääntymisen lisäksi kiertymät, leikkautumiset ja venymiset katkaisisivat törröttimen, vaikka sen jollakin ilveellä saisi kestämään gravitaation aiheuttamaa vetoa.

        Paitsi kuvitteellista teknologiaa putken valmistukseen tarvitaan sen koossa pitämiseen myös kuvitteellista tukikehikkoa Polarizin esittämään tyyliin.


      • suoraviivainen_liike
        Polariz- kirjoitti:

        Joo, alumiinin käyttäytymisellä ei ole tekemistä tähtitieteen kanssa, mutta entäs tuo että suhteellisuusteoriaan ei sovi ilmeisesti edes teorian mukaan "jäykän kappaleen" liikkuminen sen jälkeen kun sen pituus ylittää tietyn rajan, ainakaan sinun mukaasi. Luulisi että toimiva teoria antaisi ennustettavan selitysmallin sille mitä aivan konkreettisen kappaleen liikkeelle tapahtuisi siinä... ilman että täytyy turvautua aineen lytistymiseen tai taipumiseen, koska paradoksista voi sulkea ne seikat pois lisäämällä kappaleen/ kehikon kestävyyttä niin paljon kun tarvitaan.

        Niin , tuollaisen kappaleen tekeminenhän olisi käytännössä mahdotonta, koska maapallon mineraalivarat ei ei edes riittäisi siihen; MUTTA tässähän olikin kyse teoriasta, eikä käytännöstä. Eli, oletetaan että jäykkää, kutistumatonta/venymätöntä yhden valovuoden pituista kappaletta siirtää metrin tyhjiössä, eli avaruudessa. Näkyykö kappaleen siirtyminen samaan aikaa molemmissa päissä.?? Oma vastaus on, että näkyy.

        Vaikka valolta menee kyseiseen matkaan vuosi, niin sillä ei ole mitään merkitystä. Metrin siirtymä näkyy täsmälleen samaan aikaan molemmissa päissä, oli kappale sitten 1 valovuoden, tai 1 kilometrin pituinen. Kappaleen siirtymään liittyvät lait pätevät samanlailla maan päällä, sekä tyhjiössä.


      • suoraviivainen_liike
        suoraviivainen_liike kirjoitti:

        Niin , tuollaisen kappaleen tekeminenhän olisi käytännössä mahdotonta, koska maapallon mineraalivarat ei ei edes riittäisi siihen; MUTTA tässähän olikin kyse teoriasta, eikä käytännöstä. Eli, oletetaan että jäykkää, kutistumatonta/venymätöntä yhden valovuoden pituista kappaletta siirtää metrin tyhjiössä, eli avaruudessa. Näkyykö kappaleen siirtyminen samaan aikaa molemmissa päissä.?? Oma vastaus on, että näkyy.

        Vaikka valolta menee kyseiseen matkaan vuosi, niin sillä ei ole mitään merkitystä. Metrin siirtymä näkyy täsmälleen samaan aikaan molemmissa päissä, oli kappale sitten 1 valovuoden, tai 1 kilometrin pituinen. Kappaleen siirtymään liittyvät lait pätevät samanlailla maan päällä, sekä tyhjiössä.

        ...Tosin.., jos oletetaan että kummassakin päässä on ukot radiopuhelimien kanssa ilmoittamassa siirtymisestä, niin tieto siitä saadaan jompaan kumpaan päähän vasta vuoden päästä.

        Eli ts. elämme tavallaan koko maailmankaikkeudessa samaa aikaa, vaikka planeettojen, ja tähtien väliset etäisyydet ovat käsittämättömän suuria. Aikaerot johtuu siis etäisyyksistä.


      • suoraviivainen_liike kirjoitti:

        Niin , tuollaisen kappaleen tekeminenhän olisi käytännössä mahdotonta, koska maapallon mineraalivarat ei ei edes riittäisi siihen; MUTTA tässähän olikin kyse teoriasta, eikä käytännöstä. Eli, oletetaan että jäykkää, kutistumatonta/venymätöntä yhden valovuoden pituista kappaletta siirtää metrin tyhjiössä, eli avaruudessa. Näkyykö kappaleen siirtyminen samaan aikaa molemmissa päissä.?? Oma vastaus on, että näkyy.

        Vaikka valolta menee kyseiseen matkaan vuosi, niin sillä ei ole mitään merkitystä. Metrin siirtymä näkyy täsmälleen samaan aikaan molemmissa päissä, oli kappale sitten 1 valovuoden, tai 1 kilometrin pituinen. Kappaleen siirtymään liittyvät lait pätevät samanlailla maan päällä, sekä tyhjiössä.

        Ei näy, on minun vastaukseni. Mikään informaatio ei kulje ylivalonnopeudella. ExB selitti asian aivan oikein.
        Mekaniikassa voidaan puhua jäykästä kappaleesta, mutta se on vain karkeistus. Jos asiaa tarkastellaan atomitasolla, jäykkää ainetta ei ole.
        Ääniaallon eteneminen kiinteässä aineessa perustuu aineen osasten liikeeseen eli esimerkiksi atomien värähtelyliikkeeseen normaalitilansa molemmin puolin. Sama ilmiö hidastaa palkin toiseen päähän kohdistuvan liikkeen siirtymistä toiseen päähän. Käytännön mitoissa sillä ei ole merkitystä, mutta ilmiö on aivan todellinen ja estää ylivalonnopeudella siirtymisen.

        Tietenkin jos oletetaan, että olisi täysin jäykkä kappale, niin silloin varmaan, mutta kun täysin jäykkä kappale on utopiaa, ei fysiikkaa.


      • Lue_kysymys
        agnoskepo kirjoitti:

        Ei näy, on minun vastaukseni. Mikään informaatio ei kulje ylivalonnopeudella. ExB selitti asian aivan oikein.
        Mekaniikassa voidaan puhua jäykästä kappaleesta, mutta se on vain karkeistus. Jos asiaa tarkastellaan atomitasolla, jäykkää ainetta ei ole.
        Ääniaallon eteneminen kiinteässä aineessa perustuu aineen osasten liikeeseen eli esimerkiksi atomien värähtelyliikkeeseen normaalitilansa molemmin puolin. Sama ilmiö hidastaa palkin toiseen päähän kohdistuvan liikkeen siirtymistä toiseen päähän. Käytännön mitoissa sillä ei ole merkitystä, mutta ilmiö on aivan todellinen ja estää ylivalonnopeudella siirtymisen.

        Tietenkin jos oletetaan, että olisi täysin jäykkä kappale, niin silloin varmaan, mutta kun täysin jäykkä kappale on utopiaa, ei fysiikkaa.

        MUTTA, kun tässä ei ole kyse mistää mekaniikasta, fysiikasta, eikä ääniaalloista, vaan suhteellisuusteoriasta, ja aikaeroista avaruudessa Opettele lukemaan kysymykset, ennenkuin kirjottelet tänne potaskaa. !!!!


      • Lue_kysymys kirjoitti:

        MUTTA, kun tässä ei ole kyse mistää mekaniikasta, fysiikasta, eikä ääniaalloista, vaan suhteellisuusteoriasta, ja aikaeroista avaruudessa Opettele lukemaan kysymykset, ennenkuin kirjottelet tänne potaskaa. !!!!

        En vastannut alkuperäiseen kysymykseen, vaan seuraavan väitteeseen:
        " Metrin siirtymä näkyy täsmälleen samaan aikaan molemmissa päissä, oli kappale sitten 1 valovuoden, tai 1 kilometrin pituinen. "

        Vaan kun ei näy. Jos näkyisi, se antaisi mahdollisuuden ylivalonnopeudella tapahtuvaan informaationsiirtoon, jonka nimen omaan suhteellisuusteoria kieltää. Kiinteiden aineiden fysiikka selittää miksi väittämäsi ilmiö ei pidä paikkaansa ja sillä on tekemistä aaltoliikkeen myös kanssa.


      • suoraviivainen_liike
        agnoskepo kirjoitti:

        En vastannut alkuperäiseen kysymykseen, vaan seuraavan väitteeseen:
        " Metrin siirtymä näkyy täsmälleen samaan aikaan molemmissa päissä, oli kappale sitten 1 valovuoden, tai 1 kilometrin pituinen. "

        Vaan kun ei näy. Jos näkyisi, se antaisi mahdollisuuden ylivalonnopeudella tapahtuvaan informaationsiirtoon, jonka nimen omaan suhteellisuusteoria kieltää. Kiinteiden aineiden fysiikka selittää miksi väittämäsi ilmiö ei pidä paikkaansa ja sillä on tekemistä aaltoliikkeen myös kanssa.

        No juurihan kirjoitin, että tieto siirtymisestä tulee toiseen päähän vasta vuoden kuluttua.
        Jos se vastaanottaja kuittaa siirtymän valomerkillä tai radioviestillä, niin työntäjä saa vastauksen noin vuoden kuluttua. Vaikka periaatteessa se siirtyy samalla hetkellä molemmissa päissä. Tiedon, ja infon kulku on eri asia kuin suhteellinen aika.

        Jos tuota siirtoa katsotaan kaukaa valovuosien päästä, (esim. tuon janan paikalla olis vaikka se 1 vv pituinen aluputki), niin varmasti molemmat päät siirtyy yhtäaikaa.:

        https://images.cdn.yle.fi/image/upload/fl_keep_iptc,f_auto,fl_progressive/q_88/w_2884,h_1622,c_crop,x_0,y_27/w_700/v1504689773/39-42624859afbd7ceefd2.jpg


      • agnoskepo kirjoitti:

        En vastannut alkuperäiseen kysymykseen, vaan seuraavan väitteeseen:
        " Metrin siirtymä näkyy täsmälleen samaan aikaan molemmissa päissä, oli kappale sitten 1 valovuoden, tai 1 kilometrin pituinen. "

        Vaan kun ei näy. Jos näkyisi, se antaisi mahdollisuuden ylivalonnopeudella tapahtuvaan informaationsiirtoon, jonka nimen omaan suhteellisuusteoria kieltää. Kiinteiden aineiden fysiikka selittää miksi väittämäsi ilmiö ei pidä paikkaansa ja sillä on tekemistä aaltoliikkeen myös kanssa.

        Kirjoitat asiaa. Mikään fysiikan lakeja noudattava aine ei voi olla täysin jäykkää. Kysymys täysin jäykän kappaleen käyttäytymisestä suhteellisuusteorian kannalta on siis muotoa

        - Miten täysin jäykkä tanko käyttäytyisi fysiikan lakien mukaan, jos fysiikan lait eivät olisi voimassa?


      • suoraviivainen_liike kirjoitti:

        No juurihan kirjoitin, että tieto siirtymisestä tulee toiseen päähän vasta vuoden kuluttua.
        Jos se vastaanottaja kuittaa siirtymän valomerkillä tai radioviestillä, niin työntäjä saa vastauksen noin vuoden kuluttua. Vaikka periaatteessa se siirtyy samalla hetkellä molemmissa päissä. Tiedon, ja infon kulku on eri asia kuin suhteellinen aika.

        Jos tuota siirtoa katsotaan kaukaa valovuosien päästä, (esim. tuon janan paikalla olis vaikka se 1 vv pituinen aluputki), niin varmasti molemmat päät siirtyy yhtäaikaa.:

        https://images.cdn.yle.fi/image/upload/fl_keep_iptc,f_auto,fl_progressive/q_88/w_2884,h_1622,c_crop,x_0,y_27/w_700/v1504689773/39-42624859afbd7ceefd2.jpg

        " Vaikka periaatteessa se siirtyy samalla hetkellä molemmissa päissä."

        Vaan kun ei siirry. Jos siirtyisi, voisimme morsettaa sauvaa tönimällä tietoa reippaasti yli valonnopeuden. Vastauskin tulisi melkein heti, kun toisessa päässä tökittäisiin samaa sauvaa, vaikka kohteiden ero olisi valovuosi.


      • suoraviivainen_liike
        agnoskepo kirjoitti:

        " Vaikka periaatteessa se siirtyy samalla hetkellä molemmissa päissä."

        Vaan kun ei siirry. Jos siirtyisi, voisimme morsettaa sauvaa tönimällä tietoa reippaasti yli valonnopeuden. Vastauskin tulisi melkein heti, kun toisessa päässä tökittäisiin samaa sauvaa, vaikka kohteiden ero olisi valovuosi.

        "...Vaan kun ei siirry. Jos siirtyisi,..."

        No se on sinun teoriasi, ja minulla on omani, ja pidän siitä 100% kiinni. !! Turha väitellä, kun tuollaista ei ole voitu käytännössä todistaa.

        En tiedä ymmärätkö edes mitä tarkoitan,??? Joo emme tosiaankaan voi morsettaa yli valon nopeudella, mutta voisimme TEORIASSA (ei käytännössä) siirtää valovuoden pituista kappaletta esim. 10km/h nopeudella. ja se sama siirtonopeus näkyisi myös toisessa päässä. Infoa tästä siirtymästä saataisiin toiseen päähän ihmisen havaintokyvyllä korkeintaan vuoden päästä !!!
        Jos siirtymää katsottasiin sivusuunnasta kaukoputkella jostain kaukaa kymmenien valovuosien päästä, niin kyllähän se kappale siirtyisi ihan samassa ajassa molemmissa päissä.


      • suoraviivainen_liike kirjoitti:

        "...Vaan kun ei siirry. Jos siirtyisi,..."

        No se on sinun teoriasi, ja minulla on omani, ja pidän siitä 100% kiinni. !! Turha väitellä, kun tuollaista ei ole voitu käytännössä todistaa.

        En tiedä ymmärätkö edes mitä tarkoitan,??? Joo emme tosiaankaan voi morsettaa yli valon nopeudella, mutta voisimme TEORIASSA (ei käytännössä) siirtää valovuoden pituista kappaletta esim. 10km/h nopeudella. ja se sama siirtonopeus näkyisi myös toisessa päässä. Infoa tästä siirtymästä saataisiin toiseen päähän ihmisen havaintokyvyllä korkeintaan vuoden päästä !!!
        Jos siirtymää katsottasiin sivusuunnasta kaukoputkella jostain kaukaa kymmenien valovuosien päästä, niin kyllähän se kappale siirtyisi ihan samassa ajassa molemmissa päissä.

        Tasaisessa liikkeessä näin on. Alkuperäinen väite oli ihan jotain muuta:
        " Polariz 11.10.2017 19:50
        Ajatusleikki:
        Jos kaukoputki olisi pituudeltaan fyysisesti tuon valtavan matkan pituinen ja sitä työntäisi toisesta päästä, niin miten teorian mukaan fyysinen objekti liikkuisi? Jos se kaukoputki työntyisi samaan aikaan siellä toisessa päässä, niin silloin käden liike olisi valoa nopeampaa ja välitöntä. Jos taas informaatio saavuttaisi toisen pään yhtä myöhään kuin valolla kestää, niin mihin se eteenpäin työnnetty fyysinen objekti siitä lyhentyisi. "

        Kaukoputki ei työntyisi samaan aikaan toisesta päästä. Siitä on minun mielestäni tässä keskustelussa ollut kysymys. Voimavaikutuksen siirto ei onnitu ylivalonnopeudella kiinteällä kappaleellakaan.


      • huoooohhhhh
        agnoskepo kirjoitti:

        Tasaisessa liikkeessä näin on. Alkuperäinen väite oli ihan jotain muuta:
        " Polariz 11.10.2017 19:50
        Ajatusleikki:
        Jos kaukoputki olisi pituudeltaan fyysisesti tuon valtavan matkan pituinen ja sitä työntäisi toisesta päästä, niin miten teorian mukaan fyysinen objekti liikkuisi? Jos se kaukoputki työntyisi samaan aikaan siellä toisessa päässä, niin silloin käden liike olisi valoa nopeampaa ja välitöntä. Jos taas informaatio saavuttaisi toisen pään yhtä myöhään kuin valolla kestää, niin mihin se eteenpäin työnnetty fyysinen objekti siitä lyhentyisi. "

        Kaukoputki ei työntyisi samaan aikaan toisesta päästä. Siitä on minun mielestäni tässä keskustelussa ollut kysymys. Voimavaikutuksen siirto ei onnitu ylivalonnopeudella kiinteällä kappaleellakaan.

        "...Voimavaikutuksen siirto ei onnitu ylivalonnopeudella kiinteällä kappaleellakaan..."

        No ei kai sen tarvikkaan siirtyä valonnopeudella. Voihan sitä kappaletta siirtää vaikka vain 20 km/h. ???
        Jos, puolessa välissä olisi avaruusalus havaitsemassa siirtymää, niin ei se objekti, tai alumiiniputki mene siinä kohtaa valonnopeutta, vaan 20 km/h.


      • huoooohhhhh kirjoitti:

        "...Voimavaikutuksen siirto ei onnitu ylivalonnopeudella kiinteällä kappaleellakaan..."

        No ei kai sen tarvikkaan siirtyä valonnopeudella. Voihan sitä kappaletta siirtää vaikka vain 20 km/h. ???
        Jos, puolessa välissä olisi avaruusalus havaitsemassa siirtymää, niin ei se objekti, tai alumiiniputki mene siinä kohtaa valonnopeutta, vaan 20 km/h.

        Miten voi olla niin vaikeaa?
        Kyse on siitä, ettei kappaletta toisesta päästä työntämällä saada välitetyksi voimavaikutusta toiseen päähän yli valonnopeudella.
        Jos valovuoden pituisen palkin puolessa välissä olisi avaruusalus, se joutuisi odottamaan palkin töytäisemisen jälkeen (paljon) yli puoli vuotta, ennen kuin sillä kohtaa palkkia tapahtuisi mitään liikettä.


      • suoraviivainen_liike
        agnoskepo kirjoitti:

        Miten voi olla niin vaikeaa?
        Kyse on siitä, ettei kappaletta toisesta päästä työntämällä saada välitetyksi voimavaikutusta toiseen päähän yli valonnopeudella.
        Jos valovuoden pituisen palkin puolessa välissä olisi avaruusalus, se joutuisi odottamaan palkin töytäisemisen jälkeen (paljon) yli puoli vuotta, ennen kuin sillä kohtaa palkkia tapahtuisi mitään liikettä.

        No tajushan täällä joku asian. Monta kertaa olen jankuttanu, että työntäjä saa informaatiota siirrosta vasta vuoden kuluttua, Joo, ja kyllä,, puolessa välissä tarkkailija vasta puolen vuoden kuluttua. Jos työntäjä katsoo palkin loppupäätä kaukoputkella, niin hän näkee oman siirtonsa vasta vuoden kuluttua !!!

        Mutta, jos siirtoa seurataan kaukaa sivusta SUURESSA MITTAKAAVASSA, niin kyllä se putki liikkuu periaatteessa yhtäaikaa molemmissa päissä. Jos oletetaan, että se ei veny, eikä kutistu, vaan liikkuu yhtenäisenä putkena.


      • suoraviivainen_liike kirjoitti:

        No tajushan täällä joku asian. Monta kertaa olen jankuttanu, että työntäjä saa informaatiota siirrosta vasta vuoden kuluttua, Joo, ja kyllä,, puolessa välissä tarkkailija vasta puolen vuoden kuluttua. Jos työntäjä katsoo palkin loppupäätä kaukoputkella, niin hän näkee oman siirtonsa vasta vuoden kuluttua !!!

        Mutta, jos siirtoa seurataan kaukaa sivusta SUURESSA MITTAKAAVASSA, niin kyllä se putki liikkuu periaatteessa yhtäaikaa molemmissa päissä. Jos oletetaan, että se ei veny, eikä kutistu, vaan liikkuu yhtenäisenä putkena.

        "Mutta, jos siirtoa seurataan kaukaa sivusta SUURESSA MITTAKAAVASSA, niin kyllä se putki liikkuu periaatteessa yhtäaikaa molemmissa päissä."

        Juuri tästä on kysymys. Ei liiku ei, mikäli voimavaikutus kohdistuu vain putken toiseen päähän.
        Itse asiassa toista päätä kaukoputkella katsomalla liike näkyisi vasta yli kahden vuoden päästä tönäisystä. Putki todella kutistuu, koska "paineaalto" ei voi liikkua valonnopeudella.


      • KuinKöyhänTalonPorsaat
        agnoskepo kirjoitti:

        Ei näy, on minun vastaukseni. Mikään informaatio ei kulje ylivalonnopeudella. ExB selitti asian aivan oikein.
        Mekaniikassa voidaan puhua jäykästä kappaleesta, mutta se on vain karkeistus. Jos asiaa tarkastellaan atomitasolla, jäykkää ainetta ei ole.
        Ääniaallon eteneminen kiinteässä aineessa perustuu aineen osasten liikeeseen eli esimerkiksi atomien värähtelyliikkeeseen normaalitilansa molemmin puolin. Sama ilmiö hidastaa palkin toiseen päähän kohdistuvan liikkeen siirtymistä toiseen päähän. Käytännön mitoissa sillä ei ole merkitystä, mutta ilmiö on aivan todellinen ja estää ylivalonnopeudella siirtymisen.

        Tietenkin jos oletetaan, että olisi täysin jäykkä kappale, niin silloin varmaan, mutta kun täysin jäykkä kappale on utopiaa, ei fysiikkaa.

        Ei vaan nimenomaan jos asiaa tarkastellaan atomitasolla, niin jokanen molekyylikin on jäykkä kappale!

        "In the study of special relativity, a perfectly rigid body does not exist; and objects can only be assumed to be rigid if they are not moving near the speed of light. In quantum mechanics a rigid body is usually thought of as a collection of point masses. For instance, in quantum mechanics molecules (consisting of the point masses: electrons and nuclei) are often seen as rigid bodies (see classification of molecules as rigid rotors)."

        Suhteellisuusteoriassa asia on, kuten täällä jo ainakin kerran ilmeni, mutta kvanttimekaniikassa toi asia nähdään toisin. Kuten tossa yllä kerrotaan.
        Ja siinä se ei o mitään utopiaa.
        Ja mitä sä niitä ääniaaltoja yähän sekotat edes? Ei kukaan o sitä alumiinitankoa minään ksylofonina ollut köyttämässäkään? :D
        Kysehän oli pelkästään sen liikuttamisesta. Ja väitäks sä nyt ihan tosissas, et jos mä liikutan jonossa olevia atomeja (helpotetaan vähän) täällä, ja se jono sattuu olemaan vaikkapa valovuoden pitunen, ettei ne atomit siellä toisessa päässäkin liiku!? :D

        Unohetaas sit koko alumiini, ja yksinkertaistetaan tätä vielä enemmän.
        Mä hommaan laserin, jolla mä ammun avaruuteen valovuoden pitusen pulssin, joka koostuu yhtenäisestä jonosta fotoneita. Väitätkö sä, et sen fotonijonon tämän puoleisen pään fotonit liikkuu eri nopeudella kuin sen kaukaisemman pään fotonit? Eli sen pulssin pituus siis sun mielestä lyhenee kaiken aikaa sitä mukaa kun se etenee kauemmas avaruudessa? Entäs mitäs tapahtuu niille kaikille fotoneille siinä välissä, että se pystyy lyhenemään?
        - Litistyykö ne?
        - Katoaako niistä osa? (fyssanmaikkas varmaan taputtais tästä päähän? Kts. Aineen/energian katoamattomuuden laki)
        - Vai mitä?

        Jotainhan siinä on tapahduttava, et sun teorias olis oikea!?


      • KuinKöyhänTalonPorsaat kirjoitti:

        Ei vaan nimenomaan jos asiaa tarkastellaan atomitasolla, niin jokanen molekyylikin on jäykkä kappale!

        "In the study of special relativity, a perfectly rigid body does not exist; and objects can only be assumed to be rigid if they are not moving near the speed of light. In quantum mechanics a rigid body is usually thought of as a collection of point masses. For instance, in quantum mechanics molecules (consisting of the point masses: electrons and nuclei) are often seen as rigid bodies (see classification of molecules as rigid rotors)."

        Suhteellisuusteoriassa asia on, kuten täällä jo ainakin kerran ilmeni, mutta kvanttimekaniikassa toi asia nähdään toisin. Kuten tossa yllä kerrotaan.
        Ja siinä se ei o mitään utopiaa.
        Ja mitä sä niitä ääniaaltoja yähän sekotat edes? Ei kukaan o sitä alumiinitankoa minään ksylofonina ollut köyttämässäkään? :D
        Kysehän oli pelkästään sen liikuttamisesta. Ja väitäks sä nyt ihan tosissas, et jos mä liikutan jonossa olevia atomeja (helpotetaan vähän) täällä, ja se jono sattuu olemaan vaikkapa valovuoden pitunen, ettei ne atomit siellä toisessa päässäkin liiku!? :D

        Unohetaas sit koko alumiini, ja yksinkertaistetaan tätä vielä enemmän.
        Mä hommaan laserin, jolla mä ammun avaruuteen valovuoden pitusen pulssin, joka koostuu yhtenäisestä jonosta fotoneita. Väitätkö sä, et sen fotonijonon tämän puoleisen pään fotonit liikkuu eri nopeudella kuin sen kaukaisemman pään fotonit? Eli sen pulssin pituus siis sun mielestä lyhenee kaiken aikaa sitä mukaa kun se etenee kauemmas avaruudessa? Entäs mitäs tapahtuu niille kaikille fotoneille siinä välissä, että se pystyy lyhenemään?
        - Litistyykö ne?
        - Katoaako niistä osa? (fyssanmaikkas varmaan taputtais tästä päähän? Kts. Aineen/energian katoamattomuuden laki)
        - Vai mitä?

        Jotainhan siinä on tapahduttava, et sun teorias olis oikea!?

        "Mä hommaan laserin, jolla mä ammun avaruuteen valovuoden pitusen pulssin, joka koostuu yhtenäisestä jonosta fotoneita. Väitätkö sä, et sen fotonijonon tämän puoleisen pään fotonit liikkuu eri nopeudella kuin sen kaukaisemman pään fotonit?"

        Onko fotonoijono kiinteä kappale?
        Jokainen fotoni liikkuu valonnopeudella.


        Monesko kerta?
        "Ja väitäks sä nyt ihan tosissas, et jos mä liikutan jonossa olevia atomeja (helpotetaan vähän) täällä, ja se jono sattuu olemaan vaikkapa valovuoden pitunen, ettei ne atomit siellä toisessa päässäkin liiku"

        Väitän, että jos liikutat atomeja täällä, niin jonon toisessa päässä atomit liikahtavat vasta yli vuoden päästä. Liikeen välittyminen tämän pään ensimmäisistä atomeista valovuoden pituisen palkin viimeisiin atomeihin kestää enemmän kuin vuoden. Näin väittää jokainen fysiikasta jotain ymmärtävä. ExB selitti sen jo ensimmäisessä kommentissaan, joskin vähän monimutkaisemmin.


      • KuinKöyhänTalonPorsaat kirjoitti:

        Ei vaan nimenomaan jos asiaa tarkastellaan atomitasolla, niin jokanen molekyylikin on jäykkä kappale!

        "In the study of special relativity, a perfectly rigid body does not exist; and objects can only be assumed to be rigid if they are not moving near the speed of light. In quantum mechanics a rigid body is usually thought of as a collection of point masses. For instance, in quantum mechanics molecules (consisting of the point masses: electrons and nuclei) are often seen as rigid bodies (see classification of molecules as rigid rotors)."

        Suhteellisuusteoriassa asia on, kuten täällä jo ainakin kerran ilmeni, mutta kvanttimekaniikassa toi asia nähdään toisin. Kuten tossa yllä kerrotaan.
        Ja siinä se ei o mitään utopiaa.
        Ja mitä sä niitä ääniaaltoja yähän sekotat edes? Ei kukaan o sitä alumiinitankoa minään ksylofonina ollut köyttämässäkään? :D
        Kysehän oli pelkästään sen liikuttamisesta. Ja väitäks sä nyt ihan tosissas, et jos mä liikutan jonossa olevia atomeja (helpotetaan vähän) täällä, ja se jono sattuu olemaan vaikkapa valovuoden pitunen, ettei ne atomit siellä toisessa päässäkin liiku!? :D

        Unohetaas sit koko alumiini, ja yksinkertaistetaan tätä vielä enemmän.
        Mä hommaan laserin, jolla mä ammun avaruuteen valovuoden pitusen pulssin, joka koostuu yhtenäisestä jonosta fotoneita. Väitätkö sä, et sen fotonijonon tämän puoleisen pään fotonit liikkuu eri nopeudella kuin sen kaukaisemman pään fotonit? Eli sen pulssin pituus siis sun mielestä lyhenee kaiken aikaa sitä mukaa kun se etenee kauemmas avaruudessa? Entäs mitäs tapahtuu niille kaikille fotoneille siinä välissä, että se pystyy lyhenemään?
        - Litistyykö ne?
        - Katoaako niistä osa? (fyssanmaikkas varmaan taputtais tästä päähän? Kts. Aineen/energian katoamattomuuden laki)
        - Vai mitä?

        Jotainhan siinä on tapahduttava, et sun teorias olis oikea!?

        "...are often seen as rigid bodies ..."
        "Ei vaan nimenomaan jos asiaa tarkastellaan atomitasolla, niin jokanen molekyylikin on jäykkä kappale!"

        Noinko suomensit tuon kohdan?
        Molekyyliä voidaan käsitellä kiinteänä kappaleena, vaikka kvanttifysiikassa kaikki alkeishiukkaset ovat paikkansa ja nopeutensa suhteen epämääräisiä. Kyse ei kuitenkaan ole molekyyleistä, vaan atomien muodostaman aineen rakenteen jäykkyydestä ja se ei ole jäykkä. Atomit elävät toistensa suhteen. Niiden väliset hylkimisvoimat ovat sähkömagneettisia voimia, eivätkä johdu siitä, että alkeishiukkaset tuuppisivat mekaanisesti toisiaan.


      • suoraviivainen_liike
        agnoskepo kirjoitti:

        "Mutta, jos siirtoa seurataan kaukaa sivusta SUURESSA MITTAKAAVASSA, niin kyllä se putki liikkuu periaatteessa yhtäaikaa molemmissa päissä."

        Juuri tästä on kysymys. Ei liiku ei, mikäli voimavaikutus kohdistuu vain putken toiseen päähän.
        Itse asiassa toista päätä kaukoputkella katsomalla liike näkyisi vasta yli kahden vuoden päästä tönäisystä. Putki todella kutistuu, koska "paineaalto" ei voi liikkua valonnopeudella.

        Hä ! Voiko joku olla noi tyhmä ??? Kyllä se nyt vaan tasan tarkkaa LIIKKUU, jos katsotaan puolesta välistä, mistä putken alkupisteeseen A, ja loppupisteeseen B on sama matka. Niin silloin se liikkuu samaan aikaan koko putki. Se putki ei witttu kutistu, eikä veny yhtään mihinkään olettaen se on jäykkää materiaalia.

        Jos sä liikutat täällä maan päälläkin 10 metrin pituista putkea niin väitätkö, että toinen pää liikkuu eri aikaan ?? Hohhohoijjaaa !!!

        ps. onko kiva peukutella omia kommentteja ?


      • suoraviivainen_liike kirjoitti:

        Hä ! Voiko joku olla noi tyhmä ??? Kyllä se nyt vaan tasan tarkkaa LIIKKUU, jos katsotaan puolesta välistä, mistä putken alkupisteeseen A, ja loppupisteeseen B on sama matka. Niin silloin se liikkuu samaan aikaan koko putki. Se putki ei witttu kutistu, eikä veny yhtään mihinkään olettaen se on jäykkää materiaalia.

        Jos sä liikutat täällä maan päälläkin 10 metrin pituista putkea niin väitätkö, että toinen pää liikkuu eri aikaan ?? Hohhohoijjaaa !!!

        ps. onko kiva peukutella omia kommentteja ?

        En koskaan peukuta omia kommenttejani.
        Kymmenen metrin putkella hitautta ei huomaa, mutta siinäkin liikkeen vaikutuksen siirtyminen toiseen päähän kestää. Valo kulkee 10 m matkan n. 0,033 mikrosekunnissa eikä mikään vuorovaikutus voi kulkea nopeammin. Muuten informaatiotakin voitaisiin siirtää ylivalonnopeudella ja se on fysiikan lakien mukaan mahdotonta.

        Jos oletetaan jäykkä materia, se tarkoittaa samalla oletusta materiasta, jonka olemassaolon fysiikka kieltää. Jäykkää materiaa ei atomitasolla ole.

        Hohhoijaa. Lopetan turhan jankuttamisen tähän.


      • lisää_jankutusta
        agnoskepo kirjoitti:

        En koskaan peukuta omia kommenttejani.
        Kymmenen metrin putkella hitautta ei huomaa, mutta siinäkin liikkeen vaikutuksen siirtyminen toiseen päähän kestää. Valo kulkee 10 m matkan n. 0,033 mikrosekunnissa eikä mikään vuorovaikutus voi kulkea nopeammin. Muuten informaatiotakin voitaisiin siirtää ylivalonnopeudella ja se on fysiikan lakien mukaan mahdotonta.

        Jos oletetaan jäykkä materia, se tarkoittaa samalla oletusta materiasta, jonka olemassaolon fysiikka kieltää. Jäykkää materiaa ei atomitasolla ole.

        Hohhoijaa. Lopetan turhan jankuttamisen tähän.

        Kannattais lukea tarkemmin fysiikkaa:
        https://fi.wikipedia.org/wiki/Jäykkä_kappale
        "..Monia kappaleita voidaan kuitenkin käytännössä pitää jäykkinä, jos niiden nopeus on paljon valon­nopeutta pienempi.."

        Eli ts. jos putkea liikuttaa hitaasti se voi säilyttää muotonsa, eikä siis veny, eikä vanu.
        ja silloin se liikkuu samaan aikaan joka kohdasta.

        Tässähän on ollu kokoajan kyse oletuksesta, että siirretään 1vv pituista jäykkää kappaletta, vaikka se olisi käytännössä mahdotonta.
        Se milloin siirto nähdään, riippuu mistä kohdin siirtoa katsotaan.. Työntäjä näkee heti että putki liikkuu, ja laittaa valomerkin putken toiseen pääähän. Toisessa päässä nähdään että putki liikkui, mutta valomerkki tulee perille vasta vuoden päästä, johtuen juuri tuosta valonnopeudesta , jota ei voi ylittää.


      • KepposAgno
        agnoskepo kirjoitti:

        "Mä hommaan laserin, jolla mä ammun avaruuteen valovuoden pitusen pulssin, joka koostuu yhtenäisestä jonosta fotoneita. Väitätkö sä, et sen fotonijonon tämän puoleisen pään fotonit liikkuu eri nopeudella kuin sen kaukaisemman pään fotonit?"

        Onko fotonoijono kiinteä kappale?
        Jokainen fotoni liikkuu valonnopeudella.


        Monesko kerta?
        "Ja väitäks sä nyt ihan tosissas, et jos mä liikutan jonossa olevia atomeja (helpotetaan vähän) täällä, ja se jono sattuu olemaan vaikkapa valovuoden pitunen, ettei ne atomit siellä toisessa päässäkin liiku"

        Väitän, että jos liikutat atomeja täällä, niin jonon toisessa päässä atomit liikahtavat vasta yli vuoden päästä. Liikeen välittyminen tämän pään ensimmäisistä atomeista valovuoden pituisen palkin viimeisiin atomeihin kestää enemmän kuin vuoden. Näin väittää jokainen fysiikasta jotain ymmärtävä. ExB selitti sen jo ensimmäisessä kommentissaan, joskin vähän monimutkaisemmin.

        Onko kiinteä kappale atomijonoista muodostunut kappale, jossa on noita jonoja useita päällekkäin ja vierekkäin? Joten mitä periaatteellista eroa muka on fotonijonolla ja kiinteällä kappaleella?

        Ja vaihetaan sen laserin tilalle sit elektronisuihku? Joko se sulle kelpais?
        Entäs jos muokataan teollisesta vesisuihkuleikkurista sellanen, et se ampuis sinne taivaalle vaan 1 molekyylin levysen valovuoden mittasen vesisuihkun, jokos sitten kelpais?

        Joten vastaas nyt! Liikkuisko noissa tapauksissa koko jono tismalleen samaa nopeutta, ja samaan aikaan, vai se toinen pää vuoden myöhässä!?


      • KepposAgno
        agnoskepo kirjoitti:

        "Mä hommaan laserin, jolla mä ammun avaruuteen valovuoden pitusen pulssin, joka koostuu yhtenäisestä jonosta fotoneita. Väitätkö sä, et sen fotonijonon tämän puoleisen pään fotonit liikkuu eri nopeudella kuin sen kaukaisemman pään fotonit?"

        Onko fotonoijono kiinteä kappale?
        Jokainen fotoni liikkuu valonnopeudella.


        Monesko kerta?
        "Ja väitäks sä nyt ihan tosissas, et jos mä liikutan jonossa olevia atomeja (helpotetaan vähän) täällä, ja se jono sattuu olemaan vaikkapa valovuoden pitunen, ettei ne atomit siellä toisessa päässäkin liiku"

        Väitän, että jos liikutat atomeja täällä, niin jonon toisessa päässä atomit liikahtavat vasta yli vuoden päästä. Liikeen välittyminen tämän pään ensimmäisistä atomeista valovuoden pituisen palkin viimeisiin atomeihin kestää enemmän kuin vuoden. Näin väittää jokainen fysiikasta jotain ymmärtävä. ExB selitti sen jo ensimmäisessä kommentissaan, joskin vähän monimutkaisemmin.

        ... ja sä et sit näköjään ymmärtäny hölkäsen pöläystä et mistä ExB puhui? :)
        Se puhu siitä, et miten tässä, ja nyt, sitä suntankoas työntävä havaitsija kokee tapahtuvan SIINÄ PAIKALLAAN OLLESSAAN sille tangolle tapahtuvan siellä toisessa oäässä, jos se kykenis sinne näkemään!

        Mutta siitähän tässä ei o ollu kyse koko aikana! Se tanko, ihan kokonaisuudessaan ja koko matkalta, on olemassa ihan "tässä ja nyt", tismalleen samassa ajassa ja avaruudessa, eikä kuten joku avaruudessa lentävä hiukkanen, vaikkapa fotoni, jonka käyttäytymiseen ja havainnointiin ExB kirjotus ennemminkin viittasi. Vai väitätkö sä et sen tangon toinen pää on eri ajassakin kun toinen? Siis SEN TANGON KANNALTA! Täs ei o nyt kukaan (tai ainakaan minä) puhunutkaan et mitä se jossain toisessa päässä olevanHAVAITSIJA on näkevinään, vaan että mitä sille itse TANGOLLE tapahuu! Kokonaisuutena! Ja on se nyt vittu soikoon ihme fysiikkaa, jos mä tönäsen sitä tankoa, ettei se liiku perkele kokonaisuutena! Sillä ei o paskan väliä vaikka sitä ei olis sit KUKAAN edes havainnoimassa. Mut sä oot kyllä ilmiselvästi menettäny järkes, jos sä kehtaat edes väittää et sen kiinteän kappaleen molemmat päät ei o samassa ajassa edes! Ja älä nyt sit ala taas sönköttämään mistään havatsijoista, nehän me jo poistettiin!


      • KepposAgno
        agnoskepo kirjoitti:

        "...are often seen as rigid bodies ..."
        "Ei vaan nimenomaan jos asiaa tarkastellaan atomitasolla, niin jokanen molekyylikin on jäykkä kappale!"

        Noinko suomensit tuon kohdan?
        Molekyyliä voidaan käsitellä kiinteänä kappaleena, vaikka kvanttifysiikassa kaikki alkeishiukkaset ovat paikkansa ja nopeutensa suhteen epämääräisiä. Kyse ei kuitenkaan ole molekyyleistä, vaan atomien muodostaman aineen rakenteen jäykkyydestä ja se ei ole jäykkä. Atomit elävät toistensa suhteen. Niiden väliset hylkimisvoimat ovat sähkömagneettisia voimia, eivätkä johdu siitä, että alkeishiukkaset tuuppisivat mekaanisesti toisiaan.

        En, vaan tän koko paskan:
        "In quantum mechanics a rigid body is usually thought of as a collection of point masses. For instance, in quantum mechanics molecules (consisting of the point masses: electrons and nuclei) are often seen as rigid bodies."

        Ja kyllä mun mielestäni ainakin molekyyleistä puhuttaessa kyse on molekyyleistä!? :) Eikö sinusta? Ja eikö se tossa yllä ihan selvästi sanota?
        (jonka muuten *köh* "unohdit" näppärästi jättää kopioimatta omaan lainaukseesi! :) ) Ja ja jos tossa lainauksessa sanotaan, että sen voi mieltää jäykäksi massapisteiden kokoelmaksi, niin: Hell! I buy it! Mut sä et, miks?
        Sun kannattaa sit varmaan käydä korjaamassa toi enkku-wikin artikkeli, koska siinähän on sit sun mielestäs sit selkeä virhe?


      • teoriassakin.mahdotonta
        lisää_jankutusta kirjoitti:

        Kannattais lukea tarkemmin fysiikkaa:
        https://fi.wikipedia.org/wiki/Jäykkä_kappale
        "..Monia kappaleita voidaan kuitenkin käytännössä pitää jäykkinä, jos niiden nopeus on paljon valon­nopeutta pienempi.."

        Eli ts. jos putkea liikuttaa hitaasti se voi säilyttää muotonsa, eikä siis veny, eikä vanu.
        ja silloin se liikkuu samaan aikaan joka kohdasta.

        Tässähän on ollu kokoajan kyse oletuksesta, että siirretään 1vv pituista jäykkää kappaletta, vaikka se olisi käytännössä mahdotonta.
        Se milloin siirto nähdään, riippuu mistä kohdin siirtoa katsotaan.. Työntäjä näkee heti että putki liikkuu, ja laittaa valomerkin putken toiseen pääähän. Toisessa päässä nähdään että putki liikkui, mutta valomerkki tulee perille vasta vuoden päästä, johtuen juuri tuosta valonnopeudesta , jota ei voi ylittää.

        "Tässähän on ollu kokoajan kyse oletuksesta, että siirretään 1vv pituista jäykkää kappaletta, vaikka se olisi käytännössä mahdotonta. "

        Se on paitsi käytännössä myös teoriassa mahdotonta. Kysyt edellenkin mitä tapahtuisi fysiikan lakien mukaan jos fysiikan lait eivät olisi voimassa.


      • lisää_jankutusta
        teoriassakin.mahdotonta kirjoitti:

        "Tässähän on ollu kokoajan kyse oletuksesta, että siirretään 1vv pituista jäykkää kappaletta, vaikka se olisi käytännössä mahdotonta. "

        Se on paitsi käytännössä myös teoriassa mahdotonta. Kysyt edellenkin mitä tapahtuisi fysiikan lakien mukaan jos fysiikan lait eivät olisi voimassa.

        Niinpä niinn, Putkeahan käytetään vain havaintokappaleena.

        Yhtä hyvin se kappale voisi olla vaikka halkaisijaltaan 1 vv. pituinen kiinteä pallo, esim. maan kaltainen kiviplaneetta, joka kiertää ympyrän muotoista rataa jonkin tähden ympärillä. Sinä siis väitä että tuon planeetan toinen puoli ei tule samassa suhteessa perässä, vaan planeetta kutistuu, tai venyy ???

        Kyllä täällä näkyy olevan jollain fysiikan lait todella hakusessa !


      • hjhgjhghjgjhgj
        lisää_jankutusta kirjoitti:

        Niinpä niinn, Putkeahan käytetään vain havaintokappaleena.

        Yhtä hyvin se kappale voisi olla vaikka halkaisijaltaan 1 vv. pituinen kiinteä pallo, esim. maan kaltainen kiviplaneetta, joka kiertää ympyrän muotoista rataa jonkin tähden ympärillä. Sinä siis väitä että tuon planeetan toinen puoli ei tule samassa suhteessa perässä, vaan planeetta kutistuu, tai venyy ???

        Kyllä täällä näkyy olevan jollain fysiikan lait todella hakusessa !

        No eihän se tule.
        "Kyllä täällä näkyy olevan jollain fysiikan lait todella hakusessa ! "
        Niin näyttää olevan. Newtonin mekaniikan mukaan liike vaikuttaisi heti koko kappaleen mitalla. Tämän Einstein kiisti todistaessaan, ettei mikään informaatio voi liikkua ylivalonnopeudella. ExB sanoi asian ensin juuri nin kuin se on ja sen jälkeen Agnoskepo väänsi useita keriä rautalankaa, eikä hömelö vieläkään tajua.

        Väitteesi on sotkettu suohon. Opettele hyvä hömelö, mikä ero on newtonilaisella fysiikalla ja modernilla fysiikalla.


      • Opettele.lukemaan
        hjhgjhghjgjhgj kirjoitti:

        No eihän se tule.
        "Kyllä täällä näkyy olevan jollain fysiikan lait todella hakusessa ! "
        Niin näyttää olevan. Newtonin mekaniikan mukaan liike vaikuttaisi heti koko kappaleen mitalla. Tämän Einstein kiisti todistaessaan, ettei mikään informaatio voi liikkua ylivalonnopeudella. ExB sanoi asian ensin juuri nin kuin se on ja sen jälkeen Agnoskepo väänsi useita keriä rautalankaa, eikä hömelö vieläkään tajua.

        Väitteesi on sotkettu suohon. Opettele hyvä hömelö, mikä ero on newtonilaisella fysiikalla ja modernilla fysiikalla.

        Opettele sinä hömelö ensin lukemaan, ja tule vasta sen jälkeen kommentoimaan tänne. !


      • Teoriasion-TäyttäSoopaa
        hjhgjhghjgjhgj kirjoitti:

        No eihän se tule.
        "Kyllä täällä näkyy olevan jollain fysiikan lait todella hakusessa ! "
        Niin näyttää olevan. Newtonin mekaniikan mukaan liike vaikuttaisi heti koko kappaleen mitalla. Tämän Einstein kiisti todistaessaan, ettei mikään informaatio voi liikkua ylivalonnopeudella. ExB sanoi asian ensin juuri nin kuin se on ja sen jälkeen Agnoskepo väänsi useita keriä rautalankaa, eikä hömelö vieläkään tajua.

        Väitteesi on sotkettu suohon. Opettele hyvä hömelö, mikä ero on newtonilaisella fysiikalla ja modernilla fysiikalla.

        Sinun teoriasi poltettu, ja hävitetty ikuisesti maan päältä. Oma teoriani, joka perustuu A.Einsteinin oppeihin, on ja pysyy kirjoissa ja kansissa niin kauan, kuin tämä maapallo on olemassa !!


      • Olehyvä.ja.laske
        hjhgjhghjgjhgj kirjoitti:

        No eihän se tule.
        "Kyllä täällä näkyy olevan jollain fysiikan lait todella hakusessa ! "
        Niin näyttää olevan. Newtonin mekaniikan mukaan liike vaikuttaisi heti koko kappaleen mitalla. Tämän Einstein kiisti todistaessaan, ettei mikään informaatio voi liikkua ylivalonnopeudella. ExB sanoi asian ensin juuri nin kuin se on ja sen jälkeen Agnoskepo väänsi useita keriä rautalankaa, eikä hömelö vieläkään tajua.

        Väitteesi on sotkettu suohon. Opettele hyvä hömelö, mikä ero on newtonilaisella fysiikalla ja modernilla fysiikalla.

        Siinä "hjhgjhghjgjhgj" hörhölle faktaa:

        https://fi.wikipedia.org/wiki/Erityinen_suhteellisuusteoria
        ".... Suhteellisuusteorian mukaan kappaleen pituus pienenee, kun se liikkuu. Pieneneminen on havaittavissa vain äärimmäisen suurilla nopeuksilla, siis lähellä valonnopeutta...."

        Eli, jos putkea liikuttaa esim. 10km/h nopeudella, kappale ei lyhene juuri hyttysen passkaa enempää. Kappaleen lyhenemisen voi laskea "Pituuskontraktio kaavalla" , joka näkyy tuossa ylläolevassa linkissä.


      • FyssaaLukenut
        Olehyvä.ja.laske kirjoitti:

        Siinä "hjhgjhghjgjhgj" hörhölle faktaa:

        https://fi.wikipedia.org/wiki/Erityinen_suhteellisuusteoria
        ".... Suhteellisuusteorian mukaan kappaleen pituus pienenee, kun se liikkuu. Pieneneminen on havaittavissa vain äärimmäisen suurilla nopeuksilla, siis lähellä valonnopeutta...."

        Eli, jos putkea liikuttaa esim. 10km/h nopeudella, kappale ei lyhene juuri hyttysen passkaa enempää. Kappaleen lyhenemisen voi laskea "Pituuskontraktio kaavalla" , joka näkyy tuossa ylläolevassa linkissä.

        Sinulle on monen kirjoittajan toimesta kerrottu, ettei pitkän kappaleen toinen pää lähde liikkeelle samanaikaisesti kuin toisesta päästä sitä työnnetään. Se on faktaa, vaikka näköjään sinulle aivan liian vaikeaa ymmärtää.
        Kysehän ei ole Lorenzin muunnoksesta pituuden suhteen, mitä yllä yrität selittää, vaan voimavaikutuksen etenemisnopeudesta. Ne ovat kokonaan eri asioita.

        Syy on siis seuraava: "Valonnopeus tyhjiössä on siis suurin mahdollinen signaalinopeus, kun oletetaan kausaliteetin invarianssi"
        (Maalampi - Perko LYHYT MODERNIN FYSIKAN JOHDATUS)
        Jos vaikka runsaan valosekunnin pituisen sauvan toinen Kuussa oleva pää liikahtaisi samaan aikaan kuin Maassa olevaa päätä tönäistään, tieto työnnöstä kulkisi Kuuhun yli valonnopeudella. Se vaatisi erityisen suhteellisuusteorian kumoutumista. Jäykät kappaleet välittömine voimavaikutuksineen kuuluvat newtonilaiseen fysiikkaan ja ovat vain likiarvoja.


      • KAPPALE_EI_LYHENE
        FyssaaLukenut kirjoitti:

        Sinulle on monen kirjoittajan toimesta kerrottu, ettei pitkän kappaleen toinen pää lähde liikkeelle samanaikaisesti kuin toisesta päästä sitä työnnetään. Se on faktaa, vaikka näköjään sinulle aivan liian vaikeaa ymmärtää.
        Kysehän ei ole Lorenzin muunnoksesta pituuden suhteen, mitä yllä yrität selittää, vaan voimavaikutuksen etenemisnopeudesta. Ne ovat kokonaan eri asioita.

        Syy on siis seuraava: "Valonnopeus tyhjiössä on siis suurin mahdollinen signaalinopeus, kun oletetaan kausaliteetin invarianssi"
        (Maalampi - Perko LYHYT MODERNIN FYSIKAN JOHDATUS)
        Jos vaikka runsaan valosekunnin pituisen sauvan toinen Kuussa oleva pää liikahtaisi samaan aikaan kuin Maassa olevaa päätä tönäistään, tieto työnnöstä kulkisi Kuuhun yli valonnopeudella. Se vaatisi erityisen suhteellisuusteorian kumoutumista. Jäykät kappaleet välittömine voimavaikutuksineen kuuluvat newtonilaiseen fysiikkaan ja ovat vain likiarvoja.

        Älä jankuta tuosta ylivalonnopeudesta. Sen olen jo kuullu ihan tarpeeksi moneen kertaan !!!!

        Pitkän kappaleen liikkuminen ei ole sama asia, kuin tiedon kulku?? Eikä missään teorioissa lausuta kappaleille maksimipituuksia. Tarpeeksi jäykkä Kappale käyttäytyy samanalailla oli sillä pituutta 1 km, tai 1 vv. Kaukana avaruudessa, eli tyhjiössä kappaleen saa myös liikkeelle huomattavsti pienemmällä voimalla, kuin planeettojen painovoimakentässä, joten kappaleeseen kohdistuu myös huomattavasti pienempi paine.

        Sinä siis kumoat em. mainitun, erityisen suhtellisuusteorian lisäksi myös A.Einsteinin yleisen suhtellisuusteorian, joka sanoo:

        "...Yleisen suhteellisuuden periaatteen mukaan fysiikan lait ovat samat kaikille havaitsijoille riippumatta siitä, ovatko ne kiihtyvässä liikkeessä, tasaisessa liikkeessä vai paikallaan. Yleisen kovarianssin mukaan fysiikan lait ovat samat kaikissa koordinaatistoissa. Yleinen suhteellisuusteoria vaatii myös, että liikkeen ja geodeettisen liikkeen välillä vallitsee yhtäsuuruus..." (A. Einstein 1907-1915)


      • FyssaaLukenut kirjoitti:

        Sinulle on monen kirjoittajan toimesta kerrottu, ettei pitkän kappaleen toinen pää lähde liikkeelle samanaikaisesti kuin toisesta päästä sitä työnnetään. Se on faktaa, vaikka näköjään sinulle aivan liian vaikeaa ymmärtää.
        Kysehän ei ole Lorenzin muunnoksesta pituuden suhteen, mitä yllä yrität selittää, vaan voimavaikutuksen etenemisnopeudesta. Ne ovat kokonaan eri asioita.

        Syy on siis seuraava: "Valonnopeus tyhjiössä on siis suurin mahdollinen signaalinopeus, kun oletetaan kausaliteetin invarianssi"
        (Maalampi - Perko LYHYT MODERNIN FYSIKAN JOHDATUS)
        Jos vaikka runsaan valosekunnin pituisen sauvan toinen Kuussa oleva pää liikahtaisi samaan aikaan kuin Maassa olevaa päätä tönäistään, tieto työnnöstä kulkisi Kuuhun yli valonnopeudella. Se vaatisi erityisen suhteellisuusteorian kumoutumista. Jäykät kappaleet välittömine voimavaikutuksineen kuuluvat newtonilaiseen fysiikkaan ja ovat vain likiarvoja.

        Pitkä metallitanko käyttäytyy kuten jousi kun sitä puristetaan:

        http://www.solitaryroad.com/c1030.html

        Kuvassa 2. näkyy, miten jousen pään liikuttaminen vaikuttaa jouseen. Siinä lähtee liikkeelle muuta jousta tiheämpi alue nopeudella, joka metallissa olisi äänen nopeus kompressioaallolle.

        Tässä youtube - video siitä, miten kompressioaalto etenee jousessa:

        https://www.youtube.com/watch?v=y7qS6SyyrFU

        Kun jousen päätä heilautetaan niin sen toisessa päässä ei heti tapahtu mitään. Tieto heilahduksesta tulee perille vasta kun aalto etenee perille asti. Sama koskee pitkää tankoa, jonka toista päätä liikautetaan. Liikautus lähtee aaltona etenemään pitkin tankoa.

        Kaikki atomeista koostuva aine pysyy koossa sähkömagneettisen vuorovaikutuksen ansiosta. Kvanttimekaniikka tekee tästä paljon monimutkaisempaa mutta ei muuta vuorovakutuksen tyyppiä.

        Kun metallitangon pään uloimpia atomeita liikutetaan niin ne alkavat sähkökenttänsä avulla vetämään ja työntämään seuraavan kerroksen atomeita. Nuo atomit jäävät hieman jälkeen liikkeestä oman hitautensa vuoksi mutta kun ne hiljakseen saavat vauhtia niin puolestaan liikuttavat seuraavaa atomikerrosta sähkökentän avulla jne.

        Mekaaninen aalto enetee aineeseen enintään sähkömagneettisen vuorovaikutuksen nopeudella (valon nopeus tyhjiössä) mutta jokaisen atomin kohdalla se hidastuu kun atomille pitää saada vauhtia ennenkuin se puolestaan pääsee siirtymään pois tasapainoasemastaan ja vaikuttamaan seuraaviin atomeihin sähkökentällään. Tämä rajoittaa äänen etenemisnopeuden aineessa paljon valon tyhjiönopeutta pienemmäksi.

        Fysiikan lakeja noudattavasta aineesta valmistetussa tangossa mekaaninen liike ei siis voi edetä edes valon tyhjiönopeudella vaan selvästi sitä hitaammin.


      • ExB kirjoitti:

        Pitkä metallitanko käyttäytyy kuten jousi kun sitä puristetaan:

        http://www.solitaryroad.com/c1030.html

        Kuvassa 2. näkyy, miten jousen pään liikuttaminen vaikuttaa jouseen. Siinä lähtee liikkeelle muuta jousta tiheämpi alue nopeudella, joka metallissa olisi äänen nopeus kompressioaallolle.

        Tässä youtube - video siitä, miten kompressioaalto etenee jousessa:

        https://www.youtube.com/watch?v=y7qS6SyyrFU

        Kun jousen päätä heilautetaan niin sen toisessa päässä ei heti tapahtu mitään. Tieto heilahduksesta tulee perille vasta kun aalto etenee perille asti. Sama koskee pitkää tankoa, jonka toista päätä liikautetaan. Liikautus lähtee aaltona etenemään pitkin tankoa.

        Kaikki atomeista koostuva aine pysyy koossa sähkömagneettisen vuorovaikutuksen ansiosta. Kvanttimekaniikka tekee tästä paljon monimutkaisempaa mutta ei muuta vuorovakutuksen tyyppiä.

        Kun metallitangon pään uloimpia atomeita liikutetaan niin ne alkavat sähkökenttänsä avulla vetämään ja työntämään seuraavan kerroksen atomeita. Nuo atomit jäävät hieman jälkeen liikkeestä oman hitautensa vuoksi mutta kun ne hiljakseen saavat vauhtia niin puolestaan liikuttavat seuraavaa atomikerrosta sähkökentän avulla jne.

        Mekaaninen aalto enetee aineeseen enintään sähkömagneettisen vuorovaikutuksen nopeudella (valon nopeus tyhjiössä) mutta jokaisen atomin kohdalla se hidastuu kun atomille pitää saada vauhtia ennenkuin se puolestaan pääsee siirtymään pois tasapainoasemastaan ja vaikuttamaan seuraaviin atomeihin sähkökentällään. Tämä rajoittaa äänen etenemisnopeuden aineessa paljon valon tyhjiönopeutta pienemmäksi.

        Fysiikan lakeja noudattavasta aineesta valmistetussa tangossa mekaaninen liike ei siis voi edetä edes valon tyhjiönopeudella vaan selvästi sitä hitaammin.

        Jäykät kappaleet suhteellisuusteoriassa tulivat esille jo varsin varhain. Ehrenfestin paradoksi on yksi esimerkki siitä, mitä kaikkea kivaa päänvaivaa ne aiheuttavat. Born-jäykkyys jne. Näitä siis mietittiin jo 1900 - luvun alkuvuosina ennen 1. maailmansotaa.

        https://en.wikipedia.org/wiki/Ehrenfest_paradox

        Luennoitsijat tykkäävät kiusata opiskelijoitaan tälläisillä laskutehtävillä:

        "Oletetaan että hyrrä pyörii 100 kierrosta sekunnissa. Mikä pitää hyrrän säteen olla, jotta sen kehän pituus on mahdollisimman suuri?"

        Suhteellisuusteoriaan ja muuhunkin fysiikkaan liittyviä kysymyksiä on käsitelty Fysiikan Usein Kysytyt Kysymykset - sivustolla

        http://www.math.ucr.edu/home/baez/physics/


      • FyssaaLukenut
        KAPPALE_EI_LYHENE kirjoitti:

        Älä jankuta tuosta ylivalonnopeudesta. Sen olen jo kuullu ihan tarpeeksi moneen kertaan !!!!

        Pitkän kappaleen liikkuminen ei ole sama asia, kuin tiedon kulku?? Eikä missään teorioissa lausuta kappaleille maksimipituuksia. Tarpeeksi jäykkä Kappale käyttäytyy samanalailla oli sillä pituutta 1 km, tai 1 vv. Kaukana avaruudessa, eli tyhjiössä kappaleen saa myös liikkeelle huomattavsti pienemmällä voimalla, kuin planeettojen painovoimakentässä, joten kappaleeseen kohdistuu myös huomattavasti pienempi paine.

        Sinä siis kumoat em. mainitun, erityisen suhtellisuusteorian lisäksi myös A.Einsteinin yleisen suhtellisuusteorian, joka sanoo:

        "...Yleisen suhteellisuuden periaatteen mukaan fysiikan lait ovat samat kaikille havaitsijoille riippumatta siitä, ovatko ne kiihtyvässä liikkeessä, tasaisessa liikkeessä vai paikallaan. Yleisen kovarianssin mukaan fysiikan lait ovat samat kaikissa koordinaatistoissa. Yleinen suhteellisuusteoria vaatii myös, että liikkeen ja geodeettisen liikkeen välillä vallitsee yhtäsuuruus..." (A. Einstein 1907-1915)

        " Kaukana avaruudessa, eli tyhjiössä kappaleen saa myös liikkeelle huomattavsti pienemmällä voimalla, kuin planeettojen painovoimakentässä,"

        Newtonin toisen lain eli dynamiikan paruslain mukaan a = F : m. Millä kohtaa näet kaavassa planeetan painovoimakentän? Massan ominaisuus on hitaus.

        Kuten ExB:kin tuossa selitti, "Mekaaninen aalto enetee aineeseen enintään sähkömagneettisen vuorovaikutuksen nopeudella (valon nopeus tyhjiössä) mutta jokaisen atomin kohdalla se hidastuu kun atomille pitää saada vauhtia ennenkuin se puolestaan pääsee siirtymään pois tasapainoasemastaan ja vaikuttamaan seuraaviin atomeihin sähkökentällään. Tämä rajoittaa äänen etenemisnopeuden aineessa paljon valon tyhjiönopeutta pienemmäksi."

        1 vv:n pituisen tangon toinen pää lähtee liikkeelle vasta useiden vuosien kuluttua toisen pään työntämisestä, kuten sinulle on monen kirjoittajan toimesta selitetty. Nopeimmillaan se voisi lähteä liikkeelle vuoden kuluttua, mutta mekaaninen vaikutus on paljon valonnopeutta hitaampi, joten pitkäksi ehtii parta toisessa päässä kasvaa, ennen kuin se pää inahtaakaan. Jos se siirtyisi heti, siirtyisi myös informaatio siitä, että tankoa on työnnetty. Se rikkoisi aivan kiistatta erityisen suhteellisuusteorian periaatteita vastaan.

        Asia taitaa olla sinulle vähän liian vaikea. Koulutko vielä kesken?


      • käy.ensin.peruskoulu
        FyssaaLukenut kirjoitti:

        " Kaukana avaruudessa, eli tyhjiössä kappaleen saa myös liikkeelle huomattavsti pienemmällä voimalla, kuin planeettojen painovoimakentässä,"

        Newtonin toisen lain eli dynamiikan paruslain mukaan a = F : m. Millä kohtaa näet kaavassa planeetan painovoimakentän? Massan ominaisuus on hitaus.

        Kuten ExB:kin tuossa selitti, "Mekaaninen aalto enetee aineeseen enintään sähkömagneettisen vuorovaikutuksen nopeudella (valon nopeus tyhjiössä) mutta jokaisen atomin kohdalla se hidastuu kun atomille pitää saada vauhtia ennenkuin se puolestaan pääsee siirtymään pois tasapainoasemastaan ja vaikuttamaan seuraaviin atomeihin sähkökentällään. Tämä rajoittaa äänen etenemisnopeuden aineessa paljon valon tyhjiönopeutta pienemmäksi."

        1 vv:n pituisen tangon toinen pää lähtee liikkeelle vasta useiden vuosien kuluttua toisen pään työntämisestä, kuten sinulle on monen kirjoittajan toimesta selitetty. Nopeimmillaan se voisi lähteä liikkeelle vuoden kuluttua, mutta mekaaninen vaikutus on paljon valonnopeutta hitaampi, joten pitkäksi ehtii parta toisessa päässä kasvaa, ennen kuin se pää inahtaakaan. Jos se siirtyisi heti, siirtyisi myös informaatio siitä, että tankoa on työnnetty. Se rikkoisi aivan kiistatta erityisen suhteellisuusteorian periaatteita vastaan.

        Asia taitaa olla sinulle vähän liian vaikea. Koulutko vielä kesken?

        Just joo. Sinulla on kirjoituksestasi päätellen vielä peruskoulukin kesken.

        ÄLÄKÄ VAAN HAKEUDU NOILLA TIEDOILLA MIHINKÄÄN TEKNIIKKAAN JA TIETEISIIN LIITTYVÄÄN KOULUTUKSEEN. SINUT NAURETAAN PIHALLE SIELTÄ VÄLITTÖMÄSTI !!!


      • Hohhoijakkaa
        FyssaaLukenut kirjoitti:

        Sinulle on monen kirjoittajan toimesta kerrottu, ettei pitkän kappaleen toinen pää lähde liikkeelle samanaikaisesti kuin toisesta päästä sitä työnnetään. Se on faktaa, vaikka näköjään sinulle aivan liian vaikeaa ymmärtää.
        Kysehän ei ole Lorenzin muunnoksesta pituuden suhteen, mitä yllä yrität selittää, vaan voimavaikutuksen etenemisnopeudesta. Ne ovat kokonaan eri asioita.

        Syy on siis seuraava: "Valonnopeus tyhjiössä on siis suurin mahdollinen signaalinopeus, kun oletetaan kausaliteetin invarianssi"
        (Maalampi - Perko LYHYT MODERNIN FYSIKAN JOHDATUS)
        Jos vaikka runsaan valosekunnin pituisen sauvan toinen Kuussa oleva pää liikahtaisi samaan aikaan kuin Maassa olevaa päätä tönäistään, tieto työnnöstä kulkisi Kuuhun yli valonnopeudella. Se vaatisi erityisen suhteellisuusteorian kumoutumista. Jäykät kappaleet välittömine voimavaikutuksineen kuuluvat newtonilaiseen fysiikkaan ja ovat vain likiarvoja.

        Niin.
        Mutta kun te perkeleet ette osaa LUKEA! :/

        "Erityisen suhteellisuusteorian matemaattinen perusta ovat Lorentzin muunnokset, jotka voi johtaa teorian postulaateista. Nämä muunnokset _kuvaavat_avaruutta_kahden _inertiaalihavaitsijan _näkökulmasta_, ts. havaitsijat liikkuvat toisiinsa nähden mutta eivät koe kiihtyvyyttä. Suhteellisuusperiaate koskee siis kaikkia inertiaalihavaitsijoita."

        Eli: nakkaa ne sun havainnoijat helvettiin koko yhtälöstä, niin se tikku liikkuu vallan mainiosti sen tikun näkökulmasta siinä avaruudessa ihan kuten se metrin sauvakin!

        Joko vihdoin?


      • Hohhotihoijakkaa
        Hohhoijakkaa kirjoitti:

        Niin.
        Mutta kun te perkeleet ette osaa LUKEA! :/

        "Erityisen suhteellisuusteorian matemaattinen perusta ovat Lorentzin muunnokset, jotka voi johtaa teorian postulaateista. Nämä muunnokset _kuvaavat_avaruutta_kahden _inertiaalihavaitsijan _näkökulmasta_, ts. havaitsijat liikkuvat toisiinsa nähden mutta eivät koe kiihtyvyyttä. Suhteellisuusperiaate koskee siis kaikkia inertiaalihavaitsijoita."

        Eli: nakkaa ne sun havainnoijat helvettiin koko yhtälöstä, niin se tikku liikkuu vallan mainiosti sen tikun näkökulmasta siinä avaruudessa ihan kuten se metrin sauvakin!

        Joko vihdoin?

        Tai no!
        Otetaas peliin se yks kohtalaisen riski havainnoija, jolla on poweriä työntää sitä megatonnien painosta tankoa!
        Sovitaan et sillä on valovuoden levyinen vartalo, ja siinä valovuoden levyinen silmä! Jolla se sit mulkkaa sitä saatanan tankoa sen koko valovuoden matkalta parinkymmenen sentin etäisyydeltä.

        Missä vaiheessa fotonit siitä tangosta alkaa tulemaan hitaammin sen silmään kun se tanko liikkuu, kun se työntää sitä? Vai huomaako se vaan jossain kohtaa et se tanko ei enää liikukaan? Koska eikös juuri näin tapahdu teidän teorian mukaan?


      • FyssaaLukenut
        käy.ensin.peruskoulu kirjoitti:

        Just joo. Sinulla on kirjoituksestasi päätellen vielä peruskoulukin kesken.

        ÄLÄKÄ VAAN HAKEUDU NOILLA TIEDOILLA MIHINKÄÄN TEKNIIKKAAN JA TIETEISIIN LIITTYVÄÄN KOULUTUKSEEN. SINUT NAURETAAN PIHALLE SIELTÄ VÄLITTÖMÄSTI !!!

        "ÄLÄKÄ VAAN HAKEUDU NOILLA TIEDOILLA MIHINKÄÄN TEKNIIKKAAN JA TIETEISIIN LIITTYVÄÄN KOULUTUKSEEN. SINUT NAURETAAN PIHALLE SIELTÄ VÄLITTÖMÄSTI !!! "

        Olen ollut tekniikan opettaja. Nyt jo eläkkeellä.


      • NäinSeVaanMenee
        ExB kirjoitti:

        Pitkä metallitanko käyttäytyy kuten jousi kun sitä puristetaan:

        http://www.solitaryroad.com/c1030.html

        Kuvassa 2. näkyy, miten jousen pään liikuttaminen vaikuttaa jouseen. Siinä lähtee liikkeelle muuta jousta tiheämpi alue nopeudella, joka metallissa olisi äänen nopeus kompressioaallolle.

        Tässä youtube - video siitä, miten kompressioaalto etenee jousessa:

        https://www.youtube.com/watch?v=y7qS6SyyrFU

        Kun jousen päätä heilautetaan niin sen toisessa päässä ei heti tapahtu mitään. Tieto heilahduksesta tulee perille vasta kun aalto etenee perille asti. Sama koskee pitkää tankoa, jonka toista päätä liikautetaan. Liikautus lähtee aaltona etenemään pitkin tankoa.

        Kaikki atomeista koostuva aine pysyy koossa sähkömagneettisen vuorovaikutuksen ansiosta. Kvanttimekaniikka tekee tästä paljon monimutkaisempaa mutta ei muuta vuorovakutuksen tyyppiä.

        Kun metallitangon pään uloimpia atomeita liikutetaan niin ne alkavat sähkökenttänsä avulla vetämään ja työntämään seuraavan kerroksen atomeita. Nuo atomit jäävät hieman jälkeen liikkeestä oman hitautensa vuoksi mutta kun ne hiljakseen saavat vauhtia niin puolestaan liikuttavat seuraavaa atomikerrosta sähkökentän avulla jne.

        Mekaaninen aalto enetee aineeseen enintään sähkömagneettisen vuorovaikutuksen nopeudella (valon nopeus tyhjiössä) mutta jokaisen atomin kohdalla se hidastuu kun atomille pitää saada vauhtia ennenkuin se puolestaan pääsee siirtymään pois tasapainoasemastaan ja vaikuttamaan seuraaviin atomeihin sähkökentällään. Tämä rajoittaa äänen etenemisnopeuden aineessa paljon valon tyhjiönopeutta pienemmäksi.

        Fysiikan lakeja noudattavasta aineesta valmistetussa tangossa mekaaninen liike ei siis voi edetä edes valon tyhjiönopeudella vaan selvästi sitä hitaammin.

        Juu ei! :/

        Kun justiinhan mä sanoin et miten kvanttimekaniikka näkee ton asian!
        "For instance, in quantum mechanics molecules (consisting of the point masses: electrons and nuclei) are often seen as rigid bodies."
        Eli molekyylit on massapisteiden kokoelma, joka on jäykkä!
        Jos sä siirrät siinä tolpan päässä olevien molekyylien massapisteitä (ie. juurikin niitä molekyylejä itseään) niin sitä seuraavat massapisteet siirtää paikkaansa eteenpäin (eli koko jono liikkuu kun köyhän talon porsaat).

        Ehkä tää selittäis asian paremmin?
        "The approximation is actually rigorous in the case of the gravity of a solid body having mass distributed with spherical symmetry, since an analytic derivation shows that the body acts exactly as a point mass at the body's center containing the body's entire mass."


      • FysiikkaaLukenut
        Hohhotihoijakkaa kirjoitti:

        Tai no!
        Otetaas peliin se yks kohtalaisen riski havainnoija, jolla on poweriä työntää sitä megatonnien painosta tankoa!
        Sovitaan et sillä on valovuoden levyinen vartalo, ja siinä valovuoden levyinen silmä! Jolla se sit mulkkaa sitä saatanan tankoa sen koko valovuoden matkalta parinkymmenen sentin etäisyydeltä.

        Missä vaiheessa fotonit siitä tangosta alkaa tulemaan hitaammin sen silmään kun se tanko liikkuu, kun se työntää sitä? Vai huomaako se vaan jossain kohtaa et se tanko ei enää liikukaan? Koska eikös juuri näin tapahdu teidän teorian mukaan?

        "Vai huomaako se vaan jossain kohtaa et se tanko ei enää liikukaan? "

        Huomaisi, että tangon liike alkaa toisesta päästä ja siirtyy vuosien kuluessa koko tangon liikkeeksi. Useiden vuosien jälkeen myös tangon kauimmainenkin pää liikkuu.
        Näin se vaan menee kaikkien ammattifyysikoden mukaan, ei vain "meidän teorioiden" mukaan.

        Jos et ymmärrä asiaa, niin älä ammu vietin tuojaa. Tässä asiassa olet satavarmasti väärässä.


      • EiNytIhanAuennut
        ExB kirjoitti:

        Jäykät kappaleet suhteellisuusteoriassa tulivat esille jo varsin varhain. Ehrenfestin paradoksi on yksi esimerkki siitä, mitä kaikkea kivaa päänvaivaa ne aiheuttavat. Born-jäykkyys jne. Näitä siis mietittiin jo 1900 - luvun alkuvuosina ennen 1. maailmansotaa.

        https://en.wikipedia.org/wiki/Ehrenfest_paradox

        Luennoitsijat tykkäävät kiusata opiskelijoitaan tälläisillä laskutehtävillä:

        "Oletetaan että hyrrä pyörii 100 kierrosta sekunnissa. Mikä pitää hyrrän säteen olla, jotta sen kehän pituus on mahdollisimman suuri?"

        Suhteellisuusteoriaan ja muuhunkin fysiikkaan liittyviä kysymyksiä on käsitelty Fysiikan Usein Kysytyt Kysymykset - sivustolla

        http://www.math.ucr.edu/home/baez/physics/

        Entäs miten toi liittyy oikein mitenkään tähän tapaukseen, koska toi käsittelee vaan jotain relativistisillä nopeuksilla pyörivää hyrrää, ja sitäkin vain niin et havainnoija sekottaa päänsä seisomalla sillä hyrrällä!? :D
        Ei kai kukaan ollu työntämässäkään tota sauvaa läheskään valon nopeudella, varsinkaan samalla istuen sen päällä?


      • EiTäsKetääAmpumaaAleta
        FysiikkaaLukenut kirjoitti:

        "Vai huomaako se vaan jossain kohtaa et se tanko ei enää liikukaan? "

        Huomaisi, että tangon liike alkaa toisesta päästä ja siirtyy vuosien kuluessa koko tangon liikkeeksi. Useiden vuosien jälkeen myös tangon kauimmainenkin pää liikkuu.
        Näin se vaan menee kaikkien ammattifyysikoden mukaan, ei vain "meidän teorioiden" mukaan.

        Jos et ymmärrä asiaa, niin älä ammu vietin tuojaa. Tässä asiassa olet satavarmasti väärässä.

        Höpö höpö! :D
        Ensinnäkin! Kuinkas pitkä matka valovuosissa niillä fotoneilla on heijastua siitä tangosta sen havaitsija silmään parinkymmenen sentin päähän? ;)
        Joten se ei takuulla sen informaation nopeuteen vaikuta, eihän?

        Ja kuinkas se nyt onkin jo USEITA vuosia, jos kyseessä on vain valovuoden mittanen tanko? Joko tässä kinastellessa meni niin kauan aikaa et valon nopeuskin muuttui?

        Ja kyseessähän on siis ihan yks ja sama silmä. Kerros edes et miten se liikkeen näkymä edes vaihtuu siinä yhdessä ja samassa silmässä, jota pommittaa kokoajan siitä tangosta sinkovat fotonit? Siinä voi olla aivoparat kovilla kun koittaa muodostaa kuvan kappaleesta joka yhtäaikaa pysyy paikallaan, ja liikkuu? :)

        Mut muutetaas skenaarioita hieman! Tää "leveesilmä" käykin hakemassa sahan jostain, ja kovana urakoijana sahaa koko alumiinitangon metrin pituisiksi pätkiksi. Ja alkaa SIT työntämään sitä yhä hyvässä jonossa olevaa alumiinitankoletkaa! Kerro mones niistä tangoista pysähtyy, ja miksi?


      • SupistettuSuht.Teoria
        FysiikkaaLukenut kirjoitti:

        "Vai huomaako se vaan jossain kohtaa et se tanko ei enää liikukaan? "

        Huomaisi, että tangon liike alkaa toisesta päästä ja siirtyy vuosien kuluessa koko tangon liikkeeksi. Useiden vuosien jälkeen myös tangon kauimmainenkin pää liikkuu.
        Näin se vaan menee kaikkien ammattifyysikoden mukaan, ei vain "meidän teorioiden" mukaan.

        Jos et ymmärrä asiaa, niin älä ammu vietin tuojaa. Tässä asiassa olet satavarmasti väärässä.

        PS: Ja syy siihen, miks teil menee tää kokoajan mettään, on juurikin se, et te ette näköjään pysty käsittelemään tätä asiaa n ja ainoastaan ton suhteellisuusteorian premissien mukaan, joka on juurikin toi kaks eri havainnoijaa ja erot siinä että mitä he havaitsee ollessa ihan eri paikoissa toisiinsa nähden! :/ Mut missään kohtaa mikään suhteellisuusteoria ei o esittäny et havainnoijia on vain yksi! Tai voisittehannte kokeilla laittaa ne molemmat havainnoijat seisomaan toistensa varpailla? :P :D (ja antakaa niille molemmille sit se ylilevee silmä?)


      • teoria.on-suhteellista
        FyssaaLukenut kirjoitti:

        " Kaukana avaruudessa, eli tyhjiössä kappaleen saa myös liikkeelle huomattavsti pienemmällä voimalla, kuin planeettojen painovoimakentässä,"

        Newtonin toisen lain eli dynamiikan paruslain mukaan a = F : m. Millä kohtaa näet kaavassa planeetan painovoimakentän? Massan ominaisuus on hitaus.

        Kuten ExB:kin tuossa selitti, "Mekaaninen aalto enetee aineeseen enintään sähkömagneettisen vuorovaikutuksen nopeudella (valon nopeus tyhjiössä) mutta jokaisen atomin kohdalla se hidastuu kun atomille pitää saada vauhtia ennenkuin se puolestaan pääsee siirtymään pois tasapainoasemastaan ja vaikuttamaan seuraaviin atomeihin sähkökentällään. Tämä rajoittaa äänen etenemisnopeuden aineessa paljon valon tyhjiönopeutta pienemmäksi."

        1 vv:n pituisen tangon toinen pää lähtee liikkeelle vasta useiden vuosien kuluttua toisen pään työntämisestä, kuten sinulle on monen kirjoittajan toimesta selitetty. Nopeimmillaan se voisi lähteä liikkeelle vuoden kuluttua, mutta mekaaninen vaikutus on paljon valonnopeutta hitaampi, joten pitkäksi ehtii parta toisessa päässä kasvaa, ennen kuin se pää inahtaakaan. Jos se siirtyisi heti, siirtyisi myös informaatio siitä, että tankoa on työnnetty. Se rikkoisi aivan kiistatta erityisen suhteellisuusteorian periaatteita vastaan.

        Asia taitaa olla sinulle vähän liian vaikea. Koulutko vielä kesken?

        Siis, jotku täällä ottaa tuon putkiasian käytännön, ja jotku teorian kannalta. Siitäkö
        tämä lapsellinen kinastelu johtuu ???

        Juu tiedän sen, että käytännössä täysin jäykkää kappaletta ei voi tällä hetkellä maailmassa valmistaa, ja sen liikuttamien vaatisi valtavia voimia, MUTTA teoreettisesta näkökulmasta, jos 100 % jäykän, ja esim. 1 valovuoden mitaisen putken saisi liikkeelle, se kulkisi tyhjiössä samalla pituudella, kuin se on lepotilassa liikkumattomana.
        Joten sen alku, ja loppupiste A-------------->B, kulkisi työnnetyn (tai vedetyn) matkan samassa ajassa.

        Ps. Einsteinin suhteellisuusteoraakaan ei ole vielä edes kokonaan todistettu, joten nämä kaikki ovat vain olettamuksia.


      • EnNytIhanUskoTätä
        ExB kirjoitti:

        Pitkä metallitanko käyttäytyy kuten jousi kun sitä puristetaan:

        http://www.solitaryroad.com/c1030.html

        Kuvassa 2. näkyy, miten jousen pään liikuttaminen vaikuttaa jouseen. Siinä lähtee liikkeelle muuta jousta tiheämpi alue nopeudella, joka metallissa olisi äänen nopeus kompressioaallolle.

        Tässä youtube - video siitä, miten kompressioaalto etenee jousessa:

        https://www.youtube.com/watch?v=y7qS6SyyrFU

        Kun jousen päätä heilautetaan niin sen toisessa päässä ei heti tapahtu mitään. Tieto heilahduksesta tulee perille vasta kun aalto etenee perille asti. Sama koskee pitkää tankoa, jonka toista päätä liikautetaan. Liikautus lähtee aaltona etenemään pitkin tankoa.

        Kaikki atomeista koostuva aine pysyy koossa sähkömagneettisen vuorovaikutuksen ansiosta. Kvanttimekaniikka tekee tästä paljon monimutkaisempaa mutta ei muuta vuorovakutuksen tyyppiä.

        Kun metallitangon pään uloimpia atomeita liikutetaan niin ne alkavat sähkökenttänsä avulla vetämään ja työntämään seuraavan kerroksen atomeita. Nuo atomit jäävät hieman jälkeen liikkeestä oman hitautensa vuoksi mutta kun ne hiljakseen saavat vauhtia niin puolestaan liikuttavat seuraavaa atomikerrosta sähkökentän avulla jne.

        Mekaaninen aalto enetee aineeseen enintään sähkömagneettisen vuorovaikutuksen nopeudella (valon nopeus tyhjiössä) mutta jokaisen atomin kohdalla se hidastuu kun atomille pitää saada vauhtia ennenkuin se puolestaan pääsee siirtymään pois tasapainoasemastaan ja vaikuttamaan seuraaviin atomeihin sähkökentällään. Tämä rajoittaa äänen etenemisnopeuden aineessa paljon valon tyhjiönopeutta pienemmäksi.

        Fysiikan lakeja noudattavasta aineesta valmistetussa tangossa mekaaninen liike ei siis voi edetä edes valon tyhjiönopeudella vaan selvästi sitä hitaammin.

        Juuei! :/

        "Pitkä metallitanko käyttäytyy kuten jousi kun sitä puristetaan... ja blaa-blaa"

        - Juu eipäs nyt yritetä harhauttaa, ja koitetaan pitää mielessä et täs ei olla nyt asentamaan sun skodaas uutta jousitusta, eikä puhuta mistään elastisista kappaleista, vaan ihan jäykistä kappaleista! :) Kiitos! Ei tässä o otettu huomioon montaa muutakaan aitoa fysikaalista ilmiötä, niin ei oteta nyttenkään, vaan koitetan keskittyä ihan tuohon alkuperäseen hypoteettiseen dilemmaan, oisko mahollista? Koska tuskin maapallolla olis edes piitä niin paljon jotta tollai valovuoden-tanko saatais rakennettua? :D

        Ja mikä pakkomielle sulla on puhua kokoajan "aalloista" kun puhutaan liikkeestä?
        Sano vaan ihan reilusti "mekaaninen liike" äläkä "mekaaninen aalto"? Koska ei täs edelleenkä olla mitään ksylofonia rakentamassa!? Ja toi sun meriselitys atomien liikkumiseelle kiinteessä aineessa kyl sais mun mielestäni aikaan sen, et vähänki pidemmät kappaleet liikkuis sit kun hyytelö, kunnes ne sen kiinteän kappaleen viimesetkin atomit saa sen liikevoiman sinne päähän perille!? :D Eikä väliä edes sillä et mistä materiaalista se kappale on valmistettu? Ja kerroppas et mitä sille liike-energialle siellä kappaleen toisessa päässä sit tapahtuu, koska fysiikan lakien mukaanhan energia ei häviä mihinkään? Lentääkö se sieltä esim. fotoneina tms. mäkeen? Sit taskulamput olis aika turha keksintö, koska ihan mitä tahansa kappaletta liikuttamalla sais aikaan valoa! :P

        Ja jos oikea kappale liikkuis kuten sä selitit, niin kerroppas miten sellanen kappale sit liikkuu, jota ei työnnetäkään, vaan nostetaan ja siirretään? Esim: Tikkuaski. Otat tikkuaskin peukalon ja etusormen väliis, ja nostat sitä. Sun liiketeorias mukaan tässä liike-energia kohdistuu noista sormista tikkuaskia kohti, ja siirtää sen atomeita ainoastaan sisäänpäin. (jos söit aamupuuros). No mitäs sit tapahtuu jos mä lähdenkin nostamaan sitä tikkuaskia ylöspäin? Mikä voima työntää niitä tikkuaskin alaosan atomeita ylöspäin, joka saa sit aikaan ton sun hyytelöliikkeen ylöspäin sinä kappaleessa, joka siirtää sit lopulta koko palettia ylöspäin, kun se saavuttaa sen yläpinnankin?
        Anti-painovoimako? Vai mekaaninen aalto? Ja mistäs se sit yht'äkkiä tuli?


      • EnNytIhanUskoTätä kirjoitti:

        Juuei! :/

        "Pitkä metallitanko käyttäytyy kuten jousi kun sitä puristetaan... ja blaa-blaa"

        - Juu eipäs nyt yritetä harhauttaa, ja koitetaan pitää mielessä et täs ei olla nyt asentamaan sun skodaas uutta jousitusta, eikä puhuta mistään elastisista kappaleista, vaan ihan jäykistä kappaleista! :) Kiitos! Ei tässä o otettu huomioon montaa muutakaan aitoa fysikaalista ilmiötä, niin ei oteta nyttenkään, vaan koitetan keskittyä ihan tuohon alkuperäseen hypoteettiseen dilemmaan, oisko mahollista? Koska tuskin maapallolla olis edes piitä niin paljon jotta tollai valovuoden-tanko saatais rakennettua? :D

        Ja mikä pakkomielle sulla on puhua kokoajan "aalloista" kun puhutaan liikkeestä?
        Sano vaan ihan reilusti "mekaaninen liike" äläkä "mekaaninen aalto"? Koska ei täs edelleenkä olla mitään ksylofonia rakentamassa!? Ja toi sun meriselitys atomien liikkumiseelle kiinteessä aineessa kyl sais mun mielestäni aikaan sen, et vähänki pidemmät kappaleet liikkuis sit kun hyytelö, kunnes ne sen kiinteän kappaleen viimesetkin atomit saa sen liikevoiman sinne päähän perille!? :D Eikä väliä edes sillä et mistä materiaalista se kappale on valmistettu? Ja kerroppas et mitä sille liike-energialle siellä kappaleen toisessa päässä sit tapahtuu, koska fysiikan lakien mukaanhan energia ei häviä mihinkään? Lentääkö se sieltä esim. fotoneina tms. mäkeen? Sit taskulamput olis aika turha keksintö, koska ihan mitä tahansa kappaletta liikuttamalla sais aikaan valoa! :P

        Ja jos oikea kappale liikkuis kuten sä selitit, niin kerroppas miten sellanen kappale sit liikkuu, jota ei työnnetäkään, vaan nostetaan ja siirretään? Esim: Tikkuaski. Otat tikkuaskin peukalon ja etusormen väliis, ja nostat sitä. Sun liiketeorias mukaan tässä liike-energia kohdistuu noista sormista tikkuaskia kohti, ja siirtää sen atomeita ainoastaan sisäänpäin. (jos söit aamupuuros). No mitäs sit tapahtuu jos mä lähdenkin nostamaan sitä tikkuaskia ylöspäin? Mikä voima työntää niitä tikkuaskin alaosan atomeita ylöspäin, joka saa sit aikaan ton sun hyytelöliikkeen ylöspäin sinä kappaleessa, joka siirtää sit lopulta koko palettia ylöspäin, kun se saavuttaa sen yläpinnankin?
        Anti-painovoimako? Vai mekaaninen aalto? Ja mistäs se sit yht'äkkiä tuli?

        Kiinteän aineen fysiikka käsittelee aineen mikroskooppista rakennetta kvanttimekaniikkaa käyttäen. Siinä tulee esille mekaanisten aaltojen eteneminen mikrotasolla. Sivusuuntaiset voimat välittyvät samalla mekanismilla kuin pituussuuntaisetkin. Jos aihe kiinnostaa niin siitä löytyy runsaasti kirjallisuutta yliopistojen fysiikan osastojen kirjastoista ja myös netistä. Täällä ei yksityiskohtiin kannata mennä varsinkaan keskustelussa, joka on väärällä palstalla.

        Mekaanisen aaltoliikkeen etenemisnopeudet tulevat usein vastaan tutkittaessa ainetta ultraäänellä. Nopeudet ovat poikittaiselle liikkeelle (t r a n s v e r s e) yleensä selvästi pienempiä kuin pitkittäisellä johtuen erilaisesta voiman suunnasta.

        http://www.advanced-ndt.co.uk/index_htm_files/Reference Chart - Velocity Chart.pdf

        https://urly.fi/PUR

        Kaikkein suurin tunnettu äänen nopeus metalleista on berylliumissa, jossa pitkittäinen aaltoliike etenee 12878 m/s ja poikittainen aaltoliike etenee nopeudella 8870 m/s. Valitaan taulukosta hitaimman äänen nopeuden tarjoava metalli eli lyijy, jolle pitkittäinen aalto etenee 2159m/s ja poikittainen aalto vain 813m/s. Lyijyssä siis pitäisi näkyä ongelmien kaikkein selvimmin. Puussa äänen nopeus on suurempi kuin lyijyssä joten katsotaan lyijyn numeroarvoilla.

        Kun lähdet tikkuaskia liikuttamaan niin reaktionopeutesi on luokkaa 0.1 sekuntia. Siinä ajassa mekaaninen poikittaissuuntainen aalto ehtii lyijyssäkin kulkea noin 80 metrin matkan. Kun liikutettavan kappaleen koko pisimmässä suunnassa on 0.05 metriä niin liikkeelle tönäisystä syntyvä aalto on jo 0.001 sekunnin kuluessa kuluessa ehtinyt kulkea koko kappaleen lävitse edestakaisin päädyistä heijastuen 8 kertaa eli sivusuuntainen voima on ehtinyt välittyä hyvin koko kappaleeseen.

        Mekaanisen aallon etenemistä tämäkin eli näin erisuuntaiset voimat välittyvät metallilaattaan törmäävässä luodissa. Luodin kärki tai toinen kylki räiskyy sulana metallina joka suuntaan samaan aikaan kun sen perä tai kauempana laatasta oleva kylki on vielä täysin ehjä. Näin käy kun materiaaliin kohdistuu sen lujuuteen nähden liian suuria voimia. Näissä luodin perä hajoaa vasta, kun laatasta takaisinpäin virtaava

        https://www.youtube.com/watch?v=QfDoQwIAaXg


      • ExB kirjoitti:

        Kiinteän aineen fysiikka käsittelee aineen mikroskooppista rakennetta kvanttimekaniikkaa käyttäen. Siinä tulee esille mekaanisten aaltojen eteneminen mikrotasolla. Sivusuuntaiset voimat välittyvät samalla mekanismilla kuin pituussuuntaisetkin. Jos aihe kiinnostaa niin siitä löytyy runsaasti kirjallisuutta yliopistojen fysiikan osastojen kirjastoista ja myös netistä. Täällä ei yksityiskohtiin kannata mennä varsinkaan keskustelussa, joka on väärällä palstalla.

        Mekaanisen aaltoliikkeen etenemisnopeudet tulevat usein vastaan tutkittaessa ainetta ultraäänellä. Nopeudet ovat poikittaiselle liikkeelle (t r a n s v e r s e) yleensä selvästi pienempiä kuin pitkittäisellä johtuen erilaisesta voiman suunnasta.

        http://www.advanced-ndt.co.uk/index_htm_files/Reference Chart - Velocity Chart.pdf

        https://urly.fi/PUR

        Kaikkein suurin tunnettu äänen nopeus metalleista on berylliumissa, jossa pitkittäinen aaltoliike etenee 12878 m/s ja poikittainen aaltoliike etenee nopeudella 8870 m/s. Valitaan taulukosta hitaimman äänen nopeuden tarjoava metalli eli lyijy, jolle pitkittäinen aalto etenee 2159m/s ja poikittainen aalto vain 813m/s. Lyijyssä siis pitäisi näkyä ongelmien kaikkein selvimmin. Puussa äänen nopeus on suurempi kuin lyijyssä joten katsotaan lyijyn numeroarvoilla.

        Kun lähdet tikkuaskia liikuttamaan niin reaktionopeutesi on luokkaa 0.1 sekuntia. Siinä ajassa mekaaninen poikittaissuuntainen aalto ehtii lyijyssäkin kulkea noin 80 metrin matkan. Kun liikutettavan kappaleen koko pisimmässä suunnassa on 0.05 metriä niin liikkeelle tönäisystä syntyvä aalto on jo 0.001 sekunnin kuluessa kuluessa ehtinyt kulkea koko kappaleen lävitse edestakaisin päädyistä heijastuen 8 kertaa eli sivusuuntainen voima on ehtinyt välittyä hyvin koko kappaleeseen.

        Mekaanisen aallon etenemistä tämäkin eli näin erisuuntaiset voimat välittyvät metallilaattaan törmäävässä luodissa. Luodin kärki tai toinen kylki räiskyy sulana metallina joka suuntaan samaan aikaan kun sen perä tai kauempana laatasta oleva kylki on vielä täysin ehjä. Näin käy kun materiaaliin kohdistuu sen lujuuteen nähden liian suuria voimia. Näissä luodin perä hajoaa vasta, kun laatasta takaisinpäin virtaava

        https://www.youtube.com/watch?v=QfDoQwIAaXg

        Tässä hieman yksityiskohtaisemmin mutta edelleen varsin perustasolla mekaanisten aaltojen etenemisestä kiinteässä aineessa.

        https://www.binghamton.edu/physics/docs/lattice-waves.pdf

        Jos siis aihe kiinnostaa. Yleensä ihmiset tikahtuvat jo varhaisessa vaiheessa siihen matematiikkaan, jota jo ideaalisten kiderakenteidenkin käsittelyssä tarvitaan.


      • ExB kirjoitti:

        Kiinteän aineen fysiikka käsittelee aineen mikroskooppista rakennetta kvanttimekaniikkaa käyttäen. Siinä tulee esille mekaanisten aaltojen eteneminen mikrotasolla. Sivusuuntaiset voimat välittyvät samalla mekanismilla kuin pituussuuntaisetkin. Jos aihe kiinnostaa niin siitä löytyy runsaasti kirjallisuutta yliopistojen fysiikan osastojen kirjastoista ja myös netistä. Täällä ei yksityiskohtiin kannata mennä varsinkaan keskustelussa, joka on väärällä palstalla.

        Mekaanisen aaltoliikkeen etenemisnopeudet tulevat usein vastaan tutkittaessa ainetta ultraäänellä. Nopeudet ovat poikittaiselle liikkeelle (t r a n s v e r s e) yleensä selvästi pienempiä kuin pitkittäisellä johtuen erilaisesta voiman suunnasta.

        http://www.advanced-ndt.co.uk/index_htm_files/Reference Chart - Velocity Chart.pdf

        https://urly.fi/PUR

        Kaikkein suurin tunnettu äänen nopeus metalleista on berylliumissa, jossa pitkittäinen aaltoliike etenee 12878 m/s ja poikittainen aaltoliike etenee nopeudella 8870 m/s. Valitaan taulukosta hitaimman äänen nopeuden tarjoava metalli eli lyijy, jolle pitkittäinen aalto etenee 2159m/s ja poikittainen aalto vain 813m/s. Lyijyssä siis pitäisi näkyä ongelmien kaikkein selvimmin. Puussa äänen nopeus on suurempi kuin lyijyssä joten katsotaan lyijyn numeroarvoilla.

        Kun lähdet tikkuaskia liikuttamaan niin reaktionopeutesi on luokkaa 0.1 sekuntia. Siinä ajassa mekaaninen poikittaissuuntainen aalto ehtii lyijyssäkin kulkea noin 80 metrin matkan. Kun liikutettavan kappaleen koko pisimmässä suunnassa on 0.05 metriä niin liikkeelle tönäisystä syntyvä aalto on jo 0.001 sekunnin kuluessa kuluessa ehtinyt kulkea koko kappaleen lävitse edestakaisin päädyistä heijastuen 8 kertaa eli sivusuuntainen voima on ehtinyt välittyä hyvin koko kappaleeseen.

        Mekaanisen aallon etenemistä tämäkin eli näin erisuuntaiset voimat välittyvät metallilaattaan törmäävässä luodissa. Luodin kärki tai toinen kylki räiskyy sulana metallina joka suuntaan samaan aikaan kun sen perä tai kauempana laatasta oleva kylki on vielä täysin ehjä. Näin käy kun materiaaliin kohdistuu sen lujuuteen nähden liian suuria voimia. Näissä luodin perä hajoaa vasta, kun laatasta takaisinpäin virtaava

        https://www.youtube.com/watch?v=QfDoQwIAaXg

        "Näissä luodin perä hajoaa vasta, kun laatasta takaisinpäin virtaava"
        metalli työntyy vielä laattaa kohti liikkuvan materiaalin lävitse.


      • ExB kirjoitti:

        "Näissä luodin perä hajoaa vasta, kun laatasta takaisinpäin virtaava"
        metalli työntyy vielä laattaa kohti liikkuvan materiaalin lävitse.

        Kiitos taas kerran kommenteistasi. Asiantuntevaa tekstiä on mukava lukea ja aina siitä jotain saa, vaikka sitten matematiikkaan tikahtuisikin. Itse luovutan nykyään viimeistään silloin, kun nabloja alkaa tulla kehiin. ;-)


      • menkää_hoitoon
        agnoskepo kirjoitti:

        Kiitos taas kerran kommenteistasi. Asiantuntevaa tekstiä on mukava lukea ja aina siitä jotain saa, vaikka sitten matematiikkaan tikahtuisikin. Itse luovutan nykyään viimeistään silloin, kun nabloja alkaa tulla kehiin. ;-)

        Voi wattu mitä sekopäitä te oikeen ootte ??? Houritte ihan jostain muusta aiheesta, kuin mitä ketjun aloittaja kysyy !!!


      • Teoree.tikko
        FyssaaLukenut kirjoitti:

        "ÄLÄKÄ VAAN HAKEUDU NOILLA TIEDOILLA MIHINKÄÄN TEKNIIKKAAN JA TIETEISIIN LIITTYVÄÄN KOULUTUKSEEN. SINUT NAURETAAN PIHALLE SIELTÄ VÄLITTÖMÄSTI !!! "

        Olen ollut tekniikan opettaja. Nyt jo eläkkeellä.

        No sen kyllä huomaa.! Opettajat ei näköjään tajua mitä eroa on teorialla, ja käytännöllä !
        teoriassaan kaikki on mahdollista !


      • TurhaanMöliset
        Teoree.tikko kirjoitti:

        No sen kyllä huomaa.! Opettajat ei näköjään tajua mitä eroa on teorialla, ja käytännöllä !
        teoriassaan kaikki on mahdollista !

        Tuossa, mistä ExB, FysiikkaaLukenut ja Agnoskepo kirjoittavat on kyse kivikovasta käytännöstä. Heidän selityksensä ovat oikeita ja sinä olet uskomuksinesi metsässä. Sinun mekanistiseen biljardipallomaailmaasi se ei vaan tunnu uppoavan.

        Kaikki ei ole mahdollista sen paremmin teoriassa kuin käytännössäkään. Yksi sellainen asia on voimavaikutuksen siirtyminen yli valonnopeudella.


      • TurpaUmpeen.hörhö
        TurhaanMöliset kirjoitti:

        Tuossa, mistä ExB, FysiikkaaLukenut ja Agnoskepo kirjoittavat on kyse kivikovasta käytännöstä. Heidän selityksensä ovat oikeita ja sinä olet uskomuksinesi metsässä. Sinun mekanistiseen biljardipallomaailmaasi se ei vaan tunnu uppoavan.

        Kaikki ei ole mahdollista sen paremmin teoriassa kuin käytännössäkään. Yksi sellainen asia on voimavaikutuksen siirtyminen yli valonnopeudella.

        Turhaan möliset sinäkin, " ylivalonopeus hörhö". Teoriassa kaikki on madollista.! PISTE !!


      • JospaLiikutettaisJotain
        ExB kirjoitti:

        Tässä hieman yksityiskohtaisemmin mutta edelleen varsin perustasolla mekaanisten aaltojen etenemisestä kiinteässä aineessa.

        https://www.binghamton.edu/physics/docs/lattice-waves.pdf

        Jos siis aihe kiinnostaa. Yleensä ihmiset tikahtuvat jo varhaisessa vaiheessa siihen matematiikkaan, jota jo ideaalisten kiderakenteidenkin käsittelyssä tarvitaan.

        No ei oikeastaan, koska sä oot näköjään vieläkin rakentamassa sitä
        ksylofonias?

        "Phonons are a quantum mechanical version of a special type of vibrational motion, known as normal modes in classical mechanics, in which each part of a lattice oscillates with the same frequency."


      • TryToDealWithIt
        TurpaUmpeen.hörhö kirjoitti:

        Turhaan möliset sinäkin, " ylivalonopeus hörhö". Teoriassa kaikki on madollista.! PISTE !!

        Juu.
        Teoriassa on mahdollista että mä lopetan mölisemisen.
        Käytännössä ei! :D


      • HelpotetaanElämää
        ExB kirjoitti:

        Kiinteän aineen fysiikka käsittelee aineen mikroskooppista rakennetta kvanttimekaniikkaa käyttäen. Siinä tulee esille mekaanisten aaltojen eteneminen mikrotasolla. Sivusuuntaiset voimat välittyvät samalla mekanismilla kuin pituussuuntaisetkin. Jos aihe kiinnostaa niin siitä löytyy runsaasti kirjallisuutta yliopistojen fysiikan osastojen kirjastoista ja myös netistä. Täällä ei yksityiskohtiin kannata mennä varsinkaan keskustelussa, joka on väärällä palstalla.

        Mekaanisen aaltoliikkeen etenemisnopeudet tulevat usein vastaan tutkittaessa ainetta ultraäänellä. Nopeudet ovat poikittaiselle liikkeelle (t r a n s v e r s e) yleensä selvästi pienempiä kuin pitkittäisellä johtuen erilaisesta voiman suunnasta.

        http://www.advanced-ndt.co.uk/index_htm_files/Reference Chart - Velocity Chart.pdf

        https://urly.fi/PUR

        Kaikkein suurin tunnettu äänen nopeus metalleista on berylliumissa, jossa pitkittäinen aaltoliike etenee 12878 m/s ja poikittainen aaltoliike etenee nopeudella 8870 m/s. Valitaan taulukosta hitaimman äänen nopeuden tarjoava metalli eli lyijy, jolle pitkittäinen aalto etenee 2159m/s ja poikittainen aalto vain 813m/s. Lyijyssä siis pitäisi näkyä ongelmien kaikkein selvimmin. Puussa äänen nopeus on suurempi kuin lyijyssä joten katsotaan lyijyn numeroarvoilla.

        Kun lähdet tikkuaskia liikuttamaan niin reaktionopeutesi on luokkaa 0.1 sekuntia. Siinä ajassa mekaaninen poikittaissuuntainen aalto ehtii lyijyssäkin kulkea noin 80 metrin matkan. Kun liikutettavan kappaleen koko pisimmässä suunnassa on 0.05 metriä niin liikkeelle tönäisystä syntyvä aalto on jo 0.001 sekunnin kuluessa kuluessa ehtinyt kulkea koko kappaleen lävitse edestakaisin päädyistä heijastuen 8 kertaa eli sivusuuntainen voima on ehtinyt välittyä hyvin koko kappaleeseen.

        Mekaanisen aallon etenemistä tämäkin eli näin erisuuntaiset voimat välittyvät metallilaattaan törmäävässä luodissa. Luodin kärki tai toinen kylki räiskyy sulana metallina joka suuntaan samaan aikaan kun sen perä tai kauempana laatasta oleva kylki on vielä täysin ehjä. Näin käy kun materiaaliin kohdistuu sen lujuuteen nähden liian suuria voimia. Näissä luodin perä hajoaa vasta, kun laatasta takaisinpäin virtaava

        https://www.youtube.com/watch?v=QfDoQwIAaXg

        Entäs jos koittasitkin liikuttaa sitä koko kappaletta kerralla ihan näillä laskukaavoilla, niin ei tarvis jokaista atomia erikseen alkaa siirtelemään?

        en.m.wikipedia.org/wiki/Continuum_mechanics

        Ton perusteella se on vallan mahdollista sekin.


      • ei.ole.jäykkää.ainetta
        HelpotetaanElämää kirjoitti:

        Entäs jos koittasitkin liikuttaa sitä koko kappaletta kerralla ihan näillä laskukaavoilla, niin ei tarvis jokaista atomia erikseen alkaa siirtelemään?

        en.m.wikipedia.org/wiki/Continuum_mechanics

        Ton perusteella se on vallan mahdollista sekin.

        Tuosta linkistäs jäi sulta lukematta yksi lause otsikolla Fo.rc.es. in a co.nt.in.uu.m alkavan kappaleen alusta.

        Co.nt.in.uu.m m.ec.ha.ni.cs de.al.s wi.t.h de.fo.rma.bl.e bo.di.es, as o.pp.os.ed to ri.gi.d bo.di.es.

        Kyseinen mekaniikan esitysmuoto nimenomaan siis ei käsittele eikä sovellu täysin jäykän aineen kuvaamiseen vaan käsittelee vain elastista ainetta. Täysin jäykkä sauva tai putki ei kuulu tuollaiseen malliin ollenkaan vaan rikkoisi perusoletuksen muovautuvuudesta.


      • Ei.ole.ei
        ei.ole.jäykkää.ainetta kirjoitti:

        Tuosta linkistäs jäi sulta lukematta yksi lause otsikolla Fo.rc.es. in a co.nt.in.uu.m alkavan kappaleen alusta.

        Co.nt.in.uu.m m.ec.ha.ni.cs de.al.s wi.t.h de.fo.rma.bl.e bo.di.es, as o.pp.os.ed to ri.gi.d bo.di.es.

        Kyseinen mekaniikan esitysmuoto nimenomaan siis ei käsittele eikä sovellu täysin jäykän aineen kuvaamiseen vaan käsittelee vain elastista ainetta. Täysin jäykkä sauva tai putki ei kuulu tuollaiseen malliin ollenkaan vaan rikkoisi perusoletuksen muovautuvuudesta.

        Ja sukta jäi lukematta useampikin kohta, ja ymmärsit väärin senkin jonka luulit ymmärtäneesi, eli lainauksesi! :) Koska missäs kohtaa tossa muka sanotaan et on poissuljettua käyttää tätä menetelmään MYÖS jäykän kappaleen tapauksessa, mutta jäykän kappaleen menetelmät taas EI sovellu tähän tapaukseen, koska tässä kappale voi olla myös muotoutuva! Tavaa:

        "Modeling an object as a continuum assumes that the substance of the object completely fills the space it occupies. Modeling objects in this way ignores the fact that matter is made of atoms, and so is not continuous; however, on length scales much greater than that of inter-atomic distances, such models are highly accurate. Fundamental physical laws such as the conservation of mass, the conservation of momentum, and the conservation of energy may be applied to such models to derive differential equations describing the behavior of such objects, and some information about the particular material studied is added through constitutive relations."

        Ja sen jälkeen lue lisäks se kinematics linkistä löytyvä juttukin, jos viisastuisit sitten senverran ettet enää alkas hölöttämään muita lukemisen puutteesta? :)


    • riippuu_katsojasta

      Alkuperäiseen kysymykseen vielä sen verran, että mitä he näkevät 10 000 vv päästä maapallolta, riippuu siitä millainen havaintokyky sen planeetan muukalaisilla, olioilla tms. on.?

      Valonnopeus on pelkästään maapallolla mitattu nopein aikamuoto, ja perustuu vain ihmisen havaintokykyyn, ja mittauksiin. Jollain toisella planeetalla saatetaan havaita asioita valoakin nopeammin, koska mitä todennäköisimmin, elämää on muuallakin maailmankaikkeudessa.

      • "Valonnopeus on pelkästään maapallolla mitattu nopein aikamuoto, ja perustuu vain ihmisen havaintokykyyn, ja mittauksiin."

        Ei pidä paikkaansa. On erittäin hyvä syy olettaa, että c on universaali. Valonnopeuden suuruuteen vaikuttaa kaksi luonnonvakiota, tyhjiön permittiivisyys ja -permeabiliteetti. Niistä toisen tai molempien pitäisi muuttua jos c muuttuisi.


      • riippuu_katsojasta
        agnoskepo kirjoitti:

        "Valonnopeus on pelkästään maapallolla mitattu nopein aikamuoto, ja perustuu vain ihmisen havaintokykyyn, ja mittauksiin."

        Ei pidä paikkaansa. On erittäin hyvä syy olettaa, että c on universaali. Valonnopeuden suuruuteen vaikuttaa kaksi luonnonvakiota, tyhjiön permittiivisyys ja -permeabiliteetti. Niistä toisen tai molempien pitäisi muuttua jos c muuttuisi.

        Juu olettaa voi, mutta valonnopeus on vain ihmiskunnan luoma n. 100 vuotta vanha luonnonvakio. Ihmistä kehittyneempää sivilisaatioita voi olla muallakin universiumissa. Ja sitäpaitsi valonopeuden yli on päästy jo täällä maassakin.

        https://www.is.fi/ulkomaat/art-2000000433832.html


      • riippuu_katsojasta kirjoitti:

        Juu olettaa voi, mutta valonnopeus on vain ihmiskunnan luoma n. 100 vuotta vanha luonnonvakio. Ihmistä kehittyneempää sivilisaatioita voi olla muallakin universiumissa. Ja sitäpaitsi valonopeuden yli on päästy jo täällä maassakin.

        https://www.is.fi/ulkomaat/art-2000000433832.html

        Tähän mennessä eli viimeisen yli sadan vuoden ajan ovat havainnot valon tyhjiönopeuden ylittämisestä osoittautuneet lähemmin tarkasteltuna mittausvirheiksi. Niin tuossakin.

        Pettymys: Valoa nopeampia hiukkasia ei ehkä sittenkään
        https://www.is.fi/tiede/art-2000000486668.html


      • ExB kirjoitti:

        Tähän mennessä eli viimeisen yli sadan vuoden ajan ovat havainnot valon tyhjiönopeuden ylittämisestä osoittautuneet lähemmin tarkasteltuna mittausvirheiksi. Niin tuossakin.

        Pettymys: Valoa nopeampia hiukkasia ei ehkä sittenkään
        https://www.is.fi/tiede/art-2000000486668.html

        Kyseessä taisi olla monimutkaisen mittauslaitteiston huonosti kiinnitetty liitin.
        Valonnopeuden ylityksiä putkahteli vielä takavuosina lähes parin vuoden välein, mutta joka kerta kyseessä oli uutisankka.
        Ihmistä kehittyneempiäkin sivilisaatiota koskevat luonnonlait.


      • martta00
        agnoskepo kirjoitti:

        Kyseessä taisi olla monimutkaisen mittauslaitteiston huonosti kiinnitetty liitin.
        Valonnopeuden ylityksiä putkahteli vielä takavuosina lähes parin vuoden välein, mutta joka kerta kyseessä oli uutisankka.
        Ihmistä kehittyneempiäkin sivilisaatiota koskevat luonnonlait.

        paitsi jos ne tulevat toisesta universumista, niin eikös siellä voi olla omat erilaiset luonnonlait?


      • martta00 kirjoitti:

        paitsi jos ne tulevat toisesta universumista, niin eikös siellä voi olla omat erilaiset luonnonlait?

        Aivan totta. Jos ihminen tulee Suomeen vaikka Jemenistä on hänellä varmasti ollut ainakin osittain eri lait. Suomessa hän joutuu kuitenkin noudattamaan Suomen lakeja. Eikö niin ;-)

        Olikohan aito martta00?


      • martta00
        agnoskepo kirjoitti:

        Aivan totta. Jos ihminen tulee Suomeen vaikka Jemenistä on hänellä varmasti ollut ainakin osittain eri lait. Suomessa hän joutuu kuitenkin noudattamaan Suomen lakeja. Eikö niin ;-)

        Olikohan aito martta00?

        aito on ;-)

        Tarkoitin siis, että jos niitä toisia universumeita on, niin siellähän luonnonvakioilla kuten valonkin tyhjiönopeudella voi olla ihan oma arvonsa. Ja tämän arvon ei tarvitse meidän universumiin skaalattuna olla 299 792 458 m/s. Toki sitten, jos ne jotenkin pääsevät tulemaan tänne, niin niittenkin täytyy tyytyä tuohon lukemaan.


      • martta00 kirjoitti:

        aito on ;-)

        Tarkoitin siis, että jos niitä toisia universumeita on, niin siellähän luonnonvakioilla kuten valonkin tyhjiönopeudella voi olla ihan oma arvonsa. Ja tämän arvon ei tarvitse meidän universumiin skaalattuna olla 299 792 458 m/s. Toki sitten, jos ne jotenkin pääsevät tulemaan tänne, niin niittenkin täytyy tyytyä tuohon lukemaan.

        Niihän se on. Fred Adamsin populaari kirja "Elämää multiversumissa" esittää mielenkiintoisesti multiversumihypoteesin, mutta siinä perusoletus on, ettei vuorovaikutusta voi olla eri universumien välillä. Adamsin jutut multiversumista antaisivat tukea antrooppiselle periaatteelle. Olemme täällä havainnoimassa, koska juuri tämä universumi on sellainen, jossa ihmisen kaltainen biologinen olio voi kehittyä.
        Jos taas vuorovaikutusta ei voi olla, hypoteesin todentaminen taitaa olla mahdotonta.


      • yyber-speed
        ExB kirjoitti:

        Tähän mennessä eli viimeisen yli sadan vuoden ajan ovat havainnot valon tyhjiönopeuden ylittämisestä osoittautuneet lähemmin tarkasteltuna mittausvirheiksi. Niin tuossakin.

        Pettymys: Valoa nopeampia hiukkasia ei ehkä sittenkään
        https://www.is.fi/tiede/art-2000000486668.html

        Joo mut eristeessähän valon nopeus kyllä ylittyy ihan joka päivä jokaisessa ydinvoimalassa!

        fi.(kopsaa)m.wikipedia.org/wiki/Tšerenkovin_säteily


      • FotonienVaihtoviikot
        martta00 kirjoitti:

        aito on ;-)

        Tarkoitin siis, että jos niitä toisia universumeita on, niin siellähän luonnonvakioilla kuten valonkin tyhjiönopeudella voi olla ihan oma arvonsa. Ja tämän arvon ei tarvitse meidän universumiin skaalattuna olla 299 792 458 m/s. Toki sitten, jos ne jotenkin pääsevät tulemaan tänne, niin niittenkin täytyy tyytyä tuohon lukemaan.

        Juu juu! Mutta ei ne muu-universumilaiset kuitenkaan oman universuminsa fysiikan lakejaan tähän meidän universumiimme varmaan ihan pikku säädöllä saa mukanaan tuotua! ;) :D

        Vai meinaatko jotain nykyversiota hölmölän valon kannosta säkillä sisään tupaan? :P Kun muu-universumilaiset avaa avaruusaluksensa oven jonne on varastoitunut heidän oman universuminsa paljon nopeampia fotoneita, ja ne pääsee sit tuosta ovesta karkuun tänne meidän maailmaamme, niin omat fotonimme valahtavat kateudesta vihreiksi? :D


      • yyber-speed kirjoitti:

        Joo mut eristeessähän valon nopeus kyllä ylittyy ihan joka päivä jokaisessa ydinvoimalassa!

        fi.(kopsaa)m.wikipedia.org/wiki/Tšerenkovin_säteily

        Valonnopeus on eri asia kuin valon nopeus.


      • KepposAgno
        agnoskepo kirjoitti:

        Valonnopeus on eri asia kuin valon nopeus.

        Ei ole.
        Tyhjiössä valon nopeus = valonnopeus.

        Ja sanoinko mä että valonnopeus ylittyy eritteessä? Sanoinko?
        Joten mitä sä siinä oikein mussutat?


      • KepposAgno kirjoitti:

        Ei ole.
        Tyhjiössä valon nopeus = valonnopeus.

        Ja sanoinko mä että valonnopeus ylittyy eritteessä? Sanoinko?
        Joten mitä sä siinä oikein mussutat?

        Multinikkimulkero on eritteessä siellä, jonne ei päivä paista, eikä valo kajasta. ; )


      • KepposAgno kirjoitti:

        Ei ole.
        Tyhjiössä valon nopeus = valonnopeus.

        Ja sanoinko mä että valonnopeus ylittyy eritteessä? Sanoinko?
        Joten mitä sä siinä oikein mussutat?

        "Ja sanoinko mä että valonnopeus ylittyy eritteessä? Sanoinko?
        Joten mitä sä siinä oikein mussutat? "
        Et sanonut, mutta mökelsit asiaan kuulumattomia tyhjänpäiväisyyksiä.

        Valon nopeus on nopeus, jolla valo kulkee väliaineessa. Yhteen kirjoitettuna valonnopeus on luonnon limiitti eli maksiminopeus, jolla informaatio voi avaruudessa siirtyä.

        Mikä tässä muka on niin vaikeaa?


      • NopeusValon
        agnoskepo kirjoitti:

        "Ja sanoinko mä että valonnopeus ylittyy eritteessä? Sanoinko?
        Joten mitä sä siinä oikein mussutat? "
        Et sanonut, mutta mökelsit asiaan kuulumattomia tyhjänpäiväisyyksiä.

        Valon nopeus on nopeus, jolla valo kulkee väliaineessa. Yhteen kirjoitettuna valonnopeus on luonnon limiitti eli maksiminopeus, jolla informaatio voi avaruudessa siirtyä.

        Mikä tässä muka on niin vaikeaa?

        Jaahas nyt täällä nussitaan jo pilkkuja. Ihankos wikipediasta tarkistit asian, kun on sanatarkasti vastaus.. ? Taidat olla suomi24 vakiohörhöjä. Muista välillä ottaa lääkkees, että pysyt hereillä 24h/7.


      • nopeuskovalonko
        NopeusValon kirjoitti:

        Jaahas nyt täällä nussitaan jo pilkkuja. Ihankos wikipediasta tarkistit asian, kun on sanatarkasti vastaus.. ? Taidat olla suomi24 vakiohörhöjä. Muista välillä ottaa lääkkees, että pysyt hereillä 24h/7.

        Ja sinä taidat olla palstan vakiokusipää


      • paskatkosull.housussa
        nopeuskovalonko kirjoitti:

        Ja sinä taidat olla palstan vakiokusipää

        Ja sinä et taida olla, vaan olet ihan aito vakiopasskahousu !!


      • Täällähän on päästy ihan asian ytimeen :D
        Jos todetaan vaikka, että ollaan eri mieltä ja lähdetään kukin omaan suuntaan ja pidetään viikko taukoa silleen, ettei jutella keskenään?
        Tehtäiskö tolleen?


      • martta00 kirjoitti:

        paitsi jos ne tulevat toisesta universumista, niin eikös siellä voi olla omat erilaiset luonnonlait?

        Toisesta universumista tänne vuotavat luonnonlait? Kuullostaa tutulta. Sitä on vissiin luettu Isaac Asimovin kirjaa "Itse jumalat".

        https://fi.wikipedia.org/wiki/Itse_jumalat

        Nettikeskustelua seuratessa tulee muuten aika usein mieleen tuo kirjan sisällysluettelon muodostava Schillerin sitaatti kokonaisuudessaan: "Tyhmyyttä vastaan itse jumalatkin taistelevat turhaan". Jokainen sitten itse valitsee mihin asti taistelua kunkin keskustelun kohdalla jatkaa ja missä kohdassa toteaa taistelun jatkamisen turhaksi.


      • ExB kirjoitti:

        Toisesta universumista tänne vuotavat luonnonlait? Kuullostaa tutulta. Sitä on vissiin luettu Isaac Asimovin kirjaa "Itse jumalat".

        https://fi.wikipedia.org/wiki/Itse_jumalat

        Nettikeskustelua seuratessa tulee muuten aika usein mieleen tuo kirjan sisällysluettelon muodostava Schillerin sitaatti kokonaisuudessaan: "Tyhmyyttä vastaan itse jumalatkin taistelevat turhaan". Jokainen sitten itse valitsee mihin asti taistelua kunkin keskustelun kohdalla jatkaa ja missä kohdassa toteaa taistelun jatkamisen turhaksi.

        Onpas ketju pompsahtanut pitkäksi, vaikka melko nopeasti sanottiin se, mikä on oleellista.

        Asimovin kirjassa oli hieno idea jakaa Schillerin osiin sitaatti lukujen nimiksi. Itse vuosikymmeniä sitten Asimov fanina pidin vain Säätiö trilogiaa Asimovilta parempana kuin Itse Jumalat opusta.

        Tietämättömyyttä vastaan voi taistella ja tälläkin palstalla on muutama ansiokkaasti kyseistä sotaa käyvä. Muutama perusnikki on sellaisia, että voi pääsääntöisesti luottaa tekstin oikeellisuuteen nikin nähdessään. Mutta jos joku kieltäytyy hyväksymästä fysiikan lakeja siksi, ettei itse kykene niitä ymmärtämään, ei kannata liikoja ruutia tuhlata. Jostain syystä tähtitiedepalsta tuntuu vetävän juuri sitä porukkaa.

        Luulin ensin martta00:n heittoa trolliksi. Hän on yksi niitä kirjoittajia, joilla on tapana vastata vain asiallisiin ketjuihin ja aina asiallisesti ja asiantuntevasti.


      • Hohhotihoijaa
        agnoskepo kirjoitti:

        "Ja sanoinko mä että valonnopeus ylittyy eritteessä? Sanoinko?
        Joten mitä sä siinä oikein mussutat? "
        Et sanonut, mutta mökelsit asiaan kuulumattomia tyhjänpäiväisyyksiä.

        Valon nopeus on nopeus, jolla valo kulkee väliaineessa. Yhteen kirjoitettuna valonnopeus on luonnon limiitti eli maksiminopeus, jolla informaatio voi avaruudessa siirtyä.

        Mikä tässä muka on niin vaikeaa?

        Anteeks nyt kauheesti kun mökisin valonnopeudesta keskusteltaessa asiaan kuulumattomia mökinöitä juurikin valonnopeudesta!? :D

        SORIIIIII!!!!

        Ja en tiiä mikä tässä on niin vaikeeta? Kerro?
        Sekö et mä mökisen valon nopeudesta jos arvon herrat keskusteleekin valonnopeudesta? :) Ja jonka mä jo aikaa sit myönsin, ja alunperin kirjotinkin erikseen!? Et kukas se nyt sit oikeen mökisee? :) Eikö teidän mielestänne valon nopeus liitykään valonnopeuteen?


      • HuijuiMAMMAAAA
        Kollimaattori kirjoitti:

        Multinikkimulkero on eritteessä siellä, jonne ei päivä paista, eikä valo kajasta. ; )

        Saitko? ;)

        :D


      • KannatKattoonJEE
        planeetta kirjoitti:

        Täällähän on päästy ihan asian ytimeen :D
        Jos todetaan vaikka, että ollaan eri mieltä ja lähdetään kukin omaan suuntaan ja pidetään viikko taukoa silleen, ettei jutella keskenään?
        Tehtäiskö tolleen?

        Miks?

        Just ku saatiin bileet mielenkiintosimmilleen?! :P :D

        Läheks sä aina nyrkkeilymatsistakin kotiin siinä vaiheessa ku ne esiintyjät alkaa hakkaamaan toisiaan? :D


      • TulihanNeLuettuaJoo
        agnoskepo kirjoitti:

        Onpas ketju pompsahtanut pitkäksi, vaikka melko nopeasti sanottiin se, mikä on oleellista.

        Asimovin kirjassa oli hieno idea jakaa Schillerin osiin sitaatti lukujen nimiksi. Itse vuosikymmeniä sitten Asimov fanina pidin vain Säätiö trilogiaa Asimovilta parempana kuin Itse Jumalat opusta.

        Tietämättömyyttä vastaan voi taistella ja tälläkin palstalla on muutama ansiokkaasti kyseistä sotaa käyvä. Muutama perusnikki on sellaisia, että voi pääsääntöisesti luottaa tekstin oikeellisuuteen nikin nähdessään. Mutta jos joku kieltäytyy hyväksymästä fysiikan lakeja siksi, ettei itse kykene niitä ymmärtämään, ei kannata liikoja ruutia tuhlata. Jostain syystä tähtitiedepalsta tuntuu vetävän juuri sitä porukkaa.

        Luulin ensin martta00:n heittoa trolliksi. Hän on yksi niitä kirjoittajia, joilla on tapana vastata vain asiallisiin ketjuihin ja aina asiallisesti ja asiantuntevasti.

        Olihan Asimoveissa asiaa, mut hitto ku se (tai suomentaja?) kirjotti nii tylsää tekstiä! :/ Tylsempään ei kyenny (Scifissä) ku Stanislaw Lem.


      • SatunnainenLukijaEiRek
        TulihanNeLuettuaJoo kirjoitti:

        Olihan Asimoveissa asiaa, mut hitto ku se (tai suomentaja?) kirjotti nii tylsää tekstiä! :/ Tylsempään ei kyenny (Scifissä) ku Stanislaw Lem.

        Säätiötrilogian suomennos ainakin oli kerrassaan kamalaa kökköä, joka pilasi tyystin lukunautinnon jo alkuunsa. Ei juuri tekemistä alkuperäisen kirjan kanssa. Jos sama lempo käänsi Lemiä, niin voih.


      • eineuvotellaenää
        agnoskepo kirjoitti:

        Onpas ketju pompsahtanut pitkäksi, vaikka melko nopeasti sanottiin se, mikä on oleellista.

        Asimovin kirjassa oli hieno idea jakaa Schillerin osiin sitaatti lukujen nimiksi. Itse vuosikymmeniä sitten Asimov fanina pidin vain Säätiö trilogiaa Asimovilta parempana kuin Itse Jumalat opusta.

        Tietämättömyyttä vastaan voi taistella ja tälläkin palstalla on muutama ansiokkaasti kyseistä sotaa käyvä. Muutama perusnikki on sellaisia, että voi pääsääntöisesti luottaa tekstin oikeellisuuteen nikin nähdessään. Mutta jos joku kieltäytyy hyväksymästä fysiikan lakeja siksi, ettei itse kykene niitä ymmärtämään, ei kannata liikoja ruutia tuhlata. Jostain syystä tähtitiedepalsta tuntuu vetävän juuri sitä porukkaa.

        Luulin ensin martta00:n heittoa trolliksi. Hän on yksi niitä kirjoittajia, joilla on tapana vastata vain asiallisiin ketjuihin ja aina asiallisesti ja asiantuntevasti.

        Taistelu tietämättömyyttä ja varsinkin tyhmyyttä vastaan on loputon suo.Kuin taistelu terrorismia vastaan,kun yksi solu on tuhottu seuraava putkahtaa esiin.Sama täällä,kun yksi trolli on upotettu seuraava nousee esiin.

        Eikä niiden kanssa kannata neuvotella.Maailmassa on ainakin yksi instituutio joka tietää miten toimia niiden kanssa:
        https://www.youtube.com/watch?v=iSffpqObMUY
        Eli tunnista ja tuhoa!Sopii hyvin trolleihin..


      • ÄläAnnaHuomiota
        eineuvotellaenää kirjoitti:

        Taistelu tietämättömyyttä ja varsinkin tyhmyyttä vastaan on loputon suo.Kuin taistelu terrorismia vastaan,kun yksi solu on tuhottu seuraava putkahtaa esiin.Sama täällä,kun yksi trolli on upotettu seuraava nousee esiin.

        Eikä niiden kanssa kannata neuvotella.Maailmassa on ainakin yksi instituutio joka tietää miten toimia niiden kanssa:
        https://www.youtube.com/watch?v=iSffpqObMUY
        Eli tunnista ja tuhoa!Sopii hyvin trolleihin..

        Olisikin noin helppoa. Keskustelun pitäminen asiallisena ja jankutuksen katkaiseminen on vaikeaa toteuttaa edes moderoiduilla palstoilla, ellei siellä esiinnytä rekisteröidyillä tunnuksilla, jotka on rekisteröintivaiheessa tunnistettu. Aikanaan Tiede - lehden puolella moderointi toimi varsin hyvin, mutta sielläkään ei tunnistamista vaadittu. Siksi muutama multinikki pääsi riehumaan palstalla vuosien ajan varastoon rekisteröimisensä nikkien avulla.

        Muutama vuosi sitten Tiede - lehden palsta uudistettiin rikkoen kaikki vanhoihin viesteihin osoittavat linkit ja samalla kaikkien tuntemat ja aktiivisesti keskustelua itse seuranneet moderaattorit saivat kenkää. Siihen käytännössä päättyi sen palstan taru. Hetken aikaa se jaksoi vanhalla maineellaan mutta varsin nopeasti palstaa hengissä pitäneet asiakirjoittajat totesivat tilanteen ja keksivät muuta ajanvietettä itselleen. Tiede - lehden palstoja moderoi tällä hetkellä ilmeisesti sama porukka joka on vastuussa Vauva - lehden keskustelufoorumin moderoinnista. Niille moderointi on pakollinen lisätyö varsinaisen tärkeämmän homman ohessa eikä niillä ole mitään hajua keskustelun sisällöstä.

        Moderoinnilla on täällä kaksi tehtävää: Pitää huoli siitä, että ei tule oikeusjuttuja tai niiden uhkaa (lue: niistä aiheutuvia kuluja) ja säilyttää palstan aktiivisuus siten, että mainoksia ladataan. Tiukka puuttuminen trollaukseen vähentäisi aktiivisuutta, joten sitä ei tapahdu. Aller elää mainoksilla joten treffipalstojen maksut ja mainostulot miinus moderoinnin, ylläpidon ja palvelimien kulut on palstan toimivuuden ainoa mittari heille.


      • ÄläAnnaHuomiota
        ÄläAnnaHuomiota kirjoitti:

        Olisikin noin helppoa. Keskustelun pitäminen asiallisena ja jankutuksen katkaiseminen on vaikeaa toteuttaa edes moderoiduilla palstoilla, ellei siellä esiinnytä rekisteröidyillä tunnuksilla, jotka on rekisteröintivaiheessa tunnistettu. Aikanaan Tiede - lehden puolella moderointi toimi varsin hyvin, mutta sielläkään ei tunnistamista vaadittu. Siksi muutama multinikki pääsi riehumaan palstalla vuosien ajan varastoon rekisteröimisensä nikkien avulla.

        Muutama vuosi sitten Tiede - lehden palsta uudistettiin rikkoen kaikki vanhoihin viesteihin osoittavat linkit ja samalla kaikkien tuntemat ja aktiivisesti keskustelua itse seuranneet moderaattorit saivat kenkää. Siihen käytännössä päättyi sen palstan taru. Hetken aikaa se jaksoi vanhalla maineellaan mutta varsin nopeasti palstaa hengissä pitäneet asiakirjoittajat totesivat tilanteen ja keksivät muuta ajanvietettä itselleen. Tiede - lehden palstoja moderoi tällä hetkellä ilmeisesti sama porukka joka on vastuussa Vauva - lehden keskustelufoorumin moderoinnista. Niille moderointi on pakollinen lisätyö varsinaisen tärkeämmän homman ohessa eikä niillä ole mitään hajua keskustelun sisällöstä.

        Moderoinnilla on täällä kaksi tehtävää: Pitää huoli siitä, että ei tule oikeusjuttuja tai niiden uhkaa (lue: niistä aiheutuvia kuluja) ja säilyttää palstan aktiivisuus siten, että mainoksia ladataan. Tiukka puuttuminen trollaukseen vähentäisi aktiivisuutta, joten sitä ei tapahdu. Aller elää mainoksilla joten treffipalstojen maksut ja mainostulot miinus moderoinnin, ylläpidon ja palvelimien kulut on palstan toimivuuden ainoa mittari heille.

        Tyhmyyttä vastaan käytävää taistelua kannattaa harjoittaa järkevästi. Ei ole tarpeellista sanoa sitä viimeistä sanaa. Jankuttaja nimenomaan yrittää jatkaa jankuttamista ja viimeisen sanan sanoja joutuisi aina hänelle vastaamaan eli jatkamaan loputtomiin. Jos joku kirjoittelee höpöjä eikä näytä viisastuvan saamistaan vastauksista niin sitten sille ei kannata vastailla. Trollia ei kannata ruokkia, vaan siltä pitäisi ottaa tikkari pois.

        Jos tämä olisi esimerkiksi rokotusaiheinen keskustelupalsta niin silloin olisi enemmän perusteita käydä jättämässä muistutusvastaus pidempien jankutusketjujen virheellisiin väitteisiin esimerkiksi viikon välein. Kun tämän palstan viesteillä ei johdeta harhaan lasten vanhempia niin sitäkään perustetta ei löydy.

        Älkää antako trolleille huomiota vaan ottakaa niiltä tikkari pois. Sama kohtelu olipa trollauksen syynä tarkoituksellinen kiusanteko, mt-ongelmat tai pelkkä itsepäisyys nojatuolifyysikon puolustaessa omaa "teoriaansa" faktoista välittämättä.


    • 8765

      Yritänpä tiivistää kysymyksen. Jos kiinteää kappaletta kopautetaan toisesta päästä, miksi informaatio ei siirry välittömästi pisteestä A pisteeseen B? Välittömästi tarkoittaa valoa nopeammin, vaikka kappale itsessään ei liiku lähellekään valonnopeutta.
      Miksi kiinteä kappale ei olekaan kiinteä?

      • Kopautuksen informaatio etenee väliaineelle ominaisella äänennopeudella.

        Oliko tarpeeksi tiivistetty vastaus? Kullekin väliaineelle ominaisia mekaanisen signaalin siirtonopeuksia löydät taulukkokirjasta myös sähköisessä muodossa.


      • Kolli jo vastasikin, mutta toiseen kysymykseen: "Miksi kiinteä kappale ei olekaan kiinteä? "

        Siksi, että atomit ovat tyhjää höttöä, jotka vaikuttavat toisiinsa sähkömagneettisten kenttien välityksillä. Koska näiden kenttien aiheuttama voima ei ole ääretön, kaikki atomeista koostuvat rakenteet joustavat.
        Atomijono makromaailmaan tuotuna ei ole kuin jono biljardipalloja, vaan pikemminkin kuin jono pesusieniä, jotka joustavat törmäyksissä toisiinsa.


      • Valonnopeus.info

        Täysin kiinteitä kappaleita ei fysiikan lakien mukaan ole, mutta JOS oletetaan että olisi, niin ei se informaatio kulje valonnopeutta vaikka jotain "äärettömän"kappaletta työntäisi jonkun matkaa.

        Kappale kulkee sillä nopeudella, millä sitä työnnettään tai vedetään.
        Informaatio eli esim. näköhavainto siirrosta saataisiin vasta jälkeenpäin, kun valo saapuu perille.


      • Valonnopeus.info
        Valonnopeus.info kirjoitti:

        Täysin kiinteitä kappaleita ei fysiikan lakien mukaan ole, mutta JOS oletetaan että olisi, niin ei se informaatio kulje valonnopeutta vaikka jotain "äärettömän"kappaletta työntäisi jonkun matkaa.

        Kappale kulkee sillä nopeudella, millä sitä työnnettään tai vedetään.
        Informaatio eli esim. näköhavainto siirrosta saataisiin vasta jälkeenpäin, kun valo saapuu perille.

        ...siis, "äärettömän pitkää" kappaletta..


      • Valonnopeus.info
        Valonnopeus.info kirjoitti:

        Täysin kiinteitä kappaleita ei fysiikan lakien mukaan ole, mutta JOS oletetaan että olisi, niin ei se informaatio kulje valonnopeutta vaikka jotain "äärettömän"kappaletta työntäisi jonkun matkaa.

        Kappale kulkee sillä nopeudella, millä sitä työnnettään tai vedetään.
        Informaatio eli esim. näköhavainto siirrosta saataisiin vasta jälkeenpäin, kun valo saapuu perille.

        Prrkelllee .. korjataan vielä. juuri informaatio kulkee valonnopeutta, mutta ei se kappale.
        Informaatio ei siis kulje YLIvalonnopeutta !

        ..ompas vaikeaa tää kirjoittaminen...


    Ketjusta on poistettu 2 sääntöjenvastaista viestiä.

    Luetuimmat keskustelut

    1. Pupuhuhdasta löytyi lähes sadan kilon miljoonalasti huumeita

      Pupuhuhdasta löytyi lähes sadan kilon miljoonalasti huumeita – neljä Jyväskylän Outlaws MC:n jäsentä vangittu: "Määrät p
      Jyväskylä
      55
      1816
    2. Persut petti kannattajansa, totaalisesti !

      Peraujen fundamentalisteille, vaihtkaa saittia. Muille, näin sen näimme. On helppo luvata kehareille, eikä ne ymmärrä,
      Maailman menoa
      43
      1593
    3. Ei luottoa lakko maahan

      Patria menetti sovitun ksupan.
      Suomen Keskusta
      51
      1542
    4. Nähtäiskö ylihuomenna taas siellä missä viimeksikin?

      Otetaan ruokaöljyä, banaaneita ja tuorekurkkuja sinne messiin. Tehdään taas sitä meidän salakivaa.
      Ikävä
      4
      1485
    5. Sinäkö se olit...

      Vai olitko? Jostain kumman syystä katse venyi.. Ajelin sitten miten sattuu ja sanoin ääneen siinä se nyt meni😅😅... Lis
      Ikävä
      5
      1478
    6. Housuvaippojen käyttö Suomi vs Ulkomaat

      Suomessa housuvaippoja aletaan käyttämään vauvoilla heti, kun ne alkavat ryömiä. Tuntuu, että ulkomailla housuvaippoihin
      Vaipat
      5
      1384
    7. Hyvää yötä ja kauniita unia!

      Täytyy alkaa taas nukkumaan, että jaksaa taas tämän päivän haasteet. Aikainen tipu madon löytää, vai miten se ärsyttävä
      Tunteet
      7
      1298
    8. Lepakot ja lepakkopönttö

      Ajattelin tehdä lepakkopöntön. Tietääkö joku ovatko lepakot talvella lepakkopöntössä ´vai jossain muualla nukkumassa ta
      11
      1263
    9. Revi siitä ja revi siitä

      Enkä revi, ei kiinnosta hevon vittua teidän asiat ja elämä. Revi itte vaan sitä emborullaas istuessas Aamupaskalla
      Varkaus
      3
      1142
    10. Kello on puoliyö - aika lopettaa netin käyttö tältä päivältä

      Kello on 12, on aika laittaa luurit pöydälle ja sallia yörauha kaupungin asukkaille ja työntekijöille. It is past midni
      Hämeenlinna
      3
      1123
    Aihe